Re: [obm-l] Ajuda Sobre um site (estranho!!)

2002-12-20 Por tôpico Paulo Santa Rita
Ola Duda, Dirichlet e demais colegas
desta lista ... OBM-L,

A ideia de traduzir problemas olimpicos, sobretudo os de nivel 
universitario, e uma forma de contribuir para o incremento e consolidacao 
deste nivel de olimpiadas, aqui no Brasil, pois supomos que assim - em 
portugues - sera acessivel a um publico maior ...

O que desejamos e que os estudantes que se preparam para este tipo de 
olimpiada tenham um material de referencia, que prof's possam usa-lo em 
cursos preparatorios para estas competicoes e mesmo como inspiracao para os 
prof que queiram elaborar questoes honestas e nao-burocraticas, nos 
primieors anos da faculdade.

Toda ajuda e bem-vinda e se estamos felizes com o que estamos fazendo, 
ficaremos igualmente felizes se outros fizerem com antecedencia : o que 
importa e contribuir para um maior aperfeicoamento e consolidacao do 
movimento olimpico !

Um Abraco a Todos !

E aproveito a oportunidade para desejar um feliz natal e excelente ano novo 
a todos os membros  !

Com os melhores votos
de paz profunda, sou
Paulo Santa Rita
6,1256,201202







_
MSN Hotmail, o maior webmail do Brasil. http://www.hotmail.com

=
Instruções para entrar na lista, sair da lista e usar a lista em
http://www.mat.puc-rio.br/~nicolau/olimp/obm-l.html
O administrador desta lista é [EMAIL PROTECTED]
=


[obm-l] Re: [obm-l] um pouco de combinatória

2002-12-21 Por tôpico Paulo Santa Rita
Ola Rafael e demais colegas
desta lista ... OBM-L,

1) A formula que voce apresentou fornece o NUMERO DE SOLUCOES INTEIRAS E 
NAO NEGATIVAS, isto e, sao as solucoes nas quais uma
ou mais das variaveis pode(m) assumir o valor zero.

Isto significa, claramente, que se (X1,X2,...,Xn) for uma solucao nao 
negativa, a solucao (Y1,Y2,...,Yn) tal que Yi=Xi+1 sera uma solucao 
positiva, tal como voce busca ... De maneira geral, as solucoes INTEIRAS 
NAO NEGATIVAS de :

Y1 + Y2 + Y3 + ... + Yn = K - N

mantem uma bijecao com as solucoes POSITIVAS de :

X1 + X2 + X3 + ... + Xn = K

Bom. Daqui voce prossegue ...

2) O seu erro foi usar os fatores primos ... Um produto tal como o que voce 
deseja pode ser expresso como :

P = (5^A)*(6^B)*(7^C)*(9^D) onde A e B devem ser escolhidos em {0,1}, C em 
{0,1,2} e D em {0,1,2,3}. Observando  que a escolha de um nao impoe nenhum 
condicao sobre a escolha de qualquer outro, isto e, que as escolhas sao 
independentes, basta entao se lembrar do Principio Multiplicativo da Analise 
Combinatoria.

Bom. Daqui voce prossegue ...

Um Abraco
Paulo Santa Rita
7,2223,211202






From: rafaelc.l [EMAIL PROTECTED]
Reply-To: [EMAIL PROTECTED]
To: [EMAIL PROTECTED]
Subject: [obm-l] um pouco de combinatória
Date: Sat, 21 Dec 2002 20:57:02 -0200


 Pessoal:

  - É sabido que o número de soluções inteiras positivas
de uma equação do tipo: x1+x2+x3+...+xn=K , é:

 (n+K-1)!/(n-1)!.K!

Eu queria saber o número de soluções inteiras e positivas
sem que nenhuma das variáveis x1,x2...xn pudesse ser nula.


- quantos números diferentes podem ser formados
multiplicando alguns(ou todos) dos números
1,5,6,7,7,9,9,9,?

 Eu tentei fazer colocando o produto deles em fatores
primos: 2.3^7.5.7^2, aí achei todos os produtos
possíveis: 2.8.2.3=96. Mas não é a resposta correta, pois
tem produtos que dão o mesmo número. Pergunta: como vou
saber quais produtos dão o mesmo número?





__
Venha para a VilaBOL!
O melhor lugar para você construir seu site. Fácil e grátis!
http://vila.bol.com.br


=
Instruções para entrar na lista, sair da lista e usar a lista em
http://www.mat.puc-rio.br/~nicolau/olimp/obm-l.html
O administrador desta lista é [EMAIL PROTECTED]
=



_
MSN Hotmail, o maior webmail do Brasil. http://www.hotmail.com

=
Instruções para entrar na lista, sair da lista e usar a lista em
http://www.mat.puc-rio.br/~nicolau/olimp/obm-l.html
O administrador desta lista é [EMAIL PROTECTED]
=



Re: [obm-l] Re:

2002-12-22 Por tôpico Paulo Santa Rita
Oi Eder, Oi Paulo,
Ola Prof Morgado e demais
colegas desta lista ... OBM-L,

E verdade, alguem olhou na home page do John Scholes e traduziu 
literalmente, sem antes resolver o problema. Dai surgiu o enunciado errado :

Se p(x)=ax^2 + bx + c e, alem disso, P(x)=x nao tem solucao real, entao 
p(p(x))=0 nao tem solucao real.

O enunciado correto e :

Se p(x)=ax^2 + bx + c e, alem disso, P(x)=x nao tem solucao real, entao 
p(p(x))=x nao tem solucao real.

O Paulo Rodrigues e o Salvador ja apresentaram belas solucoes, baseadas na 
nocao de continuidade. Sobre a duvida de porque tem que ser p(x)  x ou p(x) 
 x para todo x real, basta imaginar os graficos :

Se p(x)=x nao tem solucao real entao e porque os graficos destas funcoes nao 
se interceptam, certo ? E isto so pode ocorrer de duas maneiras :

1) a  0 e p(x)  x para todo x ( grafico de p(x) acima  de y=x )
2) a  0 e p(x)  x para todo x ( grafico de p(x) abaixo de y=x )

Eu me dou por satisfeito imaginando os graficos. Mas se voce quer uma 
demonstracao formal disso, um esboco de uma tal prova pode ser :

p(x)=x nao tem solucao real = ax^2 + bx + c = x nao tem solucao
ax^2 + (b-1)x + c =0 nao tem solucao real.
(b-1)^2 -4ac  0. = (b-1)^2  4ac. Como (b-1)^2 = 0 segue que :
4ac  0 = a e c tem o mesmo sinal.

Supondo a  0, se para algum x1, p(x1)  x1 tome um x2 tal que
p(x2)  x2 ( x2 existe pois lim p(x)=+inf ). A continuidade de p(x) garante 
a intereseccao de y=p(x) com y=x, isto e, solucao para p(x)=x, um absurdo !

Procedimento semelhante para a  0.

Aceita um conselho ? Se voce for rigoroso demais e perder muito tempo com 
provas simples assim, voce prende o seu pensamento e a sua imaginacao nao 
decola. Primeiro se concentre nas coisas fundamentais, depois voce prova as 
acessorias.

Um abraco
Paulo Santa Rita
1,1950,221202






From: Eder [EMAIL PROTECTED]
Reply-To: [EMAIL PROTECTED]
To: [EMAIL PROTECTED]
Subject: Re: [obm-l] Re:
Date: Sun, 22 Dec 2002 10:31:09 -0200

Oi Paulo,

Acredito que minha tradução estava certa ou pelo menos não comprometia
muito.O que estava errado era o p(p(x))=0 no site do John Scholes...
- Original Message -
From: Paulo Santa Rita [EMAIL PROTECTED]
To: [EMAIL PROTECTED]
Sent: Saturday, December 21, 2002 6:16 PM
Subject: Re: [obm-l] Re:


 Ola Prof Morgado e demais
 colegas desta lista ... OBM-L,

 O contra-exemplo do Prof Morgado, dado abaixo, de forma elegante
 efetivamente encerra a questao. O enunciado esta incorreto. Considerem
agora
 o problema :

 Sejam a, b e c tres reais quaisquer. Se p(x)=ax^2 + bx + c  e
 p(x)=x nao tem raiz real entao p(p(x))=x nao tem raiz real.

 Alias, esta discussao, indiretamente, mostra o quao capciosas podem ser 
as
 traducoes, nao podendo nunca se resumirem a mera transposicao literal do
 enunciado de um idioma para outro ...

 Este espirito natalino que nos invade, me levou a pensar em Jesus, que 
os
 cristaos consideram O Cristo Prometido. Depois, por associacao de 
ideias,
me
 lembrei de um dos Profetas que o antecederam, Salomao. E dai a um dos
 proverbios deste Profeta : Nao respondas ao tolo segundo a sua
estulticia,
 para que nao tambem nao te tornes semelhante a ele

 Um abraco a Todos !
 Paulo Santa Rita
 7,1812,211202






 From: A. C. Morgado [EMAIL PROTECTED]
 Reply-To: [EMAIL PROTECTED]
 To: [EMAIL PROTECTED]
 Subject: Re: [obm-l] Re:
 Date: Sat, 21 Dec 2002 00:30:59 -0200
 
 Vou tentar encerrar a discussao. Tome p(x) = x^2 + 4x + 3. A equaçao 
p(x)
=
 x reduz-se a  x^2 + 3x + 3 = 0 ue nao tem raiz real pois seu
discriminante
 eh negativo (-3). Como p(-2) = -1, p(p(-2)) = p(-1) = 0, NAO EH VERDADE
que
 p(p(x))=0 nao possua raiz real, pois -2 eh raiz da referida equaçao.
Assim
 como esse, ha muitos contraexemplos que podem ser dados (vejam mensagem
de
 Salvador Addas Zanata).
 Peço desculpas a todos pelo contraexemplo que mandei em mensagens
 anteriores, pois ele estah errado.
 Morgado
 
 Eder wrote:
 
 Esse problema foi retirado do site do John Scholes e o enunciado é:
 
 
 
 Define p(x)=ax²+bx+c.If p(x)=x has no real roots,prove that p(p(x))=0
has
 no real roots.
 
 
 
  - Original Message -
 
  From: A. C. Morgado mailto:[EMAIL PROTECTED]
 
  To: [EMAIL PROTECTED] mailto:[EMAIL PROTECTED]
 
  Sent: Friday, December 20, 2002 5:12 PM
 
  Subject: Re: [obm-l] Re:
 
 
 
 
  Wagner wrote:
 
  Oi pessoal !
 
 
 
  2)Vou supor que a,b,c,x sejam números reais e que a é diferente
  de zero.
 
   Prove que se p(x)=x não tem nenhuma raiz real, então o módulo 
da
  ordenada do máximo ou do mínimo de f(x)=p(p(x)) é maior que o
  módulo da ordenada do máximo ou do mínimo de g(x)=p(x) -x e
  depois prove que o sinal da derivada de segunda ordem de
  f(x)=p(p(x)) e de g(x)=p(x) -x é o mesmo, assim se a segunda
  função não tem raiz real a primeira também não tem.
 
 
 
  Prova: Primeiro vou provar a segunda hipótese: g '' (x) =2a  ;
   f(x)= a(ax^2 +bx +c)^2 +b(ax^2 +bx +c) +c =
 
  f ' (x) =2a(ax^2

[obm-l] Um livro Mararavilhoso !

2002-12-25 Por tôpico Paulo Santa Rita
Ola Pessoal,

Ha pouco tempo atras eu ganhei um livro e - apos estuda-lo - cheguei a 
conclusao que - pelo menos - e muito bom, para nao dizer excelente ... Ele 
trata de teoremas elementares nao encontrados nos livros didaticos 
tradicionais e apresenta, em geral, provas elementares destes teoremas.

Qualquer bom aluno - serio e disciplinado - do fim do segundo grau ou do 
inicio da faculdade pode le-lo sem maiores dificuldades. Ele e centrado em 
resultados e provas obtidas pelo Paul Erdos. Tenho certeza que e uma 
excelente aquisicao para qualquer aluno e, mesmo, para professores.

O Livro e :

Proofs from the book
Martin Aigner e Gunter Ziegler
Springer-Verlag

Agora fiquei sabendo que foi traduzido para o protugues:

As provas estao n'O LIVRO
Martin Aigner e Gunter Ziegler
Editora Edgard Blucher

A expressao n'O LIVRO e porque o Erdos acreditava que havia um LIVRO 
imaginario, escrito por Deus, que a intuicao matematica poderia acessar e no 
qual estariam as melhores e mais bonitas provas dos teoremas.

Como exemplo cito o TEOREMA DE SILVESTER :

Se dispormos N ( N  2 ) pontos em um plano de forma que eles nao estejam em 
uma mesma reta, entao havera uma reta que contera EXATAMENTE dois deles.

OU SEJA :

Nao e possivel dispor N pontos ( nao alinhados )em um plano de forma que que 
toda reta que passe por dois deles passe tambem por um terceiro.

A prova que o Kelly da e simplesmente divina ( digna, portanto, de estar n'O 
LIVRO do Erdos ), mas o Conway fez uma generalizacao e provou esta 
generalizacao em ... 1 linha ! Nao ha palavras para descrever tamanha beleza 
!

Esse livro e realmente uma sinfonia e so mesmo Beethoven poderia fazer algo 
melhor.

Um Grande Abraco a Todos !
Paulo Santa Rita
4,1651,251202







_
MSN Messenger: converse com os seus amigos online. 
http://messenger.msn.com.br

=
Instruções para entrar na lista, sair da lista e usar a lista em
http://www.mat.puc-rio.br/~nicolau/olimp/obm-l.html
O administrador desta lista é [EMAIL PROTECTED]
=


Re: [obm-l] Teorema de Silvester

2002-12-26 Por tôpico Paulo Santa Rita
Ola Jose Francisco e demais
colegas desta lista ... OBM-L,

Oi Francisco ! Obrigado pela correcao gramatical : doravante estarei mais
atento.

A prova do Kelly e a que o Claudio reproduziu abaixo, inclusive com notacao
semelhante. E necessario corrigir apenas :

1) E necessario impor que N - o numero de pontos - seja tal que N  2,
pois se nao houver esta caracteristica o conjunto dos (P,QR) sera vazio.

2) Nunca e necessario re-nomear os pontos. O Kelly usa Q como pe da
perpendicularao tracada por P e P1 como o ponto mais proximo de Q.

3) Nao e correto supor que ha apenas um par com distancia minima : pode
haver mais de um !

A dualidade que se observa neste caso e uma consequencia das coordenadas
homogeneas.

A generalizacao do Conway e a seguinte :

Seja X um conjunto con N elementos (N2) e sejam A1, A2, ...,Am subconjuntos
proprios de X tais que todo par de elementos de X esta contido em
precisamente um dos Ai. Entao M = N.

Eu posso REPRODUZIR A PROVA do Conway, mas talvez seja interessante o 
pessoal tentar descobrir a prova simples que ele achou.

Um Abraco
Paulo Santa Rita
5,0145,271202

From: Cláudio \(Prática\) [EMAIL PROTECTED]
Reply-To: [EMAIL PROTECTED]
To: [EMAIL PROTECTED]
Subject: Re: [obm-l] Teorema de Silvester
Date: Thu, 26 Dec 2002 20:30:52 -0200

Uma solução para este problema (não deve ser a de Conway, pois é bem mais
longa do que uma linha) usa o conceito de distância de ponto a reta e chega
a uma contradição:

Dado o conjunto C dos N pontos, considere o conjunto de todos os pares ( 
P
, QR ) de ponto (P) e reta (QR) que não contém o ponto (P, Q e R
pertencentes a C).

Este conjunto não é vazio, pois nem todos os pontos de C pertencem a uma
mesma reta.

Tome o par cuja distância do respectivo ponto à respectiva reta é a menor
possível - digamos ( P , QR ). Então QR será a reta desejada.

Seja P1 o pé da perpendicular à QR traçada a partir de P. Se houver um
terceiro ponto do conjunto C na reta QR então pelo menos dois destes
pontos estarão de um mesmo lado de P1.

Re-nomeando os pontos, se necessário, chame de Q o ponto mais próximo de P1
(Q pode até coincidir com P1) e R o outro ponto situado do mesmo lado que Q
em relação a P1.

Neste caso, o par ( Q , PR ) será tal que a distância de Q a PR será menor
do que a distância de P a QR (faça o desenho), o que contradiz a escolha
inicial do par ( P , QR ).Ola Jose Francisco e demais
colegas desta lista ... OBM-L,



Curiosidade: Existe também o resultado dual:
Se dispusermos de N (N2) retas em um plano tais que nem todas passam por 
um
mesmo ponto, então existirá um ponto deste plano no qual incidirão
exatamente duas retas.

Um abraço,
Claudio.

- Original Message -
From: Jose Francisco Guimaraes Costa [EMAIL PROTECTED]
To: [EMAIL PROTECTED]
Sent: Thursday, December 26, 2002 8:50 PM
Subject: [obm-l] Teorema de Silvester


Santa Rita,

Não nos mate de curiosidade.

Qual a demonstração de Conway?

E, se não forem necessários muitos bits para descrevê-la - acho que não
serão, já que uma demonstração divinamente elegante tem que ser
necessariamente breve - também a de Kelly.

JF

PS: Uma pequena e humilde contribuição para a elegância vernácula: onde 
está
Se dispormos N ( N  2 ) pontos... deveria estar Se dispusermos N (N2)
pontos...

JF (aluno destacado do Mestre Aurélio Buarque de Holanda - a estória do
destacado certamente seria considerada off topic pelo N)

- Original Message -
From: Paulo Santa Rita [EMAIL PROTECTED]
To: [EMAIL PROTECTED]
Sent: Wednesday, December 25, 2002 2:53 PM
Subject: [obm-l] Um livro Mararavilhoso !


 Ola Pessoal,

 Ha pouco tempo atras eu ganhei um livro e - apos estuda-lo - cheguei a
(...)

 Como exemplo cito o TEOREMA DE SILVESTER :

 Se dispormos N ( N  2 ) pontos em um plano de forma que eles nao 
estejam
em
 uma mesma reta, entao havera uma reta que contera EXATAMENTE dois deles.

 OU SEJA :

 Nao e possivel dispor N pontos ( nao alinhados )em um plano de forma que
que
 toda reta que passe por dois deles passe tambem por um terceiro.

 A prova que o Kelly da e simplesmente divina ( digna, portanto, de estar
n'O
 LIVRO do Erdos ), mas o Conway fez uma generalizacao e provou esta
 generalizacao em ... 1 linha ! Nao ha palavras para descrever tamanha
beleza
 !

 Esse livro e realmente uma sinfonia e so mesmo Beethoven poderia fazer
algo
 melhor.

 Um Grande Abraco a Todos !
 Paulo Santa Rita
 4,1651,251202


=
Instruções para entrar na lista, sair da lista e usar a lista em
http://www.mat.puc-rio.br/~nicolau/olimp/obm-l.html
O administrador desta lista é [EMAIL PROTECTED]
=

=
Instruções para entrar na lista, sair da lista e usar a lista em
http://www.mat.puc-rio.br/~nicolau/olimp/obm-l.html
O administrador desta lista é [EMAIL PROTECTED

Re: [obm-l] Teorema de Silvester

2002-12-28 Por tôpico Paulo Santa Rita
Ola Dudu e demais colegas
desta lista ... OBM-L,

E ai Dudu ? Tudo Legal ?
Fico contente em ver voce participar da lista !

Leia com mais atencao o Teorema do Conway. Nao e o que voce esta pensando 
...

A1, A2, A3, ..., Am sao subconjuntos proprios quaisquer tais que qualquer 
conbinacao de dois elementos de X esta PRECISAMENTE em um
dos Ai. O Conway comeca a prova dele assim :

Se a pertence a X, seja Ra o numero de subconjuntos Ai, i em
{1,2,...,m }, tal que a pertence a Ai. Claramente 2 = Ra  m ...

Um Abraco
Paulo Santa Rita
7,1425,281202






From: Eduardo Fischer [EMAIL PROTECTED]
Reply-To: [EMAIL PROTECTED]
To: [EMAIL PROTECTED]
Subject: Re: [obm-l] Teorema de Silvester
Date: Sat, 28 Dec 2002 12:56:05 -0200

Basta tomarmos os N conjuntos unitários e os pares ( que serão três no
mínimo ), sendo maior que N a soma. Acho que é isso.
Fischer

- Original Message -
From: Paulo Santa Rita [EMAIL PROTECTED]
To: [EMAIL PROTECTED]
Sent: Friday, December 27, 2002 1:51 AM
Subject: Re: [obm-l] Teorema de Silvester


 Ola Jose Francisco e demais
 colegas desta lista ... OBM-L,

 Oi Francisco ! Obrigado pela correcao gramatical : doravante estarei 
mais
 atento.

 A prova do Kelly e a que o Claudio reproduziu abaixo, inclusive com
notacao
 semelhante. E necessario corrigir apenas :

 1) E necessario impor que N - o numero de pontos - seja tal que N  2,
 pois se nao houver esta caracteristica o conjunto dos (P,QR) sera vazio.

 2) Nunca e necessario re-nomear os pontos. O Kelly usa Q como pe da
 perpendicularao tracada por P e P1 como o ponto mais proximo de Q.

 3) Nao e correto supor que ha apenas um par com distancia minima : pode
 haver mais de um !

 A dualidade que se observa neste caso e uma consequencia das coordenadas
 homogeneas.

 A generalizacao do Conway e a seguinte :

 Seja X um conjunto con N elementos (N2) e sejam A1, A2, ...,Am
subconjuntos
 proprios de X tais que todo par de elementos de X esta contido em
 precisamente um dos Ai. Entao M = N.

 Eu posso REPRODUZIR A PROVA do Conway, mas talvez seja interessante o
 pessoal tentar descobrir a prova simples que ele achou.

 Um Abraco
 Paulo Santa Rita
 5,0145,271202

 From: Cláudio \(Prática\) [EMAIL PROTECTED]
 Reply-To: [EMAIL PROTECTED]
 To: [EMAIL PROTECTED]
 Subject: Re: [obm-l] Teorema de Silvester
 Date: Thu, 26 Dec 2002 20:30:52 -0200
 
 Uma solução para este problema (não deve ser a de Conway, pois é bem 
mais
 longa do que uma linha) usa o conceito de distância de ponto a reta e
chega
 a uma contradição:
 
 Dado o conjunto C dos N pontos, considere o conjunto de todos os 
pares
(
 P
 , QR ) de ponto (P) e reta (QR) que não contém o ponto (P, Q e R
 pertencentes a C).
 
 Este conjunto não é vazio, pois nem todos os pontos de C pertencem a 
uma
 mesma reta.
 
 Tome o par cuja distância do respectivo ponto à respectiva reta é a 
menor
 possível - digamos ( P , QR ). Então QR será a reta desejada.
 
 Seja P1 o pé da perpendicular à QR traçada a partir de P. Se houver um
 terceiro ponto do conjunto C na reta QR então pelo menos dois destes
 pontos estarão de um mesmo lado de P1.
 
 Re-nomeando os pontos, se necessário, chame de Q o ponto mais próximo 
de P1
 (Q pode até coincidir com P1) e R o outro ponto situado do mesmo lado 
que Q
 em relação a P1.
 
 Neste caso, o par ( Q , PR ) será tal que a distância de Q a PR será 
menor
 do que a distância de P a QR (faça o desenho), o que contradiz a 
escolha
 inicial do par ( P , QR ).Ola Jose Francisco e demais
 colegas desta lista ... OBM-L,

 
 Curiosidade: Existe também o resultado dual:
 Se dispusermos de N (N2) retas em um plano tais que nem todas passam 
por
 um
 mesmo ponto, então existirá um ponto deste plano no qual incidirão
 exatamente duas retas.
 
 Um abraço,
 Claudio.
 
 - Original Message -
 From: Jose Francisco Guimaraes Costa [EMAIL PROTECTED]
 To: [EMAIL PROTECTED]
 Sent: Thursday, December 26, 2002 8:50 PM
 Subject: [obm-l] Teorema de Silvester
 
 
 Santa Rita,
 
 Não nos mate de curiosidade.
 
 Qual a demonstração de Conway?
 
 E, se não forem necessários muitos bits para descrevê-la - acho que não
 serão, já que uma demonstração divinamente elegante tem que ser
 necessariamente breve - também a de Kelly.
 
 JF
 
 PS: Uma pequena e humilde contribuição para a elegância vernácula: onde
 está
 Se dispormos N ( N  2 ) pontos... deveria estar Se dispusermos N 
(N2)
 pontos...
 
 JF (aluno destacado do Mestre Aurélio Buarque de Holanda - a est
ória do
 destacado certamente seria considerada off topic pelo N)
 
 - Original Message -
 From: Paulo Santa Rita [EMAIL PROTECTED]
 To: [EMAIL PROTECTED]
 Sent: Wednesday, December 25, 2002 2:53 PM
 Subject: [obm-l] Um livro Mararavilhoso !
 
 
   Ola Pessoal,
  
   Ha pouco tempo atras eu ganhei um livro e - apos estuda-lo - cheguei 
a
 (...)
  
   Como exemplo cito o TEOREMA DE SILVESTER :
  
   Se dispormos N ( N  2 ) pontos em um plano de forma que eles nao
 estejam
 em
   uma mesma reta, entao havera uma reta que

[obm-l] Re: [obm-l] Triângulos-cont.

2003-01-11 Por tôpico Paulo Santa Rita
Ola Prof Jose Claudio e demais
colegas desta lista ... OBM-L,

Bem-Vindo a lista OBM-L Prof Jose Claudio ! É bom ve-lo participar !

São notaveis estes pontos de intersecção de cevianas, não ? Exemplos bem 
conhecidos sao o ortocentro ( alturas ), o incentro ( bissetrizes internas ) 
e o baricentro ( medianas ).

Quais são as condições necessarias e suficientes para que tres cevianas, 
cada uma partindo de um vertice, tenham um ponto comum ?

Seria o Teorema-Recíproco do Teorema de Ceva ?

Um Abraço a Todos !
Paulo Santa Rita
7,2327,110103

From: Claudio [EMAIL PROTECTED]
Reply-To: [EMAIL PROTECTED]
To: [EMAIL PROTECTED]
Subject: [obm-l] Re: [obm-l] Re: [obm-l] Triângulos-cont.
Date: Mon, 6 Jan 2003 18:08:53 -0200


   Sim, é verdade que se duas bissetrizes se interceptam num ponto, a 
terceira também passa por esse ponto. Mas nem sempre o poto de tangência 
entre a circunferência inscrita num triângulo e um dos seus lados 
corresponde à intersecção entre esse lado e a bissetriz do ângulo oposto. 
Isso só ocorre se o triângulo for isósceles ou equilátero.

   Se fosse verdade, poderíamos usar seus argumentos para provar que 
todos os triângulo são isósceles ou equiláteros, ou seja, que não existem 
triângulos escalenos, o que logicamente nao é verdade.




_
MSN Hotmail, o maior webmail do Brasil. http://www.hotmail.com

=
Instruções para entrar na lista, sair da lista e usar a lista em
http://www.mat.puc-rio.br/~nicolau/olimp/obm-l.html
O administrador desta lista é [EMAIL PROTECTED]
=



[obm-l] RE: [obm-l] classifiquem a função

2003-01-14 Por tôpico Paulo Santa Rita
Ola Olavo, Prof Morgado e demais
membros desta lista ... OBM-L,

Com respeito a mensagem do Prof Morgado e as Barbas Brancas do Prof Olavo 
talvez seja interessante perguntar qual a razao deste nome : transformação 
homográfica. Literamente seria homo=mesmo, grafica=grafico. Mesmo grafico 
?

Se Y=(Ax+B)/(Cx+D) entao Y=(A/C)*((ACx+BC)/(ACx+AD))
Y=(A/C)*((ACx+AD+BC-AD)/(ACx+AD))=(A/C)*(1  + (BC-AD)/(ACx+AD))
Y=A/C  +  (A/C)*((BC-AD)/(ACx+AD))

Se BC - AD e diferente de 0 entao :

x - +INF ou  x - -INF entao Y - A/C
(Por valores maiores ou menores que A/C, dependendo de sinais )

x - -D/C entao Y - +INF ou Y - -INF
(Para +INF ou -INF dependendo do sentido da aproximacao de x )

Ou seja, claramente que haverao duas assintotas, uma vertical e outra 
horizontal. Os graficos sao, portanto, muito parecidos. Seria esta a razao 
do nome homograficas ?

Seria possivel fazer coisas interessantes com estas funcoes ?

Seja Y = (2x+3)/(2-3x). Qual e a imagem de [1/3, 1] pela funcao Y, isto e, 
quem e Y([1/3, 1]) ? Qual e a imagem inversa de [-1, -1/3], isto e, quem e 
Y^-1([-1, -1/3]) ?

Agora, considere que as assintotas sao espelhos e que os dois ramos da 
funcao acima se refletem em ambos os espelhos. Existe uma EQUACAO
F(X,Y)=0 que tenha tal grafico ?

Um Abraco
Paulo Santa Rita
3,1708,140103

From: Antonio Neto [EMAIL PROTECTED]
Reply-To: [EMAIL PROTECTED]
To: [EMAIL PROTECTED]
Subject: Re: [obm-l] RE: [obm-l] classifiquem a função
Date: Tue, 14 Jan 2003 17:52:08 +

  Do alto das minhas venerandas barbas branquinhas, endosso as 
homograficas. Abracos, olavo.






From: A. C. Morgado [EMAIL PROTECTED]
Reply-To: [EMAIL PROTECTED]
To: [EMAIL PROTECTED]
Subject: Re: [obm-l] RE: [obm-l] classifiquem a função
Date: Sun, 12 Jan 2003 22:08:53 -0200

Tirando uma palavra do ostracismo:
essas funçoes que sao quocientes de polinomios de primeiro grau  ( de 
raizes diferentes ) tem como graficos hiperboles equilateras com uma 
assintota horizontal e outra vertical  sao (ou eram, ha muito tempo nao 
vejo ninguem uar esse termo) chamadas (pelo menos pelos franceses) de 
HOMOGRAFICAS.
Morgado

Alguem ha muito tempo escreveu:

Como vocês classificariam a seguinte função:
f(x) = x-a / bx+a. Nos livros de 2º grau temos a função afim ou do 1ºgrau 
com a expressão ax+b. Temos a função quadrática com a expressão ax^2+bx+c 
etc. E na expressão da função acima como poderiamos classificar ? Eu acho 
que é do 1º grau pois temos somente a variável x com expoente unitário, 
mas e quanto ao gráfico?





_
MSN 8: advanced junk mail protection and 2 months FREE*. 
http://join.msn.com/?page=features/junkmail

=
Instruções para entrar na lista, sair da lista e usar a lista em
http://www.mat.puc-rio.br/~nicolau/olimp/obm-l.html
O administrador desta lista é [EMAIL PROTECTED]
=


_
MSN Hotmail, o maior webmail do Brasil. http://www.hotmail.com

=
Instruções para entrar na lista, sair da lista e usar a lista em
http://www.mat.puc-rio.br/~nicolau/olimp/obm-l.html
O administrador desta lista é [EMAIL PROTECTED]
=



[obm-l] Re: [obm-l] ESTE PROBLEMA É INSOLÚVEL?

2003-01-14 Por tôpico Paulo Santa Rita
Ola Joao Carlos e demais
colegas desta lista ... OBM-L,

Este problema nao so tem solucao : voce passou por ela quando leu o livro 
sobre Geometria Euclidiana ... digo isso porque, em geral, estes livros 
falam do invariante topologico de Euler-Poincare, aplicavel a poliedros : V 
- A + F = 2. IMAGINE agora que o grafo que voce procura seja possivel, 
envolva ele em uma esfera e mostre que isso e contraditorio com o invariante 
topologico. E so. Nao e necessario nenhum conhecimento adicional.

Um Abraco
Paulo Santa Rita
3,1845,140103

From: J.C. PAREDE [EMAIL PROTECTED]
Reply-To: [EMAIL PROTECTED]
To: OBM [EMAIL PROTECTED]
Subject: [obm-l] ESTE PROBLEMA É INSOLÚVEL?
Date: Tue, 14 Jan 2003 16:07:21 -0300 (ART)


Faz um tempo que venho quebrando a cabeça para resolver o seguinte 
problema.

Em um bairro estão três casas, uma ao lado da outra e a distribuidora de 
água, de esgoto e de luz, sendo as distribuidoras também localizadas uma ao 
lado da outra em uma reta suporte paralela a reta suporte das casas. 
Deve-se por meio de tubulações levar água, esgoto e luz para todas as 
casas, sem que as tubulações se cruzem e tendo todas as tubulações a mesma 
profundidade. Como se deve fazer esta ligação?

Tentei quebrar a cabeça sozinho, dei uma olhada em termos de Geometria 
Euclidiana Plana, ouvi dizer que pode ser resolvida por grafos (porém não 
sei nada de grafos); e esses dias ouvi que este problema não tem solução; 
porém o camarada que disse isto disse que não tinha como provar.

Afinal, este problema tem ou não solução?

Já foi citado na lista problemas sobre grafos. Há alguma literatura no 
nível grafos para iniciantes que alguém conheça?

Obrigado



 JOÃO CARLOS PAREDE



-
Busca Yahoo!
O melhor lugar para encontrar tudo o que você procura na Internet


_
MSN Hotmail, o maior webmail do Brasil. http://www.hotmail.com

=
Instruções para entrar na lista, sair da lista e usar a lista em
http://www.mat.puc-rio.br/~nicolau/olimp/obm-l.html
O administrador desta lista é [EMAIL PROTECTED]
=



Re: [obm-l] Livro Geometria

2003-01-17 Por tôpico Paulo Santa Rita
Ola Leonardo e demais
colegas desta lista ... OBM-L,

Foi esse livro que despertou meu interesse pela Matematica.

Voce teve a mesma impressao que eu tive, quando o li pela primeira vez, apos 
adquiri-lo em um sebo ( sebo = livraria de livros usados ). Muitas vezes me 
perguntei o que o tornava tao interessante e diferente dos outros ...

Me parece que e porque os autores ( Eduardo Wagner e Augusto Morgado ) nao 
perdem tempo provando coisas simples e evidentes, partindo imediatamente a 
exposicao de fatoss espetaculares e inusitadas, que dificilmente 
imaginariamos que ocorrem.

Quando um matematico explica um fenomeno inusitado ( por exemplo : o circulo 
de nove pontos ) ele aguca nossa inteligencia e nos vemos claramente que a 
Matemaica nao se resume a provas de fatos evidentes e, portanto, 
desmotivadoras ( por exemplo : prove que 1+1=2 ).

Um Matematico adulto pode apreciar o formalismo, mas uma mente nova 
geralmente nao aprecia estas coisas ... Prove a uma crianca algo que ela 
duvida e nao suspeia e voce vai conquistar o interesse dela ... me parece 
que e esse simples detalhe que diferencia as grandes obras da mesmice e 
mediocridade que campeia na imensa maioria das obras didaticas da matematica 
...

As Olimpiadas de Matematica, que todos nos gostamos e admiramos e pelas 
quais fazemos verdadeiros sacrificios e uma continuacao de tudo isso ... Por 
que elas fazem sucesso ? Simplesmente porque as pessoas inteligentes odeiam 
coisas rotineiras e burocraticas, que sao os aspectos tipicos do ensino 
comum ...

Neste sentido, o Livro a que voce se refere, o Geometria II dos Prof Wagner 
e Morgado, se nao foi o marco inicial e corajoso de uma revolucao 
pedagogica, foi, ao menos, o alvorecer de tudo isso. E nos somos felizardos 
por termos em nossa lista estes dois Prof's que iniciaram esta revolucao : 
Wagner e Morgado.

Seria excelente que estes Profs dessem continuidade ao que comecaram com 
tanto brilho e eficiencia !

Para que esta mensagem nao fique totalmente fora de nossa tradicao, aqui vai 
uma joia do Geometria II :

1)Sejam a, b, c e d os lados de um quadrilatero ciclico. Mostre qua 
a area S desse quadrilatero pode ser expressa como :
S=Raiz_Quadrada((p-a)(p-b)(p-c)(p-d)) onde p e o semi-perimetro.

2)Se o quadrilatero e incritivel e circunscritivel, entao :
S=Raiz_Quadrada(abcd)

PROBLEMA : Se ABC e um triangulo e C o circulo inscrito nele, sejam C1, C2 e 
C3 as tres partes da area do triangulo que nao pertencem ao circulo. Calcule 
cada uma, separadamente, em funcao dos lados do triangulo.

Um Abraco a Todos
Paulo Santa Rita
6,,170103




From: Leonardo Borges Avelino [EMAIL PROTECTED]
To: [EMAIL PROTECTED]
Subject: [obm-l] Livro Geometria
Date: Thu, Jan 16, 2003, 10:07 PM


Caros amigos:

 Estava na casa de meu amigo e ele me mostrou um livro impressionante, 
que
se chama: Geometria II dos prof.s Eduardo Wagner, Augusto Morgado e 
Miguel.
Pergunta:

Onde consigo este livro? e (desculpem-me se a pergunta for idiota) se 
existe o Geometria I?


Valeu!!
Leonardo Borges


_
MSN Hotmail, o maior webmail do Brasil. http://www.hotmail.com

=
Instruções para entrar na lista, sair da lista e usar a lista em
http://www.mat.puc-rio.br/~nicolau/olimp/obm-l.html
O administrador desta lista é [EMAIL PROTECTED]
=



[obm-l] Re : [obm-l] Dúvidas sobre duas questões de análise real!!!

2003-01-23 Por tôpico Paulo Santa Rita
Ola Robson e demais
colegas desta lista ... OBM-L,

1) Caro Robson. Se voce nao sabe, logo  vai ficar sabendo ( quando comecar a 
estudar Analise ) que esta serie e um tipico representante do que se chama 
uma SERIE CONDICIONALMENTE CONVERGENTE.

Existe um Teorema ( de Riemann ) de Analise  que afirma que um reordenamente 
dos termos destas series pode faze-la divergir OU convergir ( convergir para 
UM NUMERO REAL QUALQUER, dado ). Claramente que este Teorema fala pouco ...

Seja A1, A2, ..., A3 um PA, isto e, Ai - Ai-1 = K, K # 0. Entao, pelo 
Teorema de Leibniz ( da Analise ), (1/A1) - (1/A2) + (1/A3) - ... converge. 
Converge pra onde ?  Isso vai depender da PA.  No seu caso :

1 - (1/2) + (1/3) - (1/4) + (1/5) - ... = Ln(2)

Outro caso bem conhecido e :

S = 1 - (1/3) + (1/5) - (1/7) + (1/9) - ... = pi/4

E uma relacao bem conhecida e que :

1 + (1/2^2) + (1/3^2) + (1/4^2) + ... = (1/3!)*((4S)^2).
A serie acima e o valor da funcao zeta em 2.

Note que  a sequencia 1, 1/(2^2),1/(3^2),1/(4^2), ... e tal que
1/Ai  -  2/Ai+1  +  1/Ai+2 = K, K constante e diferente de zero, para 
qualquer i. Toda serie que satisfaz a relacao acima e tal que :

A1 - 2*A2 + 3*A3 - 4*A4 + 5*A5 - ...

Converge condicionalmente, conforme voce pode mostrar facilmente usando o 
Teorema de Leibniz a que me referi acima.

Se admitirmos que as series alternadas cujos modulos dos inversos dos seus 
termos sao uma PA constituem um dado, entao o problema dos inversos das PA2 
fica bem posto. Mais claramente, seja A1, A2, ..., An uma sequencia tal que 
( K e S dados ) :

1) 1/Ai - 2/Ai+1 + 1/Ai+2 = K = constante nao nula, independente de i
2) A1 - 2*A2 + 3*A3 - 4*A4 + 5*A5 - 6*A6 + ... converge para S.

A serie abaixo converge para que numero real ? :

A1 + A2 + A3 + A4 + A5 + A6 + ...

Essa questao nao e simples. E uma forma diferente de abordar um problema 
resolvido apenas parcialmente pelo Euler.




2) Problema tipico de Introducao a Analise, que esta em todo livro desta 
categoria. Todo mundo mundo que estudou Analise fez este ou outro muito 
parecido. O enunciado esta um pouco confuso, mas sai assim : Suponha inf B  
sup A. Moste que isto conduz a um absurdo.
Logo sup A = inf B

Voce aqui estara usando o AXIOMA DO SUPREMO, tambem conhecido como AXIOMA DO 
COMPLETAMENTO : Todo conjunto de numeros reais limitado superiormente admite 
um supremo.

Deste axioma voce DEDUZ a existencia do infimo, vale dizer, postular o 
axioma do supremo implica no TEOREMA DO INFIMO. Mas nao existe nenhum 
razao forte para esta preferencia ...

Voce pode postular um AXIOMA DO INFIMO e deduzir o TEOREMA DO SUPREMO ( 
basta multiplicar por -1 e fazer um raciocinio bobo cheio de implicacoes 
obvias ). Um corpo ordenado no qual vale este axioma e um CORPO ORDENADO 
COMPLETO. Os numeros reais.

Ha autores que POSTULAM a existencia de um corpo ordenado completo, tais 
como o Prof Elon Lima ( Curso de Analise, Vol. 1). Todavia, historicamente, 
foi a percepcao do AXIOMA DO SUPREMO pelo Dedekin que permitiu CONSTRUIR um 
tal corpo, que alias pode ser construido por diversos outros caminhos ...

O fato de existir diversas construcoes de tais corpos ( ordenados e 
completos ) nao complica ... Dois  de tais corpos sao necessariamente 
isomorfos, vale dizer, INDISTINGUIVEIS no que se refere aos axiomas que os 
definem.

Um Abraco
Paulo Santa Rita
5,1750,230103


  - Original Message -
  From: Robson Monteiro
  To: [EMAIL PROTECTED]
  Sent: Tuesday, January 14, 2003 5:23 PM
  Subject: Dúvidas sobre duas questões de análise real!!!


  Oi Pessoal estou com duas dúvidas(sobre quetões que encontrei 
no livro do Elon Lages-Análise Real) e gostaria de saber se alguém pode me 
ajudar:

1º)  Efetue explicitamente uma reordenação dos termos da série 1 - (1/2) + 
(1/3) - (1/4) + (1/5) - ...

2º)  Sejam A,B conjuntos não vazios de números reais, tais que x Pertence 
a A e y pertence a B, com (x=y). Prove que supA=infB. Prove que 
supA=infB, se e somente se, para todo Epsilon0 dado, podem-se obter x 
pertencente a A e y pertencente a B tais que: y- x=epsilon


_
MSN Hotmail, o maior webmail do Brasil.  http://www.hotmail.com

=
Instruções para entrar na lista, sair da lista e usar a lista em
http://www.mat.puc-rio.br/~nicolau/olimp/obm-l.html
O administrador desta lista é [EMAIL PROTECTED]
=



[obm-l] Re: [obm-l] Somas de séries

2003-01-27 Por tôpico Paulo Santa Rita
Ola Claudio e demais
colegas desta lista  ... OBM-L,

A ultima pergunta e simples, em determinado sentido ... Use o MAPLE e voce 
vera a soma para qualquer 1/(an+b)^2. Mas o MAPLE faz as coisas ao modo 
dele, insatisfatorio em certo sentido ...

Por muitas razoes, eu precisei investigar as series da forma :

NIC(a,r)= 1/a  -  1/(a+r)  +  1/(a+2r)  -  1/(a+3r)  +  ...

Num certo sentido estas series sao primitivas, vale dizer, supomos que a 
funcao NIC : R^2 - R ( NIC nao existe na literatura, eu fui obrigado a 
reconhecer sua importancia ) esta bem definida e vemos o que se pode 
produzir de interessante com elas. Claramente que a serie :

1/a  +  1/(a+r)  +  1/(a+2r)  +  1/(a+3r)  +  ... smepre diverge !

Mas a serie :

1/a^2  +  1/(a+r)^2  +  1/(a+2r)^2  +  1/(a+3r)^2  + ... sempre converge ! 
Que relacao existe entre o valor para o qual esta serie converge e NIC ? 
Existe X e Y tais que :

1/a^2 + 1/(a+r)^2 + 1/(a+2r)^2 + 1/(a+3r)^2 + ... = K*(NIC(X,Y))^2 ?
Para algum K real ?

Claramente que existe alguns reordenamentos evidentes ...

( 1/a  -  1/(a+r) )  + ( 1/(a+2r)  -  1/(a+3r) )  +  ...

NIC serve para caracterizar uma classe de triangulos aritmeticos, conhecidos 
como triangulos harmonicos. O exemplo mais simples serie :

1
1/2  1/2
1/3  1/6  1/3
1/4  1/12 1/12  1/4
...

Cada termo e a diferenca entre o que esta acima a esquerda e o que esta a 
esquerda. As colunas sao numeradas da esquerda para a direita a partir de 
-1. O 1/2 da coluna -2 e tal que : 1/2 = 1 - 1/2. E assim para todos os 
termos ... Se voce tomar uma coluna qualquer desde o infinito ate um 
determinado termo, a soma e o termo da esquerda : e o teorema das colunas do 
triangulo de pascal estendido a este tipo de triangulo ... Este triangulo e 
conhecido tambem como triangulo de Leibniz.

Todo triangulo harmonico tem um numero que o caracteriza univocamente e que 
e um valor de NIC. Em particular, no triangulo de Leibniz este valor de NIC 
e Ln(2) ..., isto e, NIC(1,1) Um outro triangulo harmonico teria um outro 
valor para NIC.

Pode-se dizer que os triangulos harmonicos sao extensoes dos triangulos 
aritmeticos tipo triangulo de pascal. Eles sao como as colunas negativas 
deste triangulo ...

Considere a relacao de recorrencia :
(n e a coluna, p a linha )

A00  = 1
An,p = An-1,p-1 + An,p-1  n em {0,1,2,...} e p em {0,1,...,n}
Fixado sucessivamente  n, varie p. Voce vai obter o triangulo de pascal. 
Para outros valores de A00 e outras relacoes de recorrencia da forma An,p= 
K*An-1,p-1 + L*An,p-1 voce obetera outros triangulos aritmeticos tipo 
pascal. Observe que no triangulo de pascal K=L=1.

Existe alguma relacao entre K e L e o trianngulo harmonico  correspondente, 
isto e, entre K, L que definem um triangulo tipo pascal  e NIC(K,L), que 
caracteriza um triangulo harmonico  ?

Neste momento e imperioso que se perceba o seguinte :

No triangulo de Pascal quando nos olhamos para o N,P em Binom(N,P) nos quase 
inadvertidamente imaginamos em combinacoes de N elementos tomados P a P. 
Mas, isto, e ... UMA INTERPRETACAO : nao e A INTERPRETACAO.  o Professor 
NIColau deixa claro isso em seu livro sobre MATEMATICA QUANTICA. La ele diz 
claramente que podemos, sem receios, dar outras interpretacoes aos numeros 
binomias, que representariam outras formas de contagem ( as q-contagem ), 
vale dizer, indices. A interpretacao que damos e, assim, uma contagem 
particular, nao A CONTAGEM, A UNICA POSSIVEL !

Isto posto, a relacao de recorrencia implica claramente que estamos vemos os 
N, P em BINOM(N,P) como LOCALIZACOES ou LUGARES em alguma estrutura mais 
ampla, ou, o que da no mesmo, os valores de uma funcao definida em R^2.

Isto e muito bom, por diversas razoes ...

Seja r um real positivo. A funcao :

Z(r)=1/N^r, N={1,2,...} converge se r1. r=2 e um caso particular de
Z(r)=1/N^r. Uma compreenssao destas coisas por este caminho pode abrir uma 
vertente nova na abordagem deste fato para r complexo, em particular, para 
os zeros desta funcao ...

Observe que esta e uma abordagem por dentro, vale dizer, por fora, 
partindo diretamente de r complexo, muita gente ja tentou e nao teve 
sucesso. Isso pode ser mais uma abordagem que sera mal-sucedida. Mas ... 
pode ser ! Ate esclarecer estas coisas nos nao podemos advinhar o que vai 
acontecer !

Francamente eu acho tudo isso interessante independente de uma orientacao 
pratica qualquer, independente de qualquer novo resultado. Isso e 
interessante simplesmente porque eu tenho certeza que algo misterioso e 
profundo esta no fim deste arco-iris ...

Um abraco
Paulo Santa Rita
2,1849,270103










From: Cláudio \(Prática\) [EMAIL PROTECTED]
Reply-To: [EMAIL PROTECTED]
To: [EMAIL PROTECTED]
Subject: [obm-l] Somas de séries
Date: Mon, 27 Jan 2003 16:25:23 -0200

Caro Paulo Santa Rita:

Bem interessante essa questão da relação entre:

R = SOMA A(n)e S = SOMA (-1)^(n+1)*n*A(n).

onde A(n) = 1 / (An^2 + Bn + C), com A  0.

Dado que quando A(n) = 1/n^2, R = Pi^2 / 6 e S = Ln(2), a relação

[obm-l] Quadrados em um Quadriculado.

2003-01-29 Por tôpico Paulo Santa Rita
Ola Pessoal !

O problema abaixo e uma generalização de uma questão que foi proposta em 
outra lista, algum tempo atras. Não e de solução imediata, mas não e 
dificil.

PROBLEMA : Divide-se cada lado de um quadrado em N partes iguais. Pelos 
pontos de divisão tracam-se paralelas aos lados do quadrado, originando 
assim um quadriculado.

1) Com vertices nos pontos deste quadriculado, quantos quadrados podem ser 
construidos ( em funcao de N ) ?
2) Seja Q(N) o numero de quadrados. Para todo P natural dado diga se existe 
um natural N tal que Q(N) = 10^P.

OBS1 : Note que para responder 2 voce precisa responder 1 atraves de uma 
funcao que seja manipulavel.

OBS2 : considerar neste calculo tambem os quadrados inclinados em relação 
ao quadrado original.

UMA SUGESTAO : Supondo que o quadrado original tem lado medindo N, seja Q o 
conjunto de todos os quadrados construtiveis cujos lados sejam paralelos aos 
lados do quadrado original e cujos lados tem a mesma medida L, L = N. Em 
qualquer um destes quadrados a quantidade de quadrados inscritos e cujos 
lados nao paralelos aos lados do quadrado original e constante ...  Manipule 
com habilidade as expressoes que vao surgir que elas se reduzirao a um 
polinomio bem simples. Isto responde ao item 1 e da condicoes de encarar o 
intem 2. Para responder 2 basta aplicar o que voce sabe sobre raizes 
racionais de equacoes polinomiais inteiras.

Um Abraco
Paulo Santa Rita
4,1652,290103






_
MSN Messenger: converse com os seus amigos online.  
http://messenger.msn.com.br

=
Instruções para entrar na lista, sair da lista e usar a lista em
http://www.mat.puc-rio.br/~nicolau/olimp/obm-l.html
O administrador desta lista é [EMAIL PROTECTED]
=


[obm-l] O armario e o corredor

2003-01-30 Por tôpico Paulo Santa Rita
Ola pessoal !

O problema abaixo deve ser do conhecimento de muitos de voces ... Ele e 
facil, mas tem uma resposta surpreendente :

PROBLEMA : ( Descricao do Corredor ) Sobre uma mesa desenha-se um angulo 
reto AOB. Tomando um ponto O' no interior deste angulo, desenhamos o angulo 
reto A'O'B'. A distancia entre os segmentos paralelos OA e O'A' e a e a 
distancia entre os segmentos paralelos OB e O'B' e b. Qual e o comprimento 
maximo que uma vareta CD ( o Armario ) pode possuir de forma que ela possa, 
deslizando sobre a mesa, entrar em AA' e sair em BB' ?

OBS : O comprimento maximo de CD deve ser dado em funcao de a e b

SUGESTAO : Encoste a vareta em O'. Surgira um angulo entre a vareta e o 
segmento O'A' ( ou entre O'B'. Voce escolhe ). Expresse o comprimento da 
vareta em funcao do angulo, de a e de b.

Um Abraco a todos
Paulo Santa Rita
5,0905,300103







_
MSN Hotmail, o maior webmail do Brasil.  http://www.hotmail.com

=
Instruções para entrar na lista, sair da lista e usar a lista em
http://www.mat.puc-rio.br/~nicolau/olimp/obm-l.html
O administrador desta lista é [EMAIL PROTECTED]
=


Re: [obm-l] O armario e o corredor

2003-01-30 Por tôpico Paulo Santa Rita
Ola Claudio e demais
colegas desta lista ... OBM-L,

Resposta correta ! Com sinceridade alertei que o problema, nao obstante 
simples, tinha uma solucao surpreendente !

Em verdade esse problema me foi sugerido em uma mudanca la em casa, quando 
eu ainda era menino : meu pai e tios tentavam arrastar um grande armario 
atraves de um corredor em forma de L, quando entao os sucessivos fracassos 
os levaram a suspeitar que era impossivel, sem saberem justificar.

Provando ( Garantindo ! Ele nao conhecem Calculo. ) que era impossivel, eu 
os convenci a desmontarem o armario, previamente. So depois de muitos anos 
vim a saber que havia um problema de Calculo Diferencial muito parecido.

Eu nao acompanhei todos os calculos que voce efetuou, mas a ideia contida no 
fragmento abaixo esta correta e e o insight que mata a questao. Se 
eventualmente houver algum erro no algebrismos ( na burocracia ) e sem 
duvida apenas uma desatencao.

Vou propor agora um problema que nao e facil. Para que ele possa ser 
digerido, vou coloca-lo na forma de sub-problemas :

PROBLEMA : Seja Q um quadrado de lado unitario. Mostre que, qualquer que 
seja a forma como colocarmos no interio de Q dois outros quadrados de lados 
L1 e L2, se L1 + L2  1 entao estes dois outros quadrados terao ao menos um 
ponto em comum.

Esse e um dos problemas do Paul Erdos. Ja foi proposto aqui nesta lista.
A ideia e encontrar uma demonstracao rigorosa, analitica, que nao lance mao 
de intuicoes geometricas contestaveis.

SUGESTAO : Podemos representar Q como a regiao do R^2 na qual as coordenas 
(X,Y) de qualquer ponto obedece as condicoes :

0 = X = 1
0 = Y = 1

Precisamos encontrar uma maneira de garantir que os quadrados de lados L1 e 
L2 estejam confinados em Q. Convencionemos, pois, que :

1) O quadrado de lado L1 (L2) tem vertices ABCD (EFGH) com o lado AD (EH) 
inclinado de ALF (BET) em relacao aos eixo das abscissas.
2) A (E) e o vertice de menor ordenada. Se dois vertices tiverem a mesma 
menor ordenada, A (E) sera o de menor abscissa
3) As coordenadas de um vertice serao indexadas pela letra do vertice que 
representam. Assim : A=(Xa,Ya), E=(Xe,Ye)

Note que acima fizemos tao somente convencoes, vale dizer, essas notacoes 
nao impoe nenhuma restricao a generalidade que o problema requer, dado que 
serao adotadas apos o desenho dos quadrados. por outro lado, e claro que : 
0 = ALF,BET  pi/2.

Isto posto, adotamos qualquer vertice como referencia e exprimimos os demais 
em funcao dele. Assim ( adotando A como origem ) :

D-A=L1*(cos(ALF),sen(ALF))
C-A=L1*(cos(ALF)-sen(ALF),cos(ALF)+sen(ALF))
B-A=L1*(-sen(ALF),cos(ALF))

Substituindo os vertices por suas coordenadas, exprimindo todas em funcao 
das coordenadas do vertice A e lembrando que estes vertices devem estar na 
regiao Q, isto e, entre 0 e 1, a intersecao das inequecoes resultantes 
fornecera :

L1*sen(ALF) = Xa = 1 - L1*cos(ALF)
0 = Ya = 1 - L1*(sen(ALF) + cos(ALF))

Estas sao as CONDICOES DE CONFINAMENTO, vale dizer, qualquer que seja L1 e 
qualquer que seja L1, as coordenadas do vertice A devem satisfazer as 
condicoes acima para que o quadrado ABCD esteja contido na regiao Q. 
Claramente que uma relacao analogo vale para o quadrado EFGH, isto e :

L2*sen(BET) = Xe = 1 - L2*cos(BET)
0 = Ye = 1 - L2*(sen(BET) + cos(BET))

Bom, agora nos temos quase tudo para dar uma solucao elegante ao problema do 
Erdos. Vamos mostrar que L1+L2  1 e contaditorio com as condicoes de 
confinamento.

PRIMEIRO SUB-PROBLEMA : Prove que existe um intervalo fechado [m,n], [m,n] 
contido em [0,1], tal que qualquer reta vertical X=K que passa por [m,n] 
passa tambem no interior dos dois quadrados.

SUGESTAO : Observe que provar a afirmacao acima e o mesmo que dizer que os 
quadrados tem pontos com a mesma abscissa. Para provar isso suponha que Xa e 
diferente de Xe ( Se Xa = Xe, X=Xa e uma reta que atende as condicoes e a 
demonstracao esta conluida ). Sem perda de generalidade suponha Xa  Xe. 
Calcule a abscissa do ponto de maior abscissa de ABCD e a abscissa do ponto 
de menor abscissa de EFGH. Monte dois intervalos : [Xa, maior abscissa], 
[menor abscissa, Xe]. Prove que se L1+L2  1 os intervalos nao podem ser 
disjuntos.

O segundo sub-problema e tomar todas as retas que passam pela regiao de 
mesmas abscissas e mostrar que alguma(s) passa(m) SIMULTANEAMENTE no 
interior dos dois quadrados, vale dizer, vamos analiticamente subir a reta 
e ver o que acontece la em cima

Um Abraco a todos !
Paulo Santa Rita
5,1802,300103

From: Cláudio \(Prática\) [EMAIL PROTECTED]
Reply-To: [EMAIL PROTECTED]
To: [EMAIL PROTECTED]
Subject: Re: [obm-l] O armario e o corredor
Date: Thu, 30 Jan 2003 15:53:36 -0200

Caro Paulo e demais colegas da lista:

O maior comprimento de vareta que pode fazer a curva é igual ao 
comprimento do menor segmento com extremidades em OA e OB que contenha 
O'. Suponha que o segmento seja MN, com M em OA e N em OB.


_
MSN Hotmail, o

Re: [obm-l] O armario e o corredor

2003-01-31 Por tôpico Paulo Santa Rita
Hi Salvador e demais
colegas desta lista ... OBM-L,

Gostei do problema. Voce pode falar mais um pouco sobre ele ? Se eu 
resolve-lo ou conseguir algum progresso significativo mostro ao Conway e 
publico aqui nesta lista.

Desde agradeco.

Um abraco
Paulo Santa Rita
6,1043,310103

From: Salvador Addas Zanata [EMAIL PROTECTED]
Reply-To: [EMAIL PROTECTED]
To: [EMAIL PROTECTED]
Subject: Re: [obm-l] O armario e o corredor
Date: Thu, 30 Jan 2003 22:45:50 -0200 (EDT)



Caros amigos,

Um problema pelo que eu sei, em aberto, relacionado a esse consiste no
seguinte:

Dado um corredor com 1 metro de largura, que faz uma curva de 90 graus e
continua com a mesma largura, qual e a maior area possivel que pode fazer
essa curva? Observe que o formato dessa area pode ser qualquer, e
obviamente ela e suposta rigida. E claro que o maior segmento que essa
area contem e limitado, mas isso nao ajuda muito.

O John Conway fez algumas coisas parciais sobre isso.


Abraco,


Salvador



On Thu, 30 Jan 2003, Paulo Santa Rita wrote:

 Ola Claudio e demais
 colegas desta lista ... OBM-L,

 Resposta correta ! Com sinceridade alertei que o problema, nao obstante
 simples, tinha uma solucao surpreendente !

 Em verdade esse problema me foi sugerido em uma mudanca la em casa, 
quando
 eu ainda era menino : meu pai e tios tentavam arrastar um grande armario
 atraves de um corredor em forma de L, quando entao os sucessivos 
fracassos
 os levaram a suspeitar que era impossivel, sem saberem justificar.

 Provando ( Garantindo ! Ele nao conhecem Calculo. ) que era impossivel, 
eu
 os convenci a desmontarem o armario, previamente. So depois de muitos 
anos
 vim a saber que havia um problema de Calculo Diferencial muito parecido.

 Eu nao acompanhei todos os calculos que voce efetuou, mas a ideia 
contida no
 fragmento abaixo esta correta e e o insight que mata a questao. Se
 eventualmente houver algum erro no algebrismos ( na burocracia ) e sem
 duvida apenas uma desatencao.

 Vou propor agora um problema que nao e facil. Para que ele possa ser
 digerido, vou coloca-lo na forma de sub-problemas :

 PROBLEMA : Seja Q um quadrado de lado unitario. Mostre que, qualquer que
 seja a forma como colocarmos no interio de Q dois outros quadrados de 
lados
 L1 e L2, se L1 + L2  1 entao estes dois outros quadrados terao ao menos 
um
 ponto em comum.

 Esse e um dos problemas do Paul Erdos. Ja foi proposto aqui nesta lista.
 A ideia e encontrar uma demonstracao rigorosa, analitica, que nao lance 
mao
 de intuicoes geometricas contestaveis.

 SUGESTAO : Podemos representar Q como a regiao do R^2 na qual as 
coordenas
 (X,Y) de qualquer ponto obedece as condicoes :

 0 = X = 1
 0 = Y = 1

 Precisamos encontrar uma maneira de garantir que os quadrados de lados 
L1 e
 L2 estejam confinados em Q. Convencionemos, pois, que :

 1) O quadrado de lado L1 (L2) tem vertices ABCD (EFGH) com o lado AD 
(EH)
 inclinado de ALF (BET) em relacao aos eixo das abscissas.
 2) A (E) e o vertice de menor ordenada. Se dois vertices tiverem a 
mesma
 menor ordenada, A (E) sera o de menor abscissa
 3) As coordenadas de um vertice serao indexadas pela letra do vertice 
que
 representam. Assim : A=(Xa,Ya), E=(Xe,Ye)

 Note que acima fizemos tao somente convencoes, vale dizer, essas 
notacoes
 nao impoe nenhuma restricao a generalidade que o problema requer, dado 
que
 serao adotadas apos o desenho dos quadrados. por outro lado, e claro 
que :
 0 = ALF,BET  pi/2.

 Isto posto, adotamos qualquer vertice como referencia e exprimimos os 
demais
 em funcao dele. Assim ( adotando A como origem ) :

 D-A=L1*(cos(ALF),sen(ALF))
 C-A=L1*(cos(ALF)-sen(ALF),cos(ALF)+sen(ALF))
 B-A=L1*(-sen(ALF),cos(ALF))

 Substituindo os vertices por suas coordenadas, exprimindo todas em 
funcao
 das coordenadas do vertice A e lembrando que estes vertices devem 
estar na
 regiao Q, isto e, entre 0 e 1, a intersecao das inequecoes resultantes
 fornecera :

 L1*sen(ALF) = Xa = 1 - L1*cos(ALF)
 0 = Ya = 1 - L1*(sen(ALF) + cos(ALF))

 Estas sao as CONDICOES DE CONFINAMENTO, vale dizer, qualquer que seja L1 
e
 qualquer que seja L1, as coordenadas do vertice A devem satisfazer as
 condicoes acima para que o quadrado ABCD esteja contido na regiao Q.
 Claramente que uma relacao analogo vale para o quadrado EFGH, isto e :

 L2*sen(BET) = Xe = 1 - L2*cos(BET)
 0 = Ye = 1 - L2*(sen(BET) + cos(BET))

 Bom, agora nos temos quase tudo para dar uma solucao elegante ao 
problema do
 Erdos. Vamos mostrar que L1+L2  1 e contaditorio com as condicoes de
 confinamento.

 PRIMEIRO SUB-PROBLEMA : Prove que existe um intervalo fechado [m,n], 
[m,n]
 contido em [0,1], tal que qualquer reta vertical X=K que passa por [m,n]
 passa tambem no interior dos dois quadrados.

 SUGESTAO : Observe que provar a afirmacao acima e o mesmo que dizer que 
os
 quadrados tem pontos com a mesma abscissa. Para provar isso suponha que 
Xa e
 diferente de Xe ( Se Xa = Xe, X=Xa e uma reta que atende as condicoes e 
a
 demonstracao esta conluida

[obm-l] Um problema de sistema !

2003-01-31 Por tôpico Paulo Santa Rita
Ola Pessoal !

Com certeza, voces nao terao dificuldades em resolver a seguinte questao : 
Considere que X,Y e Z sao positivos e que satisfazem o sistema abaixo,

X^2 + XY + (Y^2)/3 = 25
(Y^2)/3 + Z^2 = 9
Z^2 + ZX + X^2 = 16

Encontre o valor de ( XY + 2YZ + 3ZX ).

SUGESTAO : Voce nao precisa, necessariamente, resolver o sistema ...

Um Abraco
Paulo Santa Rita
6,1133,310103



_
MSN Hotmail, o maior webmail do Brasil.  http://www.hotmail.com

=
Instruções para entrar na lista, sair da lista e usar a lista em
http://www.mat.puc-rio.br/~nicolau/olimp/obm-l.html
O administrador desta lista é [EMAIL PROTECTED]
=


Re: [obm-l] Quadrados em um Quadriculado.

2003-01-31 Por tôpico Paulo Santa Rita
Ola Claudio e demais
colegas desta lista ... OBM,

Resposta correta ! A percepcao que mata a questao e ver que num quadrado 
normal  de lado i cabem exatamente (i-1) Quadrados inclinados. Assim, ser 
QN(i) for o toal de quadrados de lados i, entao (i-1)*QN(i) e o total de 
quadrados inclinados associados aos quadrados normais de lado i :

T(i)= QN(i) + (i-1)*QN(i) = i*QN(i)
Total de quadrados : somatorio de i*QN(i), i variando de 1 ate N. Como 
claramente QN(i)=(N-i+1)^2, segue que :

Total de quadrados (T) : somatorio de i*(N-i+1)^2.

Vou colocar este somatorio de forma que ele seja conveniente para voce 
responder o item 2.

T= somatorio de (i*(N-i+1)^2), i variando de 1 ate N.
T= 1*N^2 + 2*(N-1)^2 + 3*(N-2)^2 + ... + (N-1)*2^2 + N*(1^2)

T=
N^2 + (N-1)^2 + (N-2)^2 + (N-3)^2 + ... + 3^2 + 2^2 + 1^2
   + (N-1)^2 + (N-2)^2 + (N-3)^2 + ... + 3^2 + 2^2 + 1^2
 + (N-2)^2 + (N-3)^2 + ... + 3^2 + 2^2 + 1~2
   + (N-3)^2 + ... + 3^2 + 2^2 + 1^2
... ...   ...
   + 3^3 + 2^2 + 1^2
 + 2^2 + 1^2
   + 1^2

A primeira linha e a soma dos N primeiros quadrados, que da :
BI(N,1) + 3*BI(N,2) + 2*BI(N,3)
A segunda linha e a soma dos N-1 primeiros qudrados, que da :
BI(N-1,1) + 3*BI(N-1,2) + 2*BI(N-1,3)
A terceira linha e a soma dos N-2 primeiros quadrados, que da :
BI(N-2,1) + 3*BI(N-2,2) + 2*BI(N-2,3)
e assim sucessivamente ... Vemos claramente que temos 3 colunas do triangulo 
de Pascal. Podemos, pois, aplicar o teorema das colunas. Portanto, o 
somatorio fica :

T=BI(N+1,2) + 3*BI(N+1,3) + 2*BI(N+1,4)
T=(BI(N+1,2) + BI(N+1,3)) + 2*(BI(N+1,3) + BI(N+1,4))
T=BI(N+2,3) + 2*BI(N+2,4)
T=(BI(N+2,3) + BI(N+2,4)) + BI(N+2,4)
T=BI(N+3,4) + BI(N+2,4) = BI(N+2,4) + BI(N+3,4)

T = BI(N+2,4) + BI(N+3,4)

Que grata surpresa ! Encontramos uma expressao ao mesmo tempo bela e simples 
! Mais que isso : sao dois termos consecutivos da quinta coluna do triangulo 
de Pascal. Como diria Goeth - Sorvei, olhos meus, o que vos der a vida ... 
A copiosa beleza no Universo difundida ! -, a beleza e realmente irma da 
simplicidade ...

Agora e facil, certo ?  Existem dois termos consecutivos da quinta coluna do 
triangulo de Pascal cuja soma e uma potencia de 10 ?

Oportunamente eu vou falar sobre combinacoes lineares de numeros binomias. 
E uma terra prenhe de tesouros poucos explorados ...

Um Abraco
Paulo Santa Rita
6,1836,310103

From: Cláudio \(Prática\) [EMAIL PROTECTED]
Reply-To: [EMAIL PROTECTED]
To: [EMAIL PROTECTED]
Subject: Re: [obm-l] Quadrados em um Quadriculado.
Date: Fri, 31 Jan 2003 12:02:05 -0200

Suponha que os lados do quadrado foram divididos em n partes iguais, cada
uma com comprimento = 1.

Sejam:
D(k) = número de quadrados direitos (com lados paralelos ao quadrado
maior) de lado com medida k contidos no quadrado maior (de lado n).
T(k) = número de quadrados tortos (com lados não paralelos aos do 
quadrado
maior) e que cabem num quadrado direito de lado k, mas não num quadrado 
de
k-1.

Não é difícil ver que:
D(1) = n^2, D(2) = (n-1)^2, D(3) = (n-2)^2, ..., D(k) = (n-k+1)^2, ..., 
D(n)
= 1^2 = 1

Além disso, temos:
T(1) = 0, T(2) = 1, T(3) = 2, ..., T(k) = k-1, ..., T(n) = n-1
Justificativa:
Suponha que os vértices do quadrado direito sejam os pontos (0,0), (k,0),
(0,k) e (k,k) no plano cartesiano.
Cada quadrado torto contado em T(k) terá que ter cada um de seus vértices
contido numa aresta distinta do quadrado direito, caso contrário, ou ele
teria dois vértices numa mesma aresta e seria direito ou então ele 
caberia
num quadrado direito menor == ambas as situações contradiriam a 
definição
de T(k).
Além disso, como um vértice desse quadrado determina todos os outros, 
existe
uma bijeção entre o conjunto de quadrados tortos contados em T(k) e o
conjunto de vértices de tais quadrados contidos, por exemplo, na aresta que
vai de (0,0) até (0,k). Estes últimos seriam (0,1), (0,2), ..., (0,k-1) -
total de k-1 vértices == T(k) = k-1.

Finalmente, temos a relação:
Q(n) = [ D(1) + D(2) + ... + D(n) ]   +   [ D(1)*T(1) + D(2)*T(2) + ... +
D(n)*T(n) ]
Justificativa:
Q(n) = no. total de quadrados direitos + no. total de quadrados tortos
O primeiro termo entre colchetes é claramente o no. total de quadrados
direitos.
Cada quadrado direito de lado k contribui T(k) quadrados tortos que 
só
cabem nele.
Assim, a contribuição dos quadrados direitos de lado k para o total de
quadrados tortos perfaz D(k)*T(k) quadrados.
Quadrados tortos menores já terão sido contados como contribuição de
quadrados direitos menores.
Logo, o segundo termo entre colchetes é realmente o no. total de quadrados
tortos.

Usando D(k) = (n-k+1)^2 e T(k) = k-1, além de manipulações algébricas e de
fórmulas manjadas para a soma dos primeiros n naturais, assim como de 
seus
quadrados e cubos, chegamos a:

Q(n) = n*(n+1)^2*(n+2)/12.

**

Ainda estou pensando na

Re: [obm-l] Quadrados em um Quadriculado - parte 2

2003-02-04 Por tôpico Paulo Santa Rita
Ola Claudio e demais
colegas desta lista ... OBM-L,

A Resposta esta correta. Eu nao acompanhei todos os seus argumentos, tanto 
por falta de tempo quanto porque ha outras formas mais diretas de 
resolve-lo.

Eu bolei esta questao especificamente para a OBM, nivel medio. Nao sei 
porque a banca nao aceitou propo-la. Nao e uma questao dificil, exige apenas 
um insight para o item 1 e, no item 2, exige conhecimentos bem divulgados.

Segue abaixo uma questao questao que eu propus para o pessoal da OBM de 
nivel 2 ( setima/oitava series do 1 grau ) :

Num conjunto de 100 numeros naturais, dois a dois distintos, verifica-se que 
quaisquer 3 numeros ( iguais ou nao ) podem ser os lados de um triangulo nao 
obtusangulo ( que nao tem um angulo maior que 90 graus ). Seja A a soma de 
todos os perimetros de todos os triangulos ( isosceles, equilateros e 
escalenos ) que podem ser construidos com estes 100 numeros. Qual o valor 
minimo que A pode ter ?

Um Abraco
Paulo Santa Rita
3,1243,040203





From: Cláudio \(Prática\) [EMAIL PROTECTED]
Reply-To: [EMAIL PROTECTED]
To: [EMAIL PROTECTED]
Subject: [obm-l] Quadrados em um Quadriculado - parte 2
Date: Tue, 4 Feb 2003 12:02:33 -0200

Caro Paulo:

A parte 2 do problema pede para determinar todos os inteiros p, para os
quais existe um inteiro positivo n tal que:

n * (n+1)^2 * (n+2) / 12 = 10^p  ==
n * (n+1)^2 * (n+2) = 2^(p+2) * 3 * 5^p

No entanto, eu achei que a única solução é p = 0 == n = 1. Será que eu
errei em algum lugar?

E dividi o problema em 3 casos: n ímpar, n = 0 (mod 4) e n = 2 (mod 4):

CASO 1: n é ímpar
n é ímpar  ==  n+1 é par e n+2 é ímpar.

Assim, (n+1)^2 = 2^(p+2) * 3^x * 5^y   e   n*(n+2) = 3^(1-x) * 5^(p-y)
com 0 = x = 1 e 0 = y = p

(n+1)^2 é quadrado == p+2 é par, x = 0 e y é par

p+2 é par == p é par == p = 2q
y é par == y = 2z == p-y = 2q-2z

Assim:  n+1 = 2^(q+1) * 5^ze   n*(n+2) = 3 * 5^(2q-2z)

Temos dois sub-casos a considerar: 5 divide n+1 ou 5 não divide n+1:

Sub-caso 1: 5 | n+1
5 | n+1 == (5,n) = (5,n+2) = 1  ==  z = q  ==
n+1 = 2^(q+1) * 5^q   e   n*(n+2) = 3  == n = 1 == q = 0 == p = 0

Sub-caso 2: 5 não | n+1
5 não | n+1 == (5,n+1) = 1  == z = 0  ==
n+1 = 2^(q+1)   e   n*(n+2) = 3 * 5^(2q)  ==
n = 2^(q+1) - 1, n+2 = 2^(q+1) + 1  ==  n*(n+2) = 2^(2q+2) - 1 = 4 *
^(2q)  -  1  ==
3 * 5^(2q)  =  4 * 2^(2q) - 1  ==  4 * 2^(2q)  -  3 * 5^(2q)  = 1  ==  q 
=
0   ==
n+1 = 2  e  n*(n+2) = 3  == n = 1 == p = 0


CASO 2: n = 0 (mod 4)
n = 0 (mod 4)  ==  n+1 é ímpar  e  n+2 = 2 (mod 4)  ==

(n+1)^2 = 3^x * 5^y   en*(n+2) = 2^(p+2) * 3^(1-x) * 5^(p-y)
com 0 = x = 1  e  0 = y = p

(n+1)^2 é quadrado  ==  x = 0  e  y = 2z  ==
n+1 = 5^z   e   n*(n+2) = 2^(p+2) * 3 * 5^(p-2z)

Sub-Caso 1: 5 | n+1
5 | n+1 == (5,n) = (5,n+2) = 1 == n*(n+2) = 2^(p+2) * 3

Sub-Caso 1.1: 5 | n+1  e  3 | n
3 | n ==  (3,n+1) = (3,n+2) = 1 == n = 2^(p+1) * 3   e   n+2 = 2  == XXX

Sub-Caso: 1.2: 5 | n+1 e 3 não | n
3 não | n == 3 | n+1 == n = 2^(p+1)   e   n+2 = 2*3 = 6  == Q(n) = 196 

10^p  == XXX

Sub-Caso 2: 5 não | n+1
5 não | n+1 == n+1 = 1 == XXX


CASO 3: n = 2 (mod 4)
n = 2 (mod 4)  ==  n+1 é ímpar  e  n+2 = 0 (mod 4)  ==

(n+1)^2 = 3^x * 5^y   en*(n+2) = 2^(p+2) * 3^(1-x) * 5^(p-y)
com 0 = x = 1  e  0 = y = p

(n+1)^2 é quadrado  ==  x = 0  e  y = 2z  ==
n+1 = 5^z   e   n*(n+2) = 2^(p+2) * 3 * 5^(p-2z)

Sub-Caso 1: 5 | n+1
5 | n+1  ==  (5,n) = (5,n+2) = 1  ==  n*(n+2) = 2^(p+2) * 3

Sub-Caso 1.1: 5 | n+1  e  3 | n
3 | n  ==  (3,n+1) = (3,n+2) = 1  ==  n = 2*3 = 6  ==  Q(n) = 196  
10^p
== XXX

Sub-Caso: 1.2: 5 | n+1  e  3 não | n
3 não | n  ==  3 | n+1  ==  n = 2  ==  Q(n) = 6  10^p == XXX

Sub-Caso 2: 5 não | n+1
5 não | n+1  ==  n+1 = 1  ==  XXX

**

Um abraço,
Claudio.


=
Instruções para entrar na lista, sair da lista e usar a lista em
http://www.mat.puc-rio.br/~nicolau/olimp/obm-l.html
O administrador desta lista é [EMAIL PROTECTED]
=


_
MSN Messenger: converse com os seus amigos online.  
http://messenger.msn.com.br

=
Instruções para entrar na lista, sair da lista e usar a lista em
http://www.mat.puc-rio.br/~nicolau/olimp/obm-l.html
O administrador desta lista é [EMAIL PROTECTED]
=


[obm-l] Re: [obm-l] Truângulos não-obtusângulos

2003-02-04 Por tôpico Paulo Santa Rita
Ola Claudio e demais
colegas desta lista ... OBM-L,

O que importa e a ideia ... Como se pode observar abaixo, o unico 
conhecimento realmente necessario e saber - como sabem todos os bons alunos 
da 7 serie - que NUM TRIANGULO NAO OBTUSANGULO O QUADRADO DE QUALQUER LADO 
E, NO MAXIMO, IGUAL A SOMA DOS QUADRADOS DOS OUTROS DOIS LADOS.

Alem disso, so e necessario ter a coragem de pensar e errar tantas vezes 
quantas forem necessarias ate esclarecer o enigma. Nunca e vergonhoso errar, 
quando estamos tentamos acertar. Claramente que so nao erram Deus e os 
Imbecis. Como diria Schiller : Oh discipulo covarde ! Rompe a inercia e a 
sonolencia e engolfa-te brioso no arrebol que anteves !

A questao seguinte foi proposta pelo *Conway em outra lista :

Seja f(x)=x^2 + x + 1. Prove que para todo numero natural N  1, os numeros 
f(N), f(f(N)), f(f(f(N))), f(f(f(f(N, ... sao dois a dois primos entre 
si.

Um Abraco
Paulo Santa Rita
3,1752,040203

*Ou foi pelo Conway ou foi pelo Katz. Nao me lembro ao certo.

From: Cláudio \(Prática\) [EMAIL PROTECTED]
Reply-To: [EMAIL PROTECTED]
To: [EMAIL PROTECTED]
Subject: [obm-l] Truângulos não-obtusângulos
Date: Tue, 4 Feb 2003 17:10:04 -0200

Caro Paulo:

Segue minha solução para o seguinte problema. Acho que a minha idéia 
inicial
é correta, mas posso ter me enrolado nas somas no final...

Num conjunto de 100 numeros naturais, dois a dois distintos, verifica-se 
que
quaisquer 3 numeros ( iguais ou nao ) podem ser os lados de um triangulo 
nao
obtusangulo ( que nao tem um angulo maior que 90 graus ). Seja A a soma de
todos os perimetros de todos os triangulos ( isosceles, equilateros e
escalenos ) que podem ser construidos com estes 100 numeros. Qual o valor
minimo que A pode ter ?

Chame o conjunto de X, e suponha que seus elementos estão ordenados:
a1  a2  ...  a100.

O triângulo com lados (a1,a1,a100) é não-obtusângulo == todos os outros
triângulos são não-obtusângulos e, além disso:
a100^2 = a1^2 + a1^2 - 2*a1^2*cos(A) = a1^2 + a1^2 - 2*a1^2*cos(Pi/2) =
a1^2 + a1^2  ==
a100 = a1*raiz(2)

A menor soma dos perímetros irá corresponder aos menores lados. Isso 
implica
que os elementos de X são naturais consecutivos e a1 é o menor natural N 
tal
que N+99 = N*raiz(2)  ==
(N+99)^2 = 2*N^2  ==
N^2 - 198*N - 9801 = 0  ==
N = 99 + 99*raiz(2) == N = 240

Assim, X = {240, 241, ..., 339 }  ==
S = soma dos elementos de X = 28.950.

Sejam:
E = soma dos perímetros dos equiláteros
I = soma dos perímetros dos isósceles não-equiláteros
C = soma dos perímetros dos escalenos

Então:
E = 3*S = 86.850
I = 99*2*S + 100*S - S = 297*S = 8.598.150
C = C(99,2)*S = 4.851*S = 140.436.450

Logo, A = E + I + C = 149.121.450


Um abraço,
Claudio.




=
Instruções para entrar na lista, sair da lista e usar a lista em
http://www.mat.puc-rio.br/~nicolau/olimp/obm-l.html
O administrador desta lista é [EMAIL PROTECTED]
=


_
MSN Messenger: converse com os seus amigos online.  
http://messenger.msn.com.br

=
Instruções para entrar na lista, sair da lista e usar a lista em
http://www.mat.puc-rio.br/~nicolau/olimp/obm-l.html
O administrador desta lista é [EMAIL PROTECTED]
=


[obm-l] Re: [obm-l] Função Iterada

2003-02-05 Por tôpico Paulo Santa Rita
Ola Claudio e demais colegas
desta lista ... OBM-L,

Eu vou encontrar o problema e a minha solucao enviarei novamente para esta 
lista. Talvez, por te-lo reconstituido de memoria, eu tenha colocado uma 
composicao a mais - deve ser so f(n), f(f(N)) e
f(f(f(N)))- no enunciado abaixo. Peco desculpas a todos.

Um Abraco
Paulo Santa Rita
4,1016,050203

OBS : Nao vi o problema. Mais tarde, quando estiver com mais tempo, eu vou 
dar uma olhada e envio a solucao.

From: Cláudio \(Prática\) [EMAIL PROTECTED]
Reply-To: [EMAIL PROTECTED]
To: [EMAIL PROTECTED]
Subject: [obm-l] Função Iterada
Date: Tue, 4 Feb 2003 20:35:22 -0200

Caro Paulo:

Acho que o enunciado abaixo não está correto, pois encontrei um
contra-exemplo: N = 4

Seja f(x)=x^2 + x + 1. Prove que para todo numero natural N  1, os 
numeros
f(N), f(f(N)), f(f(f(N))), f(f(f(f(N, ... sao dois a dois primos entre
si.

N = 4 ==
f(4) = 4^2 + 4 + 1 = 21 ==
f(f(4)) = 21^2 + 21 + 1 = 463  ==
f(f(f(4))) = 463^2 + 463 + 1 = 214.833

Mas MDC( f(4) , f(f(f(4))) ) = MDC( 21, 214.833 ) = 3



Você chegou a olhar o problema da Loteria Matemática?
Escolha 9 subconjuntos de 6 elementos de {1, 2, ..., 36 } tais que, 
qualquer
que seja T - subconjunto de 6 elementos de { 1, 2, ..., 36 } - a interseção
de T com pelo menos um dos 9 subconjuntos escolhidos é vazia.

Eu achei que tinha resolvido, mas descobri um furo na minha solução.



Um abraço,
Claudio.


=
Instruções para entrar na lista, sair da lista e usar a lista em
http://www.mat.puc-rio.br/~nicolau/olimp/obm-l.html
O administrador desta lista é [EMAIL PROTECTED]
=


_
MSN Messenger: converse com os seus amigos online.  
http://messenger.msn.com.br

=
Instruções para entrar na lista, sair da lista e usar a lista em
http://www.mat.puc-rio.br/~nicolau/olimp/obm-l.html
O administrador desta lista é [EMAIL PROTECTED]
=


[obm-l] Loteria Matematica II

2003-02-06 Por tôpico Paulo Santa Rita
Ola Claudio e demais
colegas desta lista ... OBM-L,

Voce tem certeza que o problema e esse ai embaixo ? Mais que isso : esse 
enunciado e um problema ?

Os sub-conjuntos abaixo constituem uma escolha valida :

{1,2,3,4,5,6}
{7,8,9,10,11,12}
{13,14,15,16,17,18}
{19,20,21,22,23,24}
{25,26,27,28,29,30}
{31,32,33,34,35,36}
{1,2,7,8,13,14}
{3,4,9,10,15,16}
{5,6,11,12,17,18}

Numa loteria sao sorteados 1 numeros escolhidos aleatoriamente de 
{1,2,3,...,48,49}. Cada cartao de apostas deve ser preenchido com exatamente 
7 numeros. Uma pessoa pode pode apostar quantos cartoes desejar sem pagar 
nada, desde que quaisquer dois cartoes de sua
aposta tenham, NO MAXIMO, uma dezena em comum. O primeiro premio e dado a 
pessoa que acertar o maior numero de triplos.

1 ) Exiba uma aposta gratuita que tenha a maxima probabibilidade de ganhar o 
primeiro premio
2 ) Qual o valor da probabilidade acima ?

Um Abraco a todos
Paulo Santa Rita
5,1300,060203

Você chegou a olhar o problema da Loteria Matemática?
Escolha 9 subconjuntos de 6 elementos de {1, 2, ..., 36 } tais que, 
qualquer que seja T - subconjunto de 6 elementos de { 1, 2, ..., 36 } -a 
interseção de T com pelo menos um dos 9 subconjuntos escolhidos é vazia.

Eu achei que tinha resolvido, mas descobri um furo na minha solução.



Um abraço,
Claudio.


_
MSN Messenger: converse com os seus amigos online.  
http://messenger.msn.com.br

=
Instruções para entrar na lista, sair da lista e usar a lista em
http://www.mat.puc-rio.br/~nicolau/olimp/obm-l.html
O administrador desta lista é [EMAIL PROTECTED]
=


[obm-l] Re: [obm-l] Função Iterada

2003-02-06 Por tôpico Paulo Santa Rita
Ola Claudio e demais
colegas desta lista ... OBM-L,

E verdade. Verifiquei a mensagem original do Conway. O enunciado correto e : 
Seja f(x)=x^2 + x + 1. Mostre que para todo natural N  1, os numeros N, 
f(N), f(f(N)), f(f(f(N))), ... sao primos entre si.

Um problema trivial. Basta analisar o MDC(N,f(N)).

O problema abaixo nao e facil. Este problema me foi proposto a cerca de 8 
anos atras e a pessoa me contou que o problema havia sido proposto ao Euler, 
que nao o resolveu. Mas nao sei se esta historia e verdadeira.

PROBLEMA : Num poligono convexo de N lados e tal que duas diagonais 
quaisquer nao sao paralelas. Quantos pontos no exterior do poligono sao 
pontos de interseccao de diagonais ?

OBS : Considere que nenhum ponto ( interior ou exterior ao poligono ) e 
ponto de interseccao de mais de duas diagonais.

SUJESTAO : Antes de fazer uma sugestao, gostaria de registrar que o nosso 
colega Alexandre Tessarolo resolveu esta questao aqui nesta lista. A solucao 
dele nao e essa que vou sugerir.

De um vertice partem N-3 diagonais. Se N e par havera uma unica diagonai
N-3 diagonais se encontram. Afora este caso, duas diagonais quaisquer se 
encontraram ou fora ou dentro do poligono.

IMAGINE as diagonais que partem de um vertice. Considerando qualquer uma 
delas em particular, observe que ela cinde o poligono em dois outros 
sub-poligonos que tem um lado em comum ( que e a diagonal sob analise ). 
Qualquer par de diagonais, uma de cada um dos sub-poligonos representam um 
ponto de interseccao no exterior, a excecao daqueles pares que tem um 
vertice comum.

Finalmente, para que nao surjam dificuldades devido a paridade de N, use 
[N], a funcao maximo inteiro : o maior inteiro que nao supera N. Lembre-se 
tambem que em somatorios complicados, o uso de numeros binomiais costuma 
facilitara as coisas.

Um Grande Abraco a Todos
Paulo Santa Rita
5,1214,060203

From: Cláudio \(Prática\) [EMAIL PROTECTED]
Reply-To: [EMAIL PROTECTED]
To: [EMAIL PROTECTED]
Subject: [obm-l] Função Iterada
Date: Tue, 4 Feb 2003 20:35:22 -0200

Caro Paulo:

Acho que o enunciado abaixo não está correto, pois encontrei um
contra-exemplo: N = 4

Seja f(x)=x^2 + x + 1. Prove que para todo numero natural N  1, os 
numeros
f(N), f(f(N)), f(f(f(N))), f(f(f(f(N, ... sao dois a dois primos entre
si.

N = 4 ==
f(4) = 4^2 + 4 + 1 = 21 ==
f(f(4)) = 21^2 + 21 + 1 = 463  ==
f(f(f(4))) = 463^2 + 463 + 1 = 214.833

Mas MDC( f(4) , f(f(f(4))) ) = MDC( 21, 214.833 ) = 3



_
MSN Hotmail, o maior webmail do Brasil.  http://www.hotmail.com

=
Instruções para entrar na lista, sair da lista e usar a lista em
http://www.mat.puc-rio.br/~nicolau/olimp/obm-l.html
O administrador desta lista é [EMAIL PROTECTED]
=



Re: [obm-l] Loteria Matematica II ( correcao )

2003-02-06 Por tôpico Paulo Santa Rita
Ola Pessoal !

No enunciado abaixo leiam : Numa loteria 7 (SETE) numeros escolhidos 
aleatoriamente de {1,2,3, ...,48,49} ...






From: Paulo Santa Rita [EMAIL PROTECTED]
Reply-To: [EMAIL PROTECTED]
To: [EMAIL PROTECTED]
Subject: [obm-l] Loteria Matematica II
Date: Thu, 06 Feb 2003 15:02:16 +

Ola Claudio e demais
colegas desta lista ... OBM-L,

Voce tem certeza que o problema e esse ai embaixo ? Mais que isso : esse 
enunciado e um problema ?

Os sub-conjuntos abaixo constituem uma escolha valida :

{1,2,3,4,5,6}
{7,8,9,10,11,12}
{13,14,15,16,17,18}
{19,20,21,22,23,24}
{25,26,27,28,29,30}
{31,32,33,34,35,36}
{1,2,7,8,13,14}
{3,4,9,10,15,16}
{5,6,11,12,17,18}

Numa loteria sao sorteados 1 numeros escolhidos aleatoriamente de 
{1,2,3,...,48,49}. Cada cartao de apostas deve ser preenchido com 
exatamente 7 numeros. Uma pessoa pode pode apostar quantos cartoes desejar 
sem pagar nada, desde que quaisquer dois cartoes de sua
aposta tenham, NO MAXIMO, uma dezena em comum. O primeiro premio e dado a 
pessoa que acertar o maior numero de triplos.

1 ) Exiba uma aposta gratuita que tenha a maxima probabibilidade de ganhar 
o primeiro premio
2 ) Qual o valor da probabilidade acima ?

Um Abraco a todos
Paulo Santa Rita
5,1300,060203

Você chegou a olhar o problema da Loteria Matemática?
Escolha 9 subconjuntos de 6 elementos de {1, 2, ..., 36 } tais que, 
qualquer que seja T - subconjunto de 6 elementos de { 1, 2, ..., 36 } -a 
interseção de T com pelo menos um dos 9 subconjuntos escolhidos é vazia.

Eu achei que tinha resolvido, mas descobri um furo na minha solução.



Um abraço,
Claudio.


_
MSN Messenger: converse com os seus amigos online.  
http://messenger.msn.com.br

=
Instruções para entrar na lista, sair da lista e usar a lista em
http://www.mat.puc-rio.br/~nicolau/olimp/obm-l.html
O administrador desta lista é [EMAIL PROTECTED]
=


_
MSN Messenger: converse com os seus amigos online.  
http://messenger.msn.com.br

=
Instruções para entrar na lista, sair da lista e usar a lista em
http://www.mat.puc-rio.br/~nicolau/olimp/obm-l.html
O administrador desta lista é [EMAIL PROTECTED]
=


Re: [obm-l] Loteria Matematica II

2003-02-06 Por tôpico Paulo Santa Rita
Ola Pessoal !

O enunciado diz que QUALQUER E verdade. O enunciado se refere a qualquer T







From: Cláudio \(Prática\) [EMAIL PROTECTED]
Reply-To: [EMAIL PROTECTED]
To: [EMAIL PROTECTED]
Subject: Re: [obm-l] Loteria Matematica II
Date: Thu, 6 Feb 2003 16:49:31 -0200

Caro Paulo:

Infelizmente, o problema é um pouco mais difícil do que isso.

Por exemplo, tome o subconjunto A = {1,12,15,19,25,31}

{1,2,3,4,5,6}   == encontra A em 1
{7,8,9,10,11,12}   == encontra A em 12
{13,14,15,16,17,18}   == encontra A em 15
{19,20,21,22,23,24}   == encontra A em 19
{25,26,27,28,29,30}   == encontra A em 25
{31,32,33,34,35,36}   == encontra A em 31
{1,2,7,8,13,14}  == encontra A em 1
{3,4,9,10,15,16}  == encontra A em 15
{5,6,11,12,17,18}  == encontra A em 12

Após muitos e muitos desenhos de diagramas de Venn eu finalmente encontrei
uma solução...minha suspeita é que eu dei sorte!!!

Vou pensar no problema que você propôs.

Um abraço,
Claudio.

- Original Message -
From: Paulo Santa Rita [EMAIL PROTECTED]
To: [EMAIL PROTECTED]
Sent: Thursday, February 06, 2003 1:02 PM
Subject: [obm-l] Loteria Matematica II


Ola Claudio e demais
colegas desta lista ... OBM-L,

Voce tem certeza que o problema e esse ai embaixo ? Mais que isso : esse
enunciado e um problema ?

Os sub-conjuntos abaixo constituem uma escolha valida :

{1,2,3,4,5,6}
{7,8,9,10,11,12}
{13,14,15,16,17,18}
{19,20,21,22,23,24}
{25,26,27,28,29,30}
{31,32,33,34,35,36}
{1,2,7,8,13,14}
{3,4,9,10,15,16}
{5,6,11,12,17,18}

Numa loteria sao sorteados 1 numeros escolhidos aleatoriamente de
{1,2,3,...,48,49}. Cada cartao de apostas deve ser preenchido com 
exatamente
7 numeros. Uma pessoa pode pode apostar quantos cartoes desejar sem pagar
nada, desde que quaisquer dois cartoes de sua
aposta tenham, NO MAXIMO, uma dezena em comum. O primeiro premio e dado a
pessoa que acertar o maior numero de triplos.

1 ) Exiba uma aposta gratuita que tenha a maxima probabibilidade de ganhar 
o
primeiro premio
2 ) Qual o valor da probabilidade acima ?

Um Abraco a todos
Paulo Santa Rita
5,1300,060203

Você chegou a olhar o problema da Loteria Matemática?
Escolha 9 subconjuntos de 6 elementos de {1, 2, ..., 36 } tais que,
qualquer que seja T - subconjunto de 6 elementos de { 1, 2, ..., 36 } -a
interseção de T com pelo menos um dos 9 subconjuntos escolhidos é vazia.

Eu achei que tinha resolvido, mas descobri um furo na minha solução.



Um abraço,
Claudio.


_
MSN Messenger: converse com os seus amigos online.
http://messenger.msn.com.br

=
Instruções para entrar na lista, sair da lista e usar a lista em
http://www.mat.puc-rio.br/~nicolau/olimp/obm-l.html
O administrador desta lista é [EMAIL PROTECTED]
=

=
Instruções para entrar na lista, sair da lista e usar a lista em
http://www.mat.puc-rio.br/~nicolau/olimp/obm-l.html
O administrador desta lista é [EMAIL PROTECTED]
=


_
MSN Messenger: converse com os seus amigos online.  
http://messenger.msn.com.br

=
Instruções para entrar na lista, sair da lista e usar a lista em
http://www.mat.puc-rio.br/~nicolau/olimp/obm-l.html
O administrador desta lista é [EMAIL PROTECTED]
=


Re: [obm-l] Loteria Matematica II

2003-02-06 Por tôpico Paulo Santa Rita
Ola Pessoal !

Pelo que eu entendi, o enunciado diz que QUALQUER que seja T, deve ser 
possivel encontrar AO MENOS UM tal que a intersecao seja vazia. A familia 
que eu sugeri e :

{1,2,3,4,5,6}  subconjunto 1
{7,8,9,10,11,12}  subconjunto 2
{13,14,15,16,17,18} subconjunto 3
{19,20,21,22,23,24} subconjunto 4
{25,26,27,28,29,30} subconjunto 5
{31,32,33,34,35,36} subconjunto 6
{1,2,7,8,13,14} subconjunto 7
{3,4,9,10,15,16} subconjunto 8
{5,6,11,12,17,18} subconjunto 9

Qual um dos seis primeiros tem interseccao vazia com qualquer outros dos 
seis primeiros. Qualquer um dos tres ultimos tem intersecao vazia com os 
subconjuntos de 4 a 6. Isto e : Dado T, existe sempre AO MENOS UM tal que a 
interseccao de T com ele e vazia.

Talvez voce queira dizer ( e eu nao entendi assim ) que T e a familia de 
TODOS os subconjuntos de {1,2,...,35,36} e os nove que nos devemos formar 
devem ser tais que qualquer elemento de T encontre algum dos nove que tenha 
interseccao nula. E isso ?

Um Abraco
Paulo Santa Rita


From: Cláudio \(Prática\) [EMAIL PROTECTED]
Reply-To: [EMAIL PROTECTED]
To: [EMAIL PROTECTED]
Subject: Re: [obm-l] Loteria Matematica II
Date: Thu, 6 Feb 2003 16:49:31 -0200

Caro Paulo:

Infelizmente, o problema é um pouco mais difícil do que isso.

Por exemplo, tome o subconjunto A = {1,12,15,19,25,31}

{1,2,3,4,5,6}   == encontra A em 1
{7,8,9,10,11,12}   == encontra A em 12
{13,14,15,16,17,18}   == encontra A em 15
{19,20,21,22,23,24}   == encontra A em 19
{25,26,27,28,29,30}   == encontra A em 25
{31,32,33,34,35,36}   == encontra A em 31
{1,2,7,8,13,14}  == encontra A em 1
{3,4,9,10,15,16}  == encontra A em 15
{5,6,11,12,17,18}  == encontra A em 12

Após muitos e muitos desenhos de diagramas de Venn eu finalmente encontrei
uma solução...minha suspeita é que eu dei sorte!!!

Vou pensar no problema que você propôs.

Um abraço,
Claudio.

- Original Message -
From: Paulo Santa Rita [EMAIL PROTECTED]
To: [EMAIL PROTECTED]
Sent: Thursday, February 06, 2003 1:02 PM
Subject: [obm-l] Loteria Matematica II


Ola Claudio e demais
colegas desta lista ... OBM-L,

Voce tem certeza que o problema e esse ai embaixo ? Mais que isso : esse
enunciado e um problema ?

Os sub-conjuntos abaixo constituem uma escolha valida :

{1,2,3,4,5,6}
{7,8,9,10,11,12}
{13,14,15,16,17,18}
{19,20,21,22,23,24}
{25,26,27,28,29,30}
{31,32,33,34,35,36}
{1,2,7,8,13,14}
{3,4,9,10,15,16}
{5,6,11,12,17,18}

Numa loteria sao sorteados 1 numeros escolhidos aleatoriamente de
{1,2,3,...,48,49}. Cada cartao de apostas deve ser preenchido com 
exatamente
7 numeros. Uma pessoa pode pode apostar quantos cartoes desejar sem pagar
nada, desde que quaisquer dois cartoes de sua
aposta tenham, NO MAXIMO, uma dezena em comum. O primeiro premio e dado a
pessoa que acertar o maior numero de triplos.

1 ) Exiba uma aposta gratuita que tenha a maxima probabibilidade de ganhar 
o
primeiro premio
2 ) Qual o valor da probabilidade acima ?

Um Abraco a todos
Paulo Santa Rita
5,1300,060203

Você chegou a olhar o problema da Loteria Matemática?
Escolha 9 subconjuntos de 6 elementos de {1, 2, ..., 36 } tais que,
qualquer que seja T - subconjunto de 6 elementos de { 1, 2, ..., 36 } -a
interseção de T com pelo menos um dos 9 subconjuntos escolhidos é vazia.

Eu achei que tinha resolvido, mas descobri um furo na minha solução.



Um abraço,
Claudio.


_
MSN Messenger: converse com os seus amigos online.
http://messenger.msn.com.br

=
Instruções para entrar na lista, sair da lista e usar a lista em
http://www.mat.puc-rio.br/~nicolau/olimp/obm-l.html
O administrador desta lista é [EMAIL PROTECTED]
=

=
Instruções para entrar na lista, sair da lista e usar a lista em
http://www.mat.puc-rio.br/~nicolau/olimp/obm-l.html
O administrador desta lista é [EMAIL PROTECTED]
=


_
MSN Messenger: converse com os seus amigos online.  
http://messenger.msn.com.br

=
Instruções para entrar na lista, sair da lista e usar a lista em
http://www.mat.puc-rio.br/~nicolau/olimp/obm-l.html
O administrador desta lista é [EMAIL PROTECTED]
=


Re: [obm-l] Loteria Matematica II

2003-02-07 Por tôpico Paulo Santa Rita
Ola Claudio e demais
colegas desta lista ... OBM-L,


1) A dificuldade do problema e precisamente exibir uma AGMP. E verdade que 
uma AGMP tera exatamente 56 cartoes ( neste caso ). De maneira geral, se P e 
primo e sao sorteados P numeros de {1,2,...,P^2} o total de cartoes de uma 
AGMP sera de P*(P+1), desde que cada cartao tenha exatmente P dezenas ( eu 
nao coloquei as coisas assim para que tudo ficasse mais facil ).

2) Eu conheco tres caminhos para se montar uma AGMP : usando numeros 
binomiais, planos afins ou matrizes. Em verdade, se P e primo, toda potencia 
de P (P^N) permite uma AGMP. A forma matricial e uma generalizacao do 
processo de transposicao de matrizes. Penso que este e o caminho mais 
acessivel pra voce.

Sugestao ( ; = fim de linha. Modelo : [linha 1 ; linha 2] )

[1,2 ; 3,4],[1,3 ; 2;4],[1,4 ; 2;3]
E uma AGMP para P = 2. Construa AGMP's para P=3,5,... e procure descobrir a 
lei de formacao. E uma generalizacao do processo de transposicao de 
matrizes.

3)Uma aposta pode ser um cartao ou varios. O total de triplos que uma 
pessoa acerta e a soma dos acertos de cada um de seus cartoes. Ganha o 
primeiro premio que acertar mais triplos e nao quem tenha o cartao 
individual que acertou mais triplos.

Exemplo :

Joao, 1 cartao : acertou 3 triplos
jose, 2 cartoes : 2 triplos certos no primeiro cartao, 2 triplos certos no 
segundo cartao. Total : 4 triplos.
Vencedor : Jose.

4) A quantidade de triplos apostados de uma AGMP e constante. Existem uma 
quantidade enorme de AGMP's

5) Em verdade, as AGMP's sao estruturas de grande beleza ... considere que 
UM DUPLO e um PEIXE PEQUENO. Uma AGMP seria, assim, uma rede que pega todos 
os duplos, isto e, todos os peixes pequenos, qualquer que seja o sorteio que 
algum *GMM bolar ... E possivel extender essas AGMP's de forma que um 
conjunto de AGMP's se torna uma rede de outra categoria que apanha todos os 
triplos. Um conjunto dessas ultimas pega todos as quadras e assim 
sucessivamente ...

Se voce considerar que um DUPLO e UMA COMUNICACAO e um CARTAO DE APOSTAS e 
uma equipe de pessoas, uma AGMP seria uma rede de comunicacao na qual cada 
duas pessoas estariam juntas uma unica vez em determinada equipe e qualquer 
duas pessoas estiveram ou estarao juntas, tudo isso acontecendo uma unica 
vez ... Quer dizer, voce pode aplicar isso em treinamento, telecomunicacoes 
etc.

Eu nunca li na literatura Matematica alguem falando sobre AGMP's. Eu 
descobri estas coisas ha muito tempo atras, generalizando o conceito de 
transposicao de matrizes, de forma a incluir nesta generalizacao o processo 
de Linnus Pauling para determinacao dos niveis energeticos dos eletrons em 
Mecanica Quantica.

Eu acho que isso e suficiente para voce perceber com que estrutura 
maravilhosa voce esta lidando : basta ter olhos adeguadas pra ver ...

Bom, eu vou ficando por aqui. O meu chefe quer uma planilha simploria pra 
controlar os bug's de hardware.

Um Abraco
Paulo Santa Rita
6,1712,070203


From: Cláudio \(Prática\) [EMAIL PROTECTED]
Reply-To: [EMAIL PROTECTED]
To: [EMAIL PROTECTED]
Subject: Re: [obm-l] Loteria Matematica II ( correcao )
Date: Fri, 7 Feb 2003 16:26:27 -0200

Caro Paulo:

Fiz algumas coisas com relação a este seu problema. Gostaria de 
comentários.

Numa loteria sao sorteados 7 numeros escolhidos aleatoriamente de
{1,2,3,...,48,49}. Cada cartao de apostas deve ser preenchido com
exatamente 7 numeros. Uma pessoa pode pode apostar quantos cartoes 
desejar
sem pagar nada, desde que quaisquer dois cartoes de sua
aposta tenham, NO MAXIMO, uma dezena em comum. O primeiro premio e dado a
pessoa que acertar o maior numero de triplos.

1 ) Exiba uma aposta gratuita que tenha a maxima probabibilidade de 
ganhar
o primeiro premio
2 ) Qual o valor da probabilidade acima ?


PARTE 1:
Quaisquer dois cartões de uma aposta gratuita podem ter no máximo 1 número
em comum.
Logo, qualquer par (não ordenado) de números de {1,2,...,49} só pode estar
contido em no máximo 1 cartão.

Seja M o número de cartões numa aposta gratuita de máxima probabilidade
(agmp).

Cada cartão contém C(7,2) = 21 pares de números.
Logo, o número de pares representados nos M cartões é igual a 21*M.

O número total de pares contidos em {1,2,...,49} é C(49,2) = 1.176.

Logo, vale a desigualdade: 21*M = 1.176  ==  M = 56.

Logo, uma agmp consiste de, no máximo, 56 cartões.

Pode ser que seja possível uma agmp com exatamente 56 cartões mas eu não
consegui construir uma.



PARTE 2:
Com 7 números por cartão, o número de triplos contidos em cada cartão é
C(7,3) = 35. Como são sorteados 7 números, o número de triplos vencedores
também é igual a 35.

Além disso, um dado triplo pode estar contido em no máximo um cartão de uma
agmp, a qual, portanto, estará apostando em 35*M triplos.

Neste ponto surgiu a minha dúvida: o primeiro prêmio é dado a quem tiver o
cartão com o maior número de triplos vencedores ou a quem tiver a maior 
soma
dos números de triplos vencedores em cada cartão apostado. Por

[obm-l] Re: [obm-l] Re: [obm-l] Número de Erdös

2003-02-10 Por tôpico Paulo Santa Rita
Ola Pessoal,

Falando em Erdos, aqui esta um problema famoso e antigo resolvido pelo Erdos 
( Cmado Teorema das 13 esferas ) - que Newton nao resolveu - de uma forma 
extremamente trabalhosa, mas que permite uma outra solucao, bem mais simples 
:

PROBLEMA : Seja C uma esfera de raio R, fixa. Tangentes (externamente) a C 
vamos colocando outras esferas C1, C2, ...  todas de raio R. Qual a 
quantidade maxima de esferas que podemos colocar ?

SUGESTAO : Coloque C1 e IMAGINE que voce esta em O, o centro de C. IMAGINE 
todas as semi-retas que partem de O e que sao tangentes a C1. Isto define 
um angulo-solido. Qual o valor desta angulo solido ?

O valor de um angulo-solido e medido em esfero-radianos  e, por definicao, e 
a divisao entre a area que ele intercepta na superficie esfera e o quadrado 
do raio. Assim, claramente, numa esfera ha 4pi esfero-radianos.

Calcule a ara da calota esferica ( existem formulas prontas ) e divida pelo 
quadrado do raio que voce encontrara o valor o angulo solido.

Claramente que toda nova esfera colocada representa um novo angulo-solido de 
mesmo valor. IMAGINE agora tres esferas tao proximas quanto possivel. Voce 
percebera que :

1) Surge uma regiao central que nao esta contida em nenhuma das tres calotas 
iguais definidas pelo tres angulo solidos. Qual o valor, em esfero radianos, 
do angulo solido correspondente a esta area ?

Os planos que contem O e dois outros centros de duas das tres esferas C1, 
C2, C3 interceptam a superficie de C segundo um triangulo esferico 
equilatero. A Area deste triangulo e (A+B+C - pi)*R^2 onde A, B e C sao ao 
angulos do triangulo esferico ( formados pelas tangente a esfera C nos 
vertices A, B e C. Tendo a area temos o angulo-solido correspondente. 
Subtrando esta area dos gomos em C1, C2 e C3 calculamos o valor da regiao 
central.

2) Toda nova esfera colocada com maxima aproximacao entre duas outras ja 
existente fara surgir um novo angulo-solido ( que ja calculamos ) e uma nova 
regiao ( que calculamos em 1). Esses sucessivos acrescimos nao podem 
ultrapassar 4*pi esfero-radianos ...

Um Abraco
Paulo Santa Rita
2,1230,100203

From: Cláudio \(Prática\) [EMAIL PROTECTED]
Reply-To: [EMAIL PROTECTED]
To: [EMAIL PROTECTED]
Subject: [obm-l] Re: [obm-l] Número de Erdös Date: Mon, 10 Feb 2003 
11:42:15 -0300

Número de Erdos é a distância de uma dada pessoa até Paul Erdos em termos 
de co-autoria de artigos matemáticos.

Assim, se você escreveu um artigo em co-autoria com o Paul Erdos, você tem 
Número de Erdos = 1.

Se você nunca escreveu um artigo junto com ele, mas escreveu um em 
co-autoria com alguém que tem Número de Erdos = 1, então você tem Número de 
Erdos = 2.

Em geral, se dentre os Números de Erdos de cada pessoa com quem você 
escreveu artigos, o menor é N, então o seu Número de Erdos é N+1.

Um abraço,
Claudio.
  - Original Message -
  From: [EMAIL PROTECTED]
  To: [EMAIL PROTECTED]
  Sent: Sunday, February 09, 2003 8:35 AM
  Subject: [obm-l] Número de Erdös


  Olá pessoal,

  Alguém poderia me dar uma explicação consistente do que seria o número 
de Erdös ?



_
MSN Messenger: converse com os seus amigos online.  
http://messenger.msn.com.br

=
Instruções para entrar na lista, sair da lista e usar a lista em
http://www.mat.puc-rio.br/~nicolau/olimp/obm-l.html
O administrador desta lista é [EMAIL PROTECTED]
=


[obm-l] Tres belos problemas

2003-02-11 Por tôpico Paulo Santa Rita
Ola Pessoal,

Seguem abaixo tres problemas :

1) Um quadrado e um triangulo estao circunscritos a um circulo de lado 
unitario. Prove que, qualquer que seja a posicao do quadrado e do triangulo, 
a area comum aos dois e maior que 17/5. E possivel afirmar que ela e maior 
que 7/2  ?

2) ( Olimpiada Espanhola ) Em uma reuniao existem exatamente 201 pessoas de 
5 nacionalidades diferentes. Sabe-se que em cada grupo de 6 pessoas, ao 
menos duas tem a mesma idade. Demonstrar que existem ao menos 5 pessoas do 
mesmo pais, da mesma idade e do mesmo sexo.

3) ( Olimpiada Russa ) Na regiao delimitada por um retangulo de largura 4 e 
altura 3 sao marcados 6 pontos. Prove que existe ao menos um par destes 
pontos cuja distancia entre eles nao e maior que Raiz_Quad(5).

Estes problemas nao precisam de sugestao.

Um Grande Abraco a Todos !
Paulo Santa Rita
3,1455,110203




_
MSN Messenger: converse com os seus amigos online.  
http://messenger.msn.com.br

=
Instruções para entrar na lista, sair da lista e usar a lista em
http://www.mat.puc-rio.br/~nicolau/olimp/obm-l.html
O administrador desta lista é [EMAIL PROTECTED]
=


[obm-l] Re: [obm-l] Previdência

2003-02-12 Por tôpico Paulo Santa Rita
Ola Claudio e demais
colegas desta lista ... OBM-L,

A minha intencao era ressaltar o fato de que o tempo normal de contribuicao 
( 30 a 35 anos ) e MAIS QUE SUFICIENTE para que um trabalhador qualquer se 
aposente com um salario integral, sem dpender da contribuicao do pessoal da 
ativa. Se ha deficit e porque o governo nao geriu com responsabilidade e 
inteligencia as contribuicoes mensais que os aposentados, quando na ativa, 
compulsoriamente faziam.

O que me revoltou foi ver um ancora de um programa jornalistico de grande 
audiencia, com garbo e aparente seriedade, iludindo as pessoas mais simples 
- que nao refletem sobre o que ouvem e leem - ao mascarar as reais causas do 
deficit da previdencia. Parece-me que e dever das autoridades de todos os 
niveis esclarer e proteger os menos afortunados, nao engana-los ...

Perdao pelo desabafo ( off-topic ). Vou me redimir :

Nao sei se voce percebeu, mas na resolucao abaixo voce usou o conceito de 
EQUIVALENCIA DE CAPITAIS. Dois conjuntos de capitais sao equivalentes EM 
DETERMINADA DATA se, transpostos para esta data, representam o mesmo valor.

PROBLEMA :

Basicamente existem dois regimes de juros : simples e composto.

1) Prove que no sistema de juros composto, dois conjuntos de capitais 
equivalentes em uma determinada data sao equivalente em qualquer outra.
2) Pode, no sistema de juros simples, dois conjuntos de capitais serem 
equivalentes em MAIS DE UMA DATA ?

Um Grande abraco a todos !
Paulo Santa Rita
4,1826,120203

From: Cláudio \(Prática\) [EMAIL PROTECTED]
Reply-To: [EMAIL PROTECTED]
To: [EMAIL PROTECTED]
Subject: [obm-l] Previdência
Date: Wed, 12 Feb 2003 17:31:40 -0300

Caro Paulo:

A idéia aqui é igualar o valor futuro das contribuições e o valor presente
do benefício, ambos calculados na data da aposentadoria.

 PROBLEMA : Suponha que todo mes e descontado compulsoriamente de um
 trabalhor 10% do seu salario e que o empregador contribua tambem com 
10%(
 sistema bi-partide ). Suponha que esses 20% ( geridos pelo governo ) sao
 aplicados juros de 0.5% ao mes, que sao capitalizados ( A correcao
monetaria
 e um direito. Pode ser desconsiderada para este calculo ). Qual a
quantidade
 minima de anos que o trabalhador deve contribuir para que ao fim desse
 periodo ele possa receber durante 20 anos o valor integral de seu ultimo
 salario ?

N = número de meses de contribuição
S = salário mensal do trabalhador

Valor Presente do Benefício, calculado em t = N:
VP(Benef) = S/1,005 + S/1,005^2 + ... + S/1,005^(20*12) =
= (S/1,005)*[(1/1,005)^240 - 1]/(1/1,005 - 1) = 139,58077*S   (*)

Valor Futuro (em t=N) das Contribuições Mensais de 0,20*S
VF(Contrib) = 0,20*S*[ 1 + 1,005 + 1,005^2 + ... + 1,005^(N-1) ] =
0,20*S*(1,005^N - 1)/(1,005 - 1) = 40*S*(1,005^N - 1)   (**)

Igualando (*) e (**), teremos:
1,005^N - 1  =  139,58077/40 = 3,489519  ==
1,005^N = 4,489519  == N = ln(4,489519)/ln(1,005) = 301,099 meses

Ou seja, o trbalhador terá que contribuir durante 302 meses, ou 25 anos e 2
meses (se eu não errei nas contas)

Um abraço,
Claudio.


=
Instruções para entrar na lista, sair da lista e usar a lista em
http://www.mat.puc-rio.br/~nicolau/olimp/obm-l.html
O administrador desta lista é [EMAIL PROTECTED]
=


_
MSN Hotmail, o maior webmail do Brasil.  http://www.hotmail.com

=
Instruções para entrar na lista, sair da lista e usar a lista em
http://www.mat.puc-rio.br/~nicolau/olimp/obm-l.html
O administrador desta lista é [EMAIL PROTECTED]
=



[obm-l] Dois belos problemas

2003-02-13 Por tôpico Paulo Santa Rita
Ola Pessoal,

Preservando e promovendo o CARATER OLIMPICO que esta nossa lista sempre 
teve, seguem dois novos PROBLEMAS OLIMPICOS :

(1-ASIATICO PACIFICO) Seja S o conjunto de todos os triangulos ABC que teem 
um mesma base fixa AB e altura relatica a AB (tracada de C) constante e 
igual a H. Para quais destes triangulos o produto de suas alturas e maximo ?




(2-CHINA) 10 pessoas chegaram a uma livraria. Sabe-se que :

A) Todos as pessoas compraram livros de 3 disciplinas
B) Para quaisquer duas pessoas existe ao menos uma disciplina sobre a qual 
ambas compraram livros.

Enumerando-se as disciplinas sobre as quais ha livros na livraria, seja M(i) 
o numero de pessoas que compraram livros da disciplina i. Qual e o menor 
valor positivo possivel para o MAXIMO de {M(1), M(2), ... } ?


Estes problemas nao precisam de sugestao.

Um Grande abraco a todos
Paulo Santa Rita
5,1610,130203


_
MSN Hotmail, o maior webmail do Brasil.  http://www.hotmail.com

=
Instruções para entrar na lista, sair da lista e usar a lista em
http://www.mat.puc-rio.br/~nicolau/olimp/obm-l.html
O administrador desta lista é [EMAIL PROTECTED]
=


[obm-l] Negocio da China !

2003-02-19 Por tôpico Paulo Santa Rita
Ola Pessoal,

Seguem abaixo as traduçoes ( do ingles ) de tres problemas de Olimpiadas da 
China.

(1 - CHINA 1990 ) S e o conjunto de todos os sub-conjuntos de um dado 
conjunto X que teem um mesmo numero de elementos e F e uma funcao real 
definida sobre S tal que F(A)  1990 para algum elemento A de S. Sabe-se 
tambem que : F(B uniao C)=F(B)+ F(C)-1990 para todos elementos B e C de 
S que sejam disjuntos. Mostre que nos podemos encontrar um
sub-conjunto Y de X tal que :

F(D)   1990 para todo D contido em Y
F(D) = 1990 para todo D contido em X-Y

(2 - CHINA 1992 ) As diagonais de um quadrilatero ciclico (inscritivel) 
encontram-se em X. O circulo circunscrito ao triangulo ABX encontra o 
circulo circunscrito ao triangulo CDX em X e Y. Se O e o centro do circulo 
circunscrito ao quadrilatero ABCD e O,X e Y sao distintos dois a dois, 
mostre que OY e perpendicular a OX.

(3 - CHINA 1994 ) Seja p(z)= z^N + An-1*z^(N-1) + ... + A0 um polinomio com 
coeficientes complexos. Mostre que nos podemos encontrar um ponto (numero 
complexo) z com modulo(z') = 1 e tal que
modulo(p(z')) = 1 + modulo(A0).

Um Abraco a Todos
Paulo Santa Rita
4,1209,190203






_
MSN Messenger: converse com os seus amigos online.  
http://messenger.msn.com.br

=
Instruções para entrar na lista, sair da lista e usar a lista em
http://www.mat.puc-rio.br/~nicolau/olimp/obm-l.html
O administrador desta lista é [EMAIL PROTECTED]
=


[obm-l] O numero fi

2003-02-20 Por tôpico Paulo Santa Rita
Ola Pessoal,

O numero fi ( letra grega ) e um dos numeros mais notaveis da Matematica. 
Ele aparece no problema geometrico de dividir um segmento em Media e 
extrema razao ( expressao devida aos gregos ).

Seja L o comprimento de um segmento AB. Encontrar um ponto C interior a AB 
tal que :

AC/CB = AB/AC.

Tomando A por origem (AC=x) , fica :

x/(L-x)=L/x = x^2 + Lx - L2 = 0
Daqui : x=( (raiz_2(5) - 1)/2 )*L. Fazendo L=1
x = ( (raiz_2(5) - 1)/2 ). Essa solucao e o numero fi, isto e :

fi = ( (raiz_2(5) - 1)/2 )

O numero fi mantem uma relacao muito proxima com a beleza e tem relacoes 
matematicas muito interessantes ...

PROBLEMA 1) fi e uma das solucoes de x^2 + x - 1=0. Exiba uma sequencia de 
numeros reais, estritamente crescente, tal que ela seja simultaneamente uma 
PA e uma PG. Esta sequencia e unica ou existe
outra(s) ?

PROBLEMA 2) Seja 1, 1, 2, 3, 5, 8, 13, ..., An, ... a sequencia de fibonaci. 
Qual o LIM An/An-1 quando n tende ao infinito ?

Um Abraco a Todos
Paulo Santa Rita
5,1130,200203










_
MSN Hotmail, o maior webmail do Brasil.  http://www.hotmail.com

=
Instruções para entrar na lista, sair da lista e usar a lista em
http://www.mat.puc-rio.br/~nicolau/olimp/obm-l.html
O administrador desta lista é [EMAIL PROTECTED]
=


Re: [obm-l] O numero fi

2003-02-21 Por tôpico Paulo Santa Rita
Oi Claudio,

Nao ha erro. Claramente que se a-r,a,a+r (uma PA)e uma PG entao
a^2=(a-r)(a+r) = r=0, isto e, uma PA so e PG se os termos forem constantes.

No enunciado abaixo, onde se le PA, leia-se Propriedade Aritmetica :
An+1=An + An-1. Procura-se uma sequencia A1, A2, ... que tenha apropriedade 
aritmetica e que tambem seja uma Progressao Geometrica. Com o numero fi 
existem duas sequencias assim.

A serie : 1, fi, 1+fi, 1+2fi, 2+3fi, 3+5fi,5+8fi, ... e chamada SEQUENCIA 
AUREA. Aqui, fi=(1+raiz_2(5))/2.
Considere agora 1, fi, fi^2, fi^3, ...

Um Abraco
Paulo Santa rita
6,1012,210203


Um Abraco
Paulo Santa Rita
6,1005,210203






From: Cláudio \(Prática\) [EMAIL PROTECTED]
Reply-To: [EMAIL PROTECTED]
To: [EMAIL PROTECTED]
Subject: Re: [obm-l] O numero fi
Date: Thu, 20 Feb 2003 17:31:13 -0300

Caro Paulo:

Ficaria muito satisfeito se você mostrasse onde eu errei na solução do
Problema 1.

 PROBLEMA 1) fi e uma das solucoes de x^2 + x - 1=0. Exiba uma 
sequencia
de
 numeros reais, estritamente crescente, tal que ela seja simultaneamente
uma
 PA e uma PG. Esta sequencia e unica ou existe
 outra(s) ?

Seja A(0) = A
Então, para todo n: A(n) = A + D*n = A*Q^n com:
D  0
e
Q  1 se A  0   ou   0  Q  1 se A  0 (de qualquer forma, Q  1).

n = 1: A(1) = A + D = A*Q
n = 2: A(2) = A + 2D = A*Q^2

(1) == D = A*(Q - 1)
(2) - (1)  == D = A*Q*(Q-1)  == A*(Q-1) = A*Q*(Q-1)

A = 0 == PG é constante == contradição == A  0 ==
Q-1 = Q*(Q-1)
Como Q  1 == Q = 1 == contradição

Assim, não existe tal sequência. De fato, não existem sequer 3 números que
formem, ao mesmo tempo, uma PA e uma PG estritamente crescentes.

*

 PROBLEMA 2) Seja 1, 1, 2, 3, 5, 8, 13, ..., An, ... a sequencia de
fibonaci.
 Qual o LIM An/An-1 quando n tende ao infinito ?

O n-ésimo termo da sequência de Fibonacci tem uma fórmula fechada bem
conhecida e dada por:
A(n) = (1/raiz(5))*[U^n - (-1/U)^n]
onde U = (1+raiz(5))/2
(A(1) = A(2) = 1)

Assim,
A(n)/A(n-1) =
[U^n - (-1/U)^n] / [U^(n-1) - (-1/U)^(n-1)] =
[U - (-1)^n/U^(2n-1)] / [1 - (-1)^(n-1)/U^(2n-2)]

Logo, lim A(n)/A(n-1) = [U - 0]/[1 - 0] = U = (1+raiz(5))/2



Um abraço,
Claudio.

=
Instruções para entrar na lista, sair da lista e usar a lista em
http://www.mat.puc-rio.br/~nicolau/olimp/obm-l.html
O administrador desta lista é [EMAIL PROTECTED]
=


_
MSN Hotmail, o maior webmail do Brasil.  http://www.hotmail.com

=
Instruções para entrar na lista, sair da lista e usar a lista em
http://www.mat.puc-rio.br/~nicolau/olimp/obm-l.html
O administrador desta lista é [EMAIL PROTECTED]
=



[obm-l] Numeros figurados

2003-02-21 Por tôpico Paulo Santa Rita
Ola Pessoal,

Os termos da sequencia 1, 3, 6, 10, ..., (n(n+1))/2, ... sao chamados 
NUMEROS TRIANGULARES, pois considerando cada termo uma quantidade de pontos, 
e sempre possivel desenhar um triangulo com esta quantidade de pontos. Ja os 
termos da sequencia :

1, 4, 9, 16, 25, ..., n^2, ...

Sao chamados NUMEROS QUADRANGULARES : considerando cada termo como uma 
quantidade de pontos, e sempre possivel desenhar um quadrado com qualquer 
deles.

Esse conceito pode ser extendido... Podemos falar em NUMEROS PENTAGONAIS, 
NUMEROS HEXAGONAIS, ETC. Se P e o numero de lados do poligono sobre 
consideracao, os NUMEROS P-AGONAIS sao dados pela equacao :

An = (N/2)*(2 + (N-1)(P-2))

Um problema interessante - e que nao e facil, alerto ! -  e determinar quais 
os numeros que pertencem a duas ( ou mais ) categorias dadas. Por exemplo :

PROBLEMA ) Quais sao os numeros naturais que sao simultaneamente 
triangulares e pentagonais ?

Um Abraco a Todos
Paulo Santa Rita
6,1503,210203



_
MSN Messenger: converse com os seus amigos online.  
http://messenger.msn.com.br

=
Instruções para entrar na lista, sair da lista e usar a lista em
http://www.mat.puc-rio.br/~nicolau/olimp/obm-l.html
O administrador desta lista é [EMAIL PROTECTED]
=


[obm-l] Euler e Goldbach

2003-02-24 Por tôpico Paulo Santa Rita
Ola Pessoal  !

Bom, como nosso colega Igor falou sobre a conjetura de Goldbach, sobre a 
Escola Publica e como eu so estudei em Escolas Publicas, me lembrei de um 
problema que li em uma biblioteca de Escola Publica e que foi proposto a 
Euler pelo Goldbach.

PROBLEMA ) Considere um poligono convexo de N lados. Determine, em funcao de 
N, de quantas maneiras distintas e possivel dividir este poligono em areas 
triangulares usando-se tao somente as diagonais deste poligono.

NOTA : Imagine que o poligono esta fixo, nao podendo girar ou transladar.

SUGESTAO : IMAGINE uma divisao valida ! Entao e possivel imaginar o poligono 
como um quebra-cabeca no qual cada peca e um triangulo ... Dado que de 
cada vertice partem N-3 diagnais, considere sobre tal configurcao o efeito 
de se tracar outras diagonais.

Um Abraco a Todos !
Paulo Santa Rita
2,1533,240203
EM TEMPO : No fundo, conforme aprendi na Escola Publica, o que e valido 
sobre o ensino particular ou publico e a afirmacao do Fernando Pessoa : 
Tudo vale a pena quando a alma nao e pequena !

_
MSN Hotmail, o maior webmail do Brasil.  http://www.hotmail.com
=
Instruções para entrar na lista, sair da lista e usar a lista em
http://www.mat.puc-rio.br/~nicolau/olimp/obm-l.html
O administrador desta lista é [EMAIL PROTECTED]
=


[obm-l] O Primeiro Problema Russo

2003-02-24 Por tôpico Paulo Santa Rita
Ola Pessoal !

Esta mensagem e uma resposta a todos aqueles que me escreveram em off 
perguntando sobre o Primeiro Problema Russo. Se, para alguns, a resposta foi 
demorada, e por absoluta falta de tempo.

O Primeiro Problema Russo ( e 99 outros ) esta em :

http://www.mat.puc-rio.br/~nicolau/psr

Todas as perguntas podem ser resumidas nas 7 respostas abaixo.

1) Eu verifiquei o enunciado. Esta correto.

2) O Problema foi proposto na Olimpiada Nacional da Russia para alunos que 
estudam numa serie proxima ao nosso 3 ano do nivel medio.

3) Sim, voces podem resolve-lo usando a teoria dos grafos : considere cada 
regiao como um ponto e o caminho ligando duas regioes como uma aresta. 
Lembrem-se que que num grafo so ha um caminho euleriano se o grau de cada 
vertice e par

4) Sim, existe uma maneira de resolver sem usar a teoria dos grafos.
Basta observar que se ha um poligono convexo de N lados em um plano e N e 
par, entao se eu partir de fora do poligono e cruzar todas as arestas uma 
unica vez vou terminar tambem do lado de fora do poligono. Se N for impar, 
ocorre o contrario : se eu partir de fora termino dentro e vice-versa.

5) Claramente Sim. Voces podem generalizar o raciocinio acima para uma 
regiao arbitaria.

6) Nao. Caminho euleriano e uma coisa, caminho hamiltoniano e outra.

7) Nao. ( em minha opiniao ! ) Nao e necessario usar o teorema de Ramsey. Em 
verdade, nao vejo em que esse teorema pode ser util naquele caso.

Um Abraco a Todos
Paulo Santa Rita
2,1733,240203


_
MSN Messenger: converse com os seus amigos online.  
http://messenger.msn.com.br

=
Instruções para entrar na lista, sair da lista e usar a lista em
http://www.mat.puc-rio.br/~nicolau/olimp/obm-l.html
O administrador desta lista é [EMAIL PROTECTED]
=


[obm-l] Re: [obm-l] Re: [obm-l] demonstrações no dia-a-dia

2003-02-25 Por tôpico Paulo Santa Rita
 a explicacao dos fenomenos, 
isto e, avancamos ate as demonstracoes.

Primeiro voce sente, depois voce pensa ... A emocao precede a razao e o 
raciocinio. Qualquer pessoa que for sincer consigo mesmo e com os outros vai 
admitir que nunca descobre as coisas tal como apresenta em sua 
demonstracoes. Existe algo que precede a demonstracao e que nao e o pensar 
e nao o raciocinar. E o sentir, a intuicao, a inspiracao, a a forma especial 
do intelecto ´perceber os fenomenos matematicos !

Um Abraco
Paulo Santa Rita
3,1734,250203







From: Cláudio \(Prática\) [EMAIL PROTECTED]
Reply-To: [EMAIL PROTECTED]
To: [EMAIL PROTECTED]
Subject: [obm-l] Re: [obm-l] demonstrações no dia-a-dia
Date: Tue, 25 Feb 2003 11:30:15 -0300
Tenho três dúvidas, vejam:

1ª) Um segmento de reta é um exemplo de um corpo UNI-dimensional. Um 
retângulo é um exemplo de um corpo BI- dimensional.tetraedro é um exemplo 
de um corpo TRI-dimensional. E corpos TETRA, PENTA Um , HEXA-dimensionais, 
ou generalizando N-dimensionais como podem ser vistos na natureza ou em 
termos abstratos se for o caso?

Para 4 ou mais dimensões, o mais simples é usar n-uplas ordenadas de 
números reais para representar pontos no espaço n-dimensional.
Assim, um hiper-cubo de 4 dimensões e aresta = 2 teria por vértices os 16 
pontos da forma (+/-1,+/-1,+/-1,+/-1).

Na natureza eu não conheço nenhum exemplo além do espaço-tempo de 4 
dimensões (uma delas é o tempo) no qual nós vivemos - vide qualquer livro 
sobre teoria da relatividade. No entanto, existem teorias que dizem que o 
universo tem na verdade 10 ou 26 dimensões, mas as 6 ou 22 restantes estão 
tão curled up (enroladinhas) que nós não conseguimos percebê-las.

2ª) Eu tinha visto na net há algumas semanas atrás um site (em inglês, mas 
não me lembro o endereço) que dava uma demonstração geométrica (analítica) 
do número imaginário i. A única coisa que me lembro, foi que a 
demonstração foi feita a partir dos eixos cartesianos e havia uma relação 
com o ponto P (-1,0). Há pouco tempo atrás aqui na lista houve algumas 
mensagens explicando muito bem a parte histórica do número i e dos 
números complexos, mas vocês não falaram nada de demonstrações. A única 
coisa mais próxima disso foi quando disseram que o número i surgiu quando 
os matemáticos procuraram resolver a equação raiz (-1) = ?. Mas ainda essa 
passagem eu classifico dentro do contexto histórico do nº imaginário e 
complexo e não uma explicação matemática e real(real no sentido não 
matemático).

Tem um bom artigo sobre isso no livro Meu Professor de Matemática do Elon 
Lages Lima, publicado pela SBM, que fala da relação entre nos. complexos, 
logaritmos, exponenciais e funções trigonométricas.

A meu ver, as propriedades mais importantes dos complexos são os seguintes:
1) Além de se somarem como vetores, os complexos têm uma multiplicação com 
uma interpretação geométrica muito clara, que envolve dilatação/contração e 
rotação.
2) Inicialmente introduziu-se os complexos a fim de que todo polinômio de 
2o. grau com coeficientes reais tenha duas raízes. No entanto, descobriu-se 
que eles eram suficientes para que qualquer polinômio de grau n = 1 e com 
coeficientes complexos tivesse n raízes. Esse resultado é o Teorema 
Fundamental da Álgebra.
3) A extensão dos métodos do cálculo para o domínio dos complexos revelou 
propriedades surpreendentes que não existem no domínio real. Isso tem a ver 
com o fato de que a existência da derivada de uma função complexa é uma 
condição muito mais forte do que a existência da derivada de uma função 
real.

3ª) Uma outra dúvida sobre demosntrações:
Se algum leigo em matemática pedisse a mim ou a qualquer um de vcs para 
provar a existência do número Pi eu e muitos de vcs diriamos a ele para 
medir o comprimento de qualquer circunferência com uma fita métrica  e 
então dividir o valor por 2*raio. (obs: Se ele não soubesse o que era raio 
era só explicar). Agora pergunto:
É possível fazer uma demonstração semelhante (em termos de relação com o 
cotidiano) com o logaritmo neperiano (natural) ?

Medir uma circunferência com uma fita métrica não prova a existência de Pi. 
No máximo dá uma aproximação para o seu valor real.
Pi pode ser definido como a razão entre o comprimento de uma circunferência 
e o seu diâmetro. No entanto, primeiro temos que provar que, para toda e 
qualquer circunferência, a razão entre o comprimento e o diâmetro é 
constante.

A existência de Pi, e, ou de qualquer número real é uma consequência do do 
fato de o conjunto dos reais constituir um (de fato, o único) corpo 
ordenado completo.
Assim, por exemplo, e pode ser definido como o número real tal que:
  e
INTEGRAL  dx/x = 1.
  1
Pode-se provar (com base no completamento dos reais) que essa integral 
converge para um número real, que se convencionou chamar de e (acho que 
foi Euler que deu este nome).
Além disso, pode-se provar que e também é o limite das sequências:
An = (1 + 1/n)^n
ou
Bn = 1 + 1/1! + 1/2! + ... + 1/n!.

Um abraço,
Claudio

Re: [obm-l] Problemas em Aberto III

2003-03-04 Por tôpico Paulo Santa Rita
Ola Prof Nicolau e demais
colegas desta lista ... OBM-L,
Representando por (um ponto) N a nave, traçamos todas as tangentes à esfera 
que passam por N. Isso define uma calota na esfera. Se o alienígena estiver 
nesta calota entao entende-se que a Nave o
encontrou.

A solução deve prescindir da distancia da Nave à superficie do planeta e de 
eventuais variações nos sentidos e modulos das velocidades envolvidas.

A ideia - do Claudio/Pratica/ - de publicar os problemas que foram 
propostos nesta lista e que permanecem em aberto me parece muito boa

Um famoso problema pode se reformulado da seguinte maneira :

Seja S o conjunto de todas as sequencia FINITAS de INTEIROS POSITIVOS
tais que se {Xn}=X1, X2, ...,Xn pertence a S entao para todo P  N,
X1+X2+...+Xp NAO E congruo a 1 modulo 3. Mostre que existe uma bijecao entre 
S e o conjunto de todos os impares positivos.

Um Abraço
Paulo Santa Rita
3,1900,040303
From: Nicolau C. Saldanha [EMAIL PROTECTED]
Reply-To: [EMAIL PROTECTED]
To: [EMAIL PROTECTED]
Subject: Re: [obm-l] Problemas em Aberto III
Date: Sun, 2 Mar 2003 10:04:30 -0300
25) Um alienígena move-se na superfície de um planeta com velocidade
não superior a U. Uma espaçonave que procura pelo alienígena move-se com 
velocidade V. Prove que a espaçonave sempre  poderá encontrar o 
alinígena se V  10U.
Não entendi nada. Quando é que a nave encontra o alienígena?

[]s, N.


_
MSN Messenger: converse com os seus amigos online.  
http://messenger.msn.com.br

=
Instruções para entrar na lista, sair da lista e usar a lista em
http://www.mat.puc-rio.br/~nicolau/olimp/obm-l.html
O administrador desta lista é [EMAIL PROTECTED]
=


Re: [obm-l] ajuda...

2003-06-04 Por tôpico Paulo Santa Rita
Oi Celso e demais
colegas desta lista ,
Voce deve estar querendo descobrir o MENOR VALOR INTEIRO E POSITIVO. Realco 
: E POSIVITO !
Para ver isso, tome r=-3. Haverao 4 solucoes e no entanto, r=-3 e inteiro, 
menor que qualquer das alternativas apresentadas. Supondo esta correcao :

Claramente que deve ser X # 0 ( # significa DIFERENTE DE ). Substituindo Y 
na equacao do circulo ficara :

X^2 + 1/(X^2) = R^2
X^4 - (R^2)X^2 + 1 = 0
Pelas relacoes de Girard ( relacoes entre coeficientes e raizes ) vemos que 
o produto ( c/a=1 ) e a soma das raizes (-b/a=R^2) sao positivos. Resta 
portanto impor que o discriminante seja positivo, isto e, que :

R^4 - 4  0

Os zeros de R^4 - 4 sao RAIZ(2)  e -RAIZ(2) e a inequacao sera satisfeita 
para R  -RAIZ(2) ou
R  RAIZ(2). Portanto,  omenor inteiro positivo e R  RAIZ(2), isto e : R=2.

Agora, considere a seguinte variante de seu problema :

Dentre todos os valores de R para os quais o sistema :

Y=1/X
(X-A)^2 +(Y-B)^2 = R^2
tem exatamente 3 solucoes reais, caracterize aquele(s) que tem a soma A+B 
como Minimo.

Um Abraco a Todos
Paulo Santa Rita
3,2013,030603

From: Celso Junior dos Santos Francisco [EMAIL PROTECTED]
Reply-To: [EMAIL PROTECTED]
To: [EMAIL PROTECTED]
Subject: [obm-l] ajuda...
Date: Mon, 2 Jun 2003 13:55:27 -0300 (ART)
(UERJ)-Observe o sistema:

y=1/x
x^2+y^2=r^2
O menor valor inteiro de r para que o sistema acima apresente  quatro 
soluções reais é:
a)1b)2c)3   d)4

Gabarito: b



-
Yahoo! Mail
Mais espaço, mais segurança e gratuito: caixa postal de 6MB, antivírus, 
proteção contra spam.
_
MSN Hotmail, o maior webmail do Brasil.  http://www.hotmail.com
=
Instruções para entrar na lista, sair da lista e usar a lista em
http://www.mat.puc-rio.br/~nicolau/olimp/obm-l.html
=


Re: [obm-l] Correcao !

2003-06-04 Por tôpico Paulo Santa Rita
Correcao : Perdao. Na mensagem abaixo : A e B devem ser inteiros e a soma um 
MINIMO POSITIVO.


From: Paulo Santa Rita [EMAIL PROTECTED]
Reply-To: [EMAIL PROTECTED]
To: [EMAIL PROTECTED]
Subject: Re: [obm-l] ajuda...
Date: Tue, 03 Jun 2003 23:15:06 +
Oi Celso e demais
colegas desta lista ,
Voce deve estar querendo descobrir o MENOR VALOR INTEIRO E POSITIVO. Realco 
: E POSIVITO !
Para ver isso, tome r=-3. Haverao 4 solucoes e no entanto, r=-3 e inteiro, 
menor que qualquer das alternativas apresentadas. Supondo esta correcao :

Claramente que deve ser X # 0 ( # significa DIFERENTE DE ). Substituindo Y 
na equacao do circulo ficara :

X^2 + 1/(X^2) = R^2
X^4 - (R^2)X^2 + 1 = 0
Pelas relacoes de Girard ( relacoes entre coeficientes e raizes ) vemos que 
o produto ( c/a=1 ) e a soma das raizes (-b/a=R^2) sao positivos. Resta 
portanto impor que o discriminante seja positivo, isto e, que :

R^4 - 4  0

Os zeros de R^4 - 4 sao RAIZ(2)  e -RAIZ(2) e a inequacao sera satisfeita 
para R  -RAIZ(2) ou
R  RAIZ(2). Portanto,  omenor inteiro positivo e R  RAIZ(2), isto e : 
R=2.

Agora, considere a seguinte variante de seu problema :

Dentre todos os valores de R para os quais o sistema :

Y=1/X
(X-A)^2 +(Y-B)^2 = R^2
tem exatamente 3 solucoes reais, caracterize aquele(s) que tem a soma A+B 
como Minimo.

Um Abraco a Todos
Paulo Santa Rita
3,2013,030603

From: Celso Junior dos Santos Francisco [EMAIL PROTECTED]
Reply-To: [EMAIL PROTECTED]
To: [EMAIL PROTECTED]
Subject: [obm-l] ajuda...
Date: Mon, 2 Jun 2003 13:55:27 -0300 (ART)
(UERJ)-Observe o sistema:

y=1/x
x^2+y^2=r^2
O menor valor inteiro de r para que o sistema acima apresente  quatro 
soluções reais é:
a)1b)2c)3   d)4

Gabarito: b



-
Yahoo! Mail
Mais espaço, mais segurança e gratuito: caixa postal de 6MB, antivírus, 
proteção contra spam.
_
MSN Hotmail, o maior webmail do Brasil.  http://www.hotmail.com
=
Instruções para entrar na lista, sair da lista e usar a lista em
http://www.mat.puc-rio.br/~nicolau/olimp/obm-l.html
=
_
MSN Hotmail, o maior webmail do Brasil.  http://www.hotmail.com
=
Instruções para entrar na lista, sair da lista e usar a lista em
http://www.mat.puc-rio.br/~nicolau/olimp/obm-l.html
=


Re: [obm-l] Vestibular ITA

2003-06-23 Por tôpico Paulo Santa Rita
Ola Tiago e demais colega
desta lista ... OBM-L,
Acredito que o Site :

http://www.penbadu.hpg.com.br

possa lhe ajudar. Da uma olhada. E boa sorte na sua empreitada !

Um Abraco
Paulo Santa Rita
2,1016,230603

From: Tiago Carvalho de Matos Marques [EMAIL PROTECTED]
Reply-To: [EMAIL PROTECTED]
To: [EMAIL PROTECTED]
Subject: [obm-l] Vestibular ITA
Date: Mon, 23 Jun 2003 09:49:18 -0300
Olá,

Meu nome é Tiago.

Tenho 20 anos, atualmente estudo Física na USP à noite e durante o dia 
trabalho como consultor em análise e
programação.

Estou disposto a me sacrificar até o limite para entrar no ITA ou POLI.
Na verdade meu sonho é entrar no ITA.
Sei que o melhor seria eu sair do emprego, mas, sinceramente, meu emprego 
não é ruim no que diz respeito ao
retorno financeiro.
Realmente não é um emprego que se pode facilmente desperdiçar na atual 
situação em que estamos.
Além disso, caso eu entre em uma dessas duas universidades vou precisar de 
dinheiro para me manter sem
trabalhar. Estou guardando.

Pretendo trancar a Fisica e estudar todos os dias à noite e mais o fim de 
semana.

Estou me planejando.
Preciso saber o que estudar, que exercicios fazer.
Não pretendo assistir às aulas do cursinho pois assim praticamente não 
terei tempo de fazer exercícios. E eu
pessoalmente acho que aprendo mais com exercícios do que com aulas. 
Sinceramente julgo que possuo
facilidade de assimilação.

Como terei apenas o período da noite e meio ano para estudar, preciso 
adotar um meio bem eficiente.
Não sei se vale a pena sair fazendo exercícios das apostilas do cursinho, 
pois tem muitos fáceis e poucos do
ITA...

Queria ajuda no sentido de saber um meio eficiente de estudo, materiais 
eficientes.
Estou disposto a seguir um método rigoroso, mas precisa ser algo eficiente.
Estou disposto a me sacrificar.

Preciso de ajuda para saber qual o melhor material, os melhores exercicios, 
foruns e onde achar provas dos 10
ultimos anos do ITA e fuvest...

Podem, por favor, me ajudar?

Muito Obrigado!
Tiago.
=
Instruções para entrar na lista, sair da lista e usar a lista em
http://www.mat.puc-rio.br/~nicolau/olimp/obm-l.html
=
_
MSN Hotmail, o maior webmail do Brasil.  http://www.hotmail.com
=
Instruções para entrar na lista, sair da lista e usar a lista em
http://www.mat.puc-rio.br/~nicolau/olimp/obm-l.html
=


Re: [obm-l] COMO PERDER AMIGOS E ENGANAR PESSOAS

2003-07-01 Por tôpico Paulo Santa Rita
Ola jorge e demais
colegas desta lista ... OBM-L,
Se retirarmos uma ficha da “Sacola Predominantemente Verde” – Sacola V – a 
probabilidade dela ( da ficha ) ser verde e 7/10, de ser branca e 3/10. Esta 
retiradas sao repetidas 12 vezes, com reposicao da ficha. Portanto, as 
probabilidades acima se mantem em todas as retiradas, o que significa que 
estamos diante de um ENSAIO DE BERNOULLI. Segue que a probabilidade de 
surgirem 8 fichas verdes e 4 brancas ( evento C ) e :

P(C/V) = BINOM(12,8)*((7/10)^8)*((3/10)^4) = 0.231139696095

Aplicando o mesmo raciocinio para a “Sacola Predominantemente Branca” – 
Sacola B – a probabilidade sera :

P(C/B) =  BINOM(12,8)*((3/10)^8)*((7/10)^4) = 0.007797715695

O que nos precisamos e encontrar as probabilidades condicionais P(B/C) e 
P(V/C) e ver qual e maior. Isso e claramente uma aplicacao do TEOREMA DE 
BAYES :

P(B/C) = P(C/B)*P(B) / [ P(C/B)*P(B) + P(C/V)*P(V) ]
P(V/C) = P(C/V)*P(V) / [ P(C/B)*P(B) + P(C/V)*P(V) ]
Como, claramente : P(B)=P(V)=1/2, ficara :

P(B/C) = (0.5 * 0.007797715695) /[ 0.5 * 0.007797715695 + 0.5*0.231139696095 
]
P(B/C) = 0.007797715695 / ( 0.007797715695 + 0.231139696095 )
P(B/C) = 0.032634971317 = 3,26 %

P(V/C) = 0.231139696095 / (0.007797715695 + 0.231139696095 )
P(V/C) = 1 – P(B/C) = 96,74 %
Portanto, dado que P(V/C)  P(B/C), e mais provavel que as fichas tenham 
sido retiradas da sacola V, isto é, da Sacola Predominantemente Verde.

Este problema e tipicamente uma questao de decisao centrada no TEOREMA DE 
BAYES, um dos teoremas basicos que todo livro de Introducao a Probabilidades 
trata. Agora, considere o problema :

PROBLEMA : Em uma urna existem B bolas brancas e V bolas verdes, que se 
diferenciam apenas pela cor. Realiza-se o seguinte experimento : As bolas 
sao retiradas ao acaso, sem reposicao, ATE QUE A ULTIMA BOLA BRANCA APARECA, 
 quando entao o experimento acaba.

1) O espaco amostral e equiprovavel ? Por que ?
2) Qual a probabilidade que a B-esima bola branca surja na K-esima retirada 
?

Claramente que B = K = B + V

Um Abraco
Paulo Santa Rita
3,1242,010703
From: [EMAIL PROTECTED]
Reply-To: [EMAIL PROTECTED]
To: [EMAIL PROTECTED]
Subject: [obm-l] COMO PERDER AMIGOS E ENGANAR PESSOAS
Date: Mon, 30 Jun 2003 20:34:33 -0300
Olá Pessoal, Boa Noite! Estou pegando carona no título cunhado pelo 
Nicolau. OK!

Tenho duas sacolas com fichas de pôquer. A primeira sacola 
predominantemente
verde contém 70 fichas verdes e 30 brancas. A segunda sacola 
predominantemente
branca contém 70 fichas brancas e 30 verdes. As fichas são todas iguais, 
menos
na cor. Agora, misturo as duas sacolas, de forma que você não possa 
distinguir
uma da outra, e separo uma delas. Suponha agora que você escolha 12 fichas 
ao
acaso, com reposição, da sacola que sobra, verificando-se que saem oito 
fichas
verdes e 4 brancas, em alguma ordem particular. Que indicações, você acha,
existem de que a sacola, da qual você retirou fichas, é predominantemente 
verde?

(TEORIA DA DECISÃO - HOWARD 
RAIFFA)

Fico aguardando as suas valiosas opiniões um abraço e até breve!




WebMail UNIFOR - http://www.unifor.br
=
Instruções para entrar na lista, sair da lista e usar a lista em
http://www.mat.puc-rio.br/~nicolau/olimp/obm-l.html
=
_
MSN Messenger: converse com os seus amigos online.  
http://messenger.msn.com.br

=
Instruções para entrar na lista, sair da lista e usar a lista em
http://www.mat.puc-rio.br/~nicolau/olimp/obm-l.html
=


[obm-l] Re: [obm-l] Re: [obm-l] Demonstração

2003-07-01 Por tôpico Paulo Santa Rita
Ola Denisson e demais
colegas desta lista ... OBM-L,
Interessant, sehr interessant ! Oder ? Mas ... eu acho que nao entendi a sua 
questao :

Segundo a exposicao abaixo segue que o ponto B - extremo do segmento de 
comprimento minimo - fica univocamente determinado ANTES DA DESCOBERTA do 
ponto C, mas me parerce que as coisas nao podem ser assim ...

Dado que a distancia entre quaisquer dois pontos e diferente da distancia 
entre dois outros pontos quaisquer, entao, claramente, o conjunto das 
distancias possiveis tem um valor minimo. Seja M esse valor minimo e { X,Y } 
o par de pontos que lhe corresponde. Portanto, evidentemente, se tracarmos 
um circulo de centro X e raio M nao podera haver nenhum ponto no interior 
deste circulo, pois isto contrariaria a minimalidade de M. O mesmo se pode 
dizer do circulo de mesmo raio e centro Y.

Agora, quem e A e quem e B ? ( X=A e Y=B ) ou ( X=B e Y=A ) ?

Me parece que nos so podemos responder a pergunta acima APOS ANALISAR OS 
DEMAIS PONTOS ...

Suponhamos que P seja o conjunto de pontos e d(X,Y) a distancia entre os 
pontos X e Y. Neste caso, se existe Z pertencente a P - {X,Y} tal que d(Z,X) 
 d(W,Y) qualquer que seja W pertencente a
P - {X,Y,Z},  entao X=B e Y=A. Mas, na explicacao PRESSUPOE-SE que B esta 
univocamente determinado, fato que EU nao consigo perceber ...

Sera que o ponto A e previamente dado ? Isto e, existe um ponto de partida ? 
Ou, de fato, conforme eu suspeito, o ponto B e determinado a posteriori, 
tal como esbocei acima, e nao, a priori, conforme voce implicitamente 
pressupoe em sua exposicao abaixo ? Bom, se voce seriamente quer uma 
discussao, voce precisa se pronunciar.

Considere, finalmente, os pontos : (0,0), (0,1), (2,1) e (-3,0). O segmento 
minimo e { (0,0),(0,1) }.

1)Supondo A=(0,0) segue que B=(0,1). Logo C=(2,1) e D=(-3,0). Portanto, CD 
corta AB
2)Supondo A=(0,1) segue que B=(0,0). Logo C=(2,1) e D=(-3,0). Portanto, CD 
corta AB

O contra-exemplo acima E UMA PROVA de que os segmentos podem se cruzar, se e 
que eu entendi corretamente o seu enunciado ou se o seu enunciado encerra 
algo com sentido  ...

Um Abraco
Paulo Santa Rita
3,1536,010703


From: Denisson [EMAIL PROTECTED]
Reply-To: [EMAIL PROTECTED]
To: [EMAIL PROTECTED]
Subject: Re: [obm-l] Re: [obm-l] Demonstração
Date: Sun, 29 Jun 2003 23:41:51 -0300 (ART)
Ok, vejamos. Imagine uma folha, cheia de pontos, feitos aleatoriamente. A 
distância entre dois pontos distintos nunca será igual a distancia de dois 
outros pontos. Entendido até aí? Se a distancia entre o ponto A e o ponto B 
for 5 cm, então a do ponto An até o Bn deverá ser diferente de 5.
Bom, agora imagine todos os segmentos que nós podemos formar ligando dois 
pontos dessa folha. Imagine que o menor possível é AB=1 cm e o maior é 
CD=10 cm.Então nós devemos traçar o nosso primeiro segmento, a partir 
do ponto A até o ponto B. Agora você está no ponto B, vc deve ligar o ponto 
B ao próximo ponto que estiver mais perto, ou seja, se houver o ponto C a 2 
cm, e o ponto D a 3cm entaõ vc deve ligar B com C. Agora a partir do ponto 
C ligue-o até o outro ponto mais próximo de C e assim sucessivamente.
Vc para de ligar quando todos os pontos forem usados, mas a partir do 
momento que vc chegou no ultimo ponto, acaba suas ligações. imagine que vc 
tem uma folha com quatro pontos. Aí vc liga AB, depois BC, depois 
CD,pronto, pare aí, não ligue o ultimo com o primeiro A. Entendeu agora? 
Agora prove que nunca formará uma linha poligonal fechada nem haverá 
cruzamento de segmentos. Se discordar prove também  :P

Saudações,
Denisson
- Original Message -
From: Domingos Jr.
To: [EMAIL PROTECTED]
Sent: Sunday, June 29, 2003 11:58 PM
Subject: [obm-l] Re: [obm-l] Demonstração
Não entendi direito... especialmente essa parte:
e a partir desse segmento ligar outro ponto com a menor distancia
É pra ligar o ponto ao que com a menor distância?
É pra ligar dois pontos quaisquer cuja distância seja a segunda menor?
Quando você para de traçar segmentos?


-
Yahoo! Mail
O melhor e-mail gratuito da internet: 6MB de espaço, antivírus, acesso 
POP3, filtro contra spam.
_
MSN Messenger: converse com os seus amigos online.  
http://messenger.msn.com.br

=
Instruções para entrar na lista, sair da lista e usar a lista em
http://www.mat.puc-rio.br/~nicolau/olimp/obm-l.html
=


[obm-l] Re: [obm-l] Questão primária ?

2003-07-02 Por tôpico Paulo Santa Rita
Ola Carlos e demais
colegas desta lista ... OBM-L,
Fica dificil ajudar quando nao ha elementos suficientes para tanto, quando 
se torna necessario advinhar do que se esta falando. Acredito que voce 
esteja se referindo ao seguinte :

Coloca-se numa bolsa 16 bolas numeradas, identicas. Realiza-se 
reiteradamente o seguinte experimento : uma bola e retirada ao acaso e 
anota-se o numero correspondente. Apos repetir o eperimento um numero muito 
grande de vezes, observei que a frequencia relativa de alguns numeros 
afasta-se significativamente da probabilidade teorica postulada (1/16). Por 
que isto ocorreu ?

Se for isso, entao e necessario transpor o limite da teoria de 
probabilidades elementar e considerar outros aspectos desta teoria. Em 
particular, a desigualdade de Chebychef. Esta desigualdade, fazendo muito 
poucas hipoteses sobre a natureza da distruicao associada a variavel 
aleatoria, permite medir o grau de distanciamento em relacao a 
probabilidade teorica, que e o que me parece que voce esta observando em sua 
experiencia ...

Desde ja te adianto que estes aspectos da teoria da probabilidades, nao 
obstante nao aparecerem em livros elementares de nivel medio, nao exigem 
conhecimento previo algum e sao, de fato, tambem elementares. A rigor, 
exige-se apenas saber o que e variancia e experanca de uma variavel 
aleatoria, coisas realmente triviais. Vale a pena estudar isso !

Um Abraco
Paulo Santa Rita
4,1105,020703
From: Carlos Sergio Carvalho [EMAIL PROTECTED]
Reply-To: [EMAIL PROTECTED]
To: [EMAIL PROTECTED]
Subject: [obm-l] Questão primária ?
Date: Wed, 2 Jul 2003 00:36:36 -0300
Prezados
Jogando um dado, a probabilidade de ocorrência de qualquer face é 1/6.Se eu 
jogar este dado 60 vezes,o número de ocorrências de qualquer 
face,presume-se que seja 10.Entendo que quanto maior o número de 
operações,mais me aproximaria da NORMAL.Ocorre que em minha distribuição de 
16 bolas numeradas,a disparidade está muito alta.Acredito que tenham o 
mesmo peso.Alguém poderia explicar-me  porque   isto ocorre ?
_
MSN Hotmail, o maior webmail do Brasil.  http://www.hotmail.com
=
Instruções para entrar na lista, sair da lista e usar a lista em
http://www.mat.puc-rio.br/~nicolau/olimp/obm-l.html
=


Re: [obm-l] Dica de problema.

2003-07-03 Por tôpico Paulo Santa Rita
Ola Prof Nicolau e demais
colegas desta lista ... OBM-L,
Eu nao conhecia este problema, mas a sua mensagem - abaixo - e muito 
estimulante.

Existe mais de uma maneira de abordar o problema, todas na dependencia de 
alguma investigacao mais profunda. Uma que ocorre imediatamente a cabeca e a 
seguinte :

INVESTIGACAO 1 )

A primeira linha de um quadrado latino e qualquer das permutacoes que 
podemos fazer com os N elementos 1, 2, 3, ...,N. Podemos fazer isso de N! 
maneiras. De quantas maneiras podemos montar a segunda linha ? Claramente 
que a segunda linha sao todas as PERMUTACOES CAOTICAS  de N elementos ! 
Nicolau Bernoulli e Euler ja calcularam essa quantidade e vale :

N!( 1/(2!)  -  1/(3!)  +  ...  +- 1/(N!)

Quem seria a terceira linha ? Claramente que deve ser uma PERMUTACAO CAOTICA 
em relacao as duas primeiras. Que nome daremos a esta permutacao ? Bom, uma 
ideia pode ser assim : A primeira linha seriam as permitacoes de 1 especie, 
na segunda estariam a de 2 especie, na terceira as de 3 especie e assim 
sucessivamente, ate a ultima linha, onde estaria uma permutacao caotica de 
N-esima especie.

O numero de quadrados latinos, claramente, seria o produto dos numeros 
alencados acima. O problema, pelo que sei, e que nos so sabemos calcular as 
PERMUTACOES CAOTICAS de segunda especie. A solucao dos problema esta, 
portanto, na depemndencia desta investigacao.

E importante notar que esta linha de investigacao tem o duplo sabor de 
resolver em definitivo o problema dos quadrados latinos e ser uma 
continuidade de um trabalho iniciado pelo Euler e Nicolai bernoulli. E 
portanto uma contribuicao nao desprezivel e uma continuidade historica 
interessante.

Eu acredito que o numero de permutacoes caoticas de 2 especie ( e 
posteriores ) devem ser uma sub-serie interessante da seria (1/(2!) - 1/(31) 
+ ... +- 1/(N!) ), dado que esta serie surge nas permutacoes caoticas de 
primeira especie. Enfim, e um caminho interessante ...



INVESTIGACAO 2)

Existe uma classe ampla de quadrados latinos faceis de construir e calcular. 
Para ver isso, considere o caso N=4 :

1234, jogue o 4 para a primeira posicao. Resulta :
4123, jogue o 3 para a primeira posicao. Resulta :
3412  jogue o 2 para a primeira posicao. Resulta :
2341
O quadrado acima e quadrado latino. O que fizemos ? Simplesmente dispomos os 
numeros 1,2,3,4 e percorremos em um sentido previamente fixado a partir dos 
diversos elementos. Isso e o que se chama uma PERMUTACAO CIRCULAR. Estamos, 
pois, identificando um quadrado latino com uma PERMUTACAO CIRCULAR ...

Agora, permutamos, as linhas das diversas formas possiveis, mantendo os 
numeros nas mesmas colunas ( ou o inverso, permutamos as colunas e mantemos 
os elementos nas mesmas linhas ). Isso fornece 4! quadrados latinos, dois a 
dois distintos. Bom, aqui surge uma luz :

Para N elementos, existem (N-1)! permutacoes circulares. Para cada 
permutacao circular associamos um quadrado latino. As linhas podem ser 
permutadas de N! modos. Assim, existem : (N-1)!*N! quadrados latinos 
derivados diretamente da associacao QUADRADO LATINO - PERMUTACAO CAOTICA.

Seja portanto QL(N) o total de quadrados latinos de ordem N, entao :

QL(N) = (N-1)!N! + F(N), onde F(N) e uma funcao que ainda nao conhecemos ...

O que podemos dizer sobre F(N) - numa primeira observacao - que nos permite 
uma aproximacao com a sua forma definitiva ? O seguinte : Dado a associacao 
QUADRADO LATINO - PERMUTACAO CIRCULAR, no computo (N-1)!N! estao TODAS as 
permutacoes circulares de 1,2,...,N, isto e, associando os quadraodos 
latinos computados no calculo de (N-1)!N! pela PERMUTACAO CIRCULAR que o 
originou, teremos todas as permutacoes de 1,2,...,N.  Isso significa, 
claramente, que qualquer outro quadrado latino e, em verdade, uma associacao 
de DUAS ou mais permutacoes circulares, isto e :

F(N) = A1*G(N,2) + A2*G(N,3) + ... + An-1*G(N,N)

onde A1, A2, ..., An sao inteiros nao todos nulos e G(N,P) e o total de 
quadrados latinos obtidos da associacao de P PERMUTACOES CIRCULARES. A 
questao e saber como associar, operar, estas permutacoes circulares de 
forma a gerar os quadrados latinos que serao computados em F(N).

A meu ver, aqui trata-se, a principio, de um trabalho experimental. Pegamos 
os quadrados latinos de ordem N=3, N=4, retiramos aqueles oriundos das 
permutacoes circulares e vemos como estao montados os outros. Deve surgir 
algum padrao de somposicao de permutacoes circulares e, a partir dai, 
adivinhamos a lei geral. E tambem uma linha de pesquisa interessante ...

INVESTIGACAO 3)

Bom, esta ideia a baseada na q-contagem, na transformacao dos indices de um 
quadrado latino em polinomios, mas ela chegou na minha cabeca muito 
rapidamente, quando eu ainda estava escrevendo as coisas acima e nao 
consegui registra-la. Mas senti que era uma ideia legal e promissora. 
Havendo tendo eu me concentro e ela volta e entao eu comunico.

Um Grande Abraco !
Paulo Santa Rita
5,1045,030703
From: Nicolau C. Saldanha [EMAIL

[obm-l] Re:

2003-07-04 Por tôpico Paulo Santa Rita
 coloquei :

QL(N) = (N-1)!N! + F(N), F(N)=somatorio Ai*G(N,i)

Bom, conforme ja falei, respeito a sua sensibilidade, mas a bem da verdade e 
certo que tudo isso sao meras suposicoes. A intuicao nos guia num primeiro 
momento, mas depois e inevitavel enfrentar o caminho nem sempre florido da 
demonstracao. O certo e tratar diretamente a questao, fazer experiencias e 
observacoes e so entao levantar hipoteses de trabalho.

Caro Faccast, a questao que voce trouxe e - como diria o Dr Tchurmann - 
Interessant, sehr interessant !. E seja bem-vindo a LISTA OBM-L !

Com os melhores votos
de paz profunda, sou
Paulo Santa Rita
6,2115,040703
From: [EMAIL PROTECTED]
Reply-To: [EMAIL PROTECTED]
To: [EMAIL PROTECTED]
Date: Fri,  4 Jul 2003 17:30:13 -0300
 Paulo, sua 1a. investigação é o que geralmente se pensa quando nos 
deparamos
com o problema e acho que este é um caminho complicado. A segunda, segue do
fato que a Tábua de um Grupo finito é um Quadrado latino (QL). Eu diria que 
em
vez de
QL(N) = (N-1)!N! + F(N), onde F(N) e uma funcao que ainda nao conhecemos
fosse
QL(N) = (N-1)!N!.F(N), onde F(N) e uma funcao que ainda nao conhecemos 
pois
considerando que, dois QL's estao relacionados quando diferem-se por
permutaçoes de filas, temos uma relaçao de equivalencia onde cada classe 
possui
exatamente n!(n-1)! e a funçao F(n) entraria com a contagem destas classes
dando um total de (N-1)!N!.F(N) QL's de ordem N. Note que permutando as 
filas
de um QL obtem-se novos QL's e com isto, fica fácil cheger ao cardinal 
n!(n-1)!
das classes.

Um abraço,

faccast



-
This mail sent through IMP: http://horde.org/imp/
=
Instruções para entrar na lista, sair da lista e usar a lista em
http://www.mat.puc-rio.br/~nicolau/olimp/obm-l.html
=
_
MSN Messenger: converse com os seus amigos online.  
http://messenger.msn.com.br

=
Instruções para entrar na lista, sair da lista e usar a lista em
http://www.mat.puc-rio.br/~nicolau/olimp/obm-l.html
=


Re: [obm-l] Sugestao para solucao

2003-07-07 Por tôpico Paulo Santa Rita
Oi Marcio !
Tudo Legal ?
Como voce so fala Anel entao eu nao vou supor que ele tem uma unidade. 
Respondendo a sua pergunta : Voce nao errou, apenas nao continuou ...

Dado que NESTE ANEL X^2 = X, entao, em particular, (-X)^2 = -X. Mas, 
conforme voce deve saber, EM QUALQUER ANEL, (-X)^2=X^2. Segue que, NESTE 
ANEL : X^2=-X. Portanto, NESTE ANEL,
X^2=X e X^2 =-X. Logo X=-X. Conclusao : NESTE ANEL, qualquer elemento e 
igual ao seu simetrico !

Voce provou que XY = -(YX). Mas, pelo que vimos, NESTE ANEL, -(YX)=YX. Dai : 
XY=YX. E portanto este Anel e, de fato, comutativo.

Nao entendi a outra questao... E claro que se tomarmos quaisquer outros dois 
inteiros e dividirmos por 5 ( que e o terceiro elemento do conjunto ! )  o 
respo pode ser diferente. Por exemplo 3*5= 15. 15 por 3 da resto zero.

Um Abraco
Paulo Santa rita
2,0948,070703

From: mmrocha1 [EMAIL PROTECTED]
Reply-To: [EMAIL PROTECTED]
To: [EMAIL PROTECTED]
Subject: [obm-l] Sugestao para solucao
Date: Sun,  6 Jul 2003 10:50:25 -0300
Saudacoes a todos!
Faco Matematica na UERJ e gosto de me divertir resolvendo (na verdade, na 
maior
parte das vezes, soh tentando) resolver problemas olimpicos. Nao sou aquilo 
que se
poderia chamar de aluno talentoso, mas sou curioso, persistente e estudo
Matematica porque realmente gosto. Alem disso, penso que estudar temas
olimpicos pode ajudar a melhorar a minha formacao. Jah escrevi para a lista
ajudando na solucao de problemas bem simples, jah fui corrigido pelo Prof 
Morgado,
e tudo isso para mim eh muito bom!
Bem vamos ao que interessa.
Tem dois problemas que gostaria de uma ajuda para resolver. Nao eh 
necessario
dar a solucao, mas uma sugestao jah serve. Um eh de Algebra, do livro do 
Adilson
Goncalves, e o outro eh de uma apostila de treinamento olimpico.

1) Seja A um anel, tal que x^2 = x para todo x de A. Prove que A eh 
comutativo.
A minha tentativa foi a seguinte: Tomei x e y de A. Assim, (x + y)^2 = x + 
y.
Desenvolvendo, temos:
x.x + x.y + y.x + y.y  = x + y.
x^2 + x.y + y.x  +  y^2 = x + y.
Apos a simplificacoes possiveis, cheguei a
xy = -(yx)
Mas isso nao significa que A eh comutativo. Onde errei?

Aqui vai o outro problema.

2) Considere o conjunto formado pelos elementos 2, 3 e 5. Se multiplicamos
quaisquer dois elementos e dividimos o resultado pelo terceiro elemento, o 
resto da
divisao eh 1. Existem outros conjuntos de 3 elementos com esta mesma
propriedade, ou este eh o unico? 
Nao creio que seja dificil, mas nao consigo enxergar um bom caminho.

Gostaria de agradecer a atencao de todos, e me desculpem se os problemas 
nao
sao um desafio intelectual para a maioria.

Um abraco,
Marcio Rocha.
__
Acabe com aquelas janelinhas que pulam na sua tela.
AntiPop-up UOL - É grátis!
http://antipopup.uol.com.br/
=
Instruções para entrar na lista, sair da lista e usar a lista em
http://www.mat.puc-rio.br/~nicolau/olimp/obm-l.html
=
_
MSN Messenger: converse com os seus amigos online.  
http://messenger.msn.com.br

=
Instruções para entrar na lista, sair da lista e usar a lista em
http://www.mat.puc-rio.br/~nicolau/olimp/obm-l.html
=


[obm-l] Re: Como os Matemáticos Complicam II

2003-07-10 Por tôpico Paulo Santa Rita
Ola Pessoal !

A NOSSA LISTA foi originalmente planejada para ser um PONTO DE ENCONTRO 
INFORMAL de Professores de Matematica e de estudantes que se preparam para 
as OLIMPIADAS DE MATEMATICA.
Qualquer destes dois grupos e formado por pessoas que, minimamente, gostam 
de Matematica, quando, nao raro, fazem desta Ciencia a sua maior paixao.

A LISTA vem atingindo seus objetivos e o seu sucesso e inegavel ...

Muitos estudantes olimpicos participam, estudantes do Brasil e do Exterior. 
Muitas pessoas usam os resultados que surgem aqui como inspiracao e 
estimulos para trabalhos escolares e mesmo para dissertacoes e seminarios. 
Pesquisadores escrevem para estas lista e acompanham algumas de suas 
discussoes.

Quando eu disponibilizei a traducao dos Problemas Russos, fiquei 
impressionado como varios estudantes de outros paises me escreveram pedindo 
que lhes remetessem copias. O mesmo ocorrei quando discutimos o Jogo Vida, 
varias pessoas queriam os fontes em C. Tudo isso e indicativo do alto nivel 
de nossa plateia e o quanto e importante imprimirmos qualidade em nossas 
manifestacoes.

O Moderador, Prof Nicolau Saldanha, deixou claro em mensagens anteriores que 
poderiamos discutir Fisica, Teoria da Computacao, Xadrez e mesmo Filosofia e 
Historia da Matematica, desde que o objetivo principal seja evidenciar a 
importancia do raciocinio matematico em todos estes ramos.

Nos podemos PROPOR PROBLEMAS, sobretudo olimpicos. Podemos apresentar 
solucoes, podemos dar uma dica de solucao, podemos mostrar que a solucao de 
um problema implica na solucao de outro e assim por sucessivamente. E 
verdade que a Matematica nao e so problemas, mas parece que toda questao 
matematica so fica bem formulada quando se corporifica num problema. Por 
isso, qualquer questao matematica deve implicar num problema e o problema e, 
em ultima analise, a essencia da comunicacao matematica.

Eu francamente confesso que nao entendo nada de EDUCACAO MATEMATICA, nao 
obstante achar que esta ciencia, caso exista, deve ter questoes e ja ter 
atingido algum objetivo palpavel. Por esta razao, nao entraria numa lista de 
EDUCACAO e penso quem buscar aqui discutir EDUCACAO MATEAMATICA vai se 
frustar, pois estara desvirtuando esta nossa lista, tao bem sucedida em seus 
objetivos originais, conforme destacamos acima.

Finalmente, nos, os Matematicos, nao complicamos as coisas : nos 
SIMPLIFICAMOS ! Nos captamos o que ha de essencial entre as coisas e 
exoressamos isso num simbolismo economico, pois a linguagem matematica, 
diferentemente da linguagem natural, possui poucos simbolos e de significado 
claro. A Matematica, e a masi simples de todas as ciencias. E acho no minimo 
deselegante que alguem entre aqui, na nossa casa, e nos ofendam 
gratuitamente.

Quem nao entende como se pode gostar de Matematica, nao deve permanecer aqui 
!

Um Abraco a Todos !
Paulo Santa Rita
5,1946,100703
From: J.Paulo roxer ´til the end [EMAIL PROTECTED]
Reply-To: [EMAIL PROTECTED]
To: [EMAIL PROTECTED]
Subject: [obm-l] Re:Re: Re: RE: Re:Re: Como os Matemáticos Complicam II
Date: Thu, 10 Jul 2003 16:45:00 -0300
Estou  'falando' muito na lista coisas q não têm muito a ver com mat. e 
espero q o listmaster não me retire... só vou complementar algumas coisas.

Não sou convencido,mas preciso dizer q sou o único na lista que enfrenta o 
bicho papão.Garanto q só está nesta lista pessoas
q estão diretamente envolvidas com matemática,o q não é meu caso.E estou 
enfrentando,por mais q seja um tanto difícil entender tantas coisas 
complexas sem poder aplicar.

Matemática,pra mim não tem a ver com o lado emocional.Só não consigo 
entender bem uma coisa que não estou vendo um motivo lógico pra 
existir(Caso de raiz,x e y etc)
É muito mais fácil eu entender um texto todo em japonês.Pelo menos posso ir 
atrás de um dicionário e traduzir.
Já com matemática,não tem aonde ir.

Tb não entendo muito bem porq vcs são tão pacientes pra entender as 
questões matemáticas,mas não são pra escrever,ler e explicar a origem dos 
problemas matemáticos e a base da existência.

Acho q existe um certo exagero quanto aos assuntos colocados nas 
provas,mais especificamente em vestibulares.
Não sei porq um aluno q não vai ser engenheiro precisa  resolver problemas 
com trigonometria,por exemplo.
Algo q não tem nada a ver :Um futuro estudante de letras resolvendo essas 
questões.

João Paulo

  - Original Message -
  From: Diego Navarro
  To: [EMAIL PROTECTED]
  Sent: Thursday, July 10, 2003 1:33 PM
  Subject: [obm-l] Re: [obm-l] Re: [obm-l] RE: [obm-l] Re: [obm-l] Re: 
Como os Matemáticos Complicam II

  Na boa? Eu não vejo afronta. Vejo um pouco de ingenuidade, mas eu já 
estive errado antes,
  e ainda vou estar errado muitas vezes. E tenho a impressão que no fundo, 
ele levantou um
  ponto interessante - eu só não consigo ver o que é. Talvez porque a 
relação dele com a
  matemática seja uma mistura confusa e emocional de revolta e frustração.

  Eu me identifico um pouquinho com ele

[obm-l] Jornal Eletronico de Combinatoria

2003-07-11 Por tôpico Paulo Santa Rita
Ola Pessoal !

Em :

http://www.combinatorics.org

ha um bom jornal de combinatoria. Neste site ha tambem links para outros 
periodicos cientificos referentes a Matematica. Vale a pena dar uma olhada !

Um Abraco a Todos
Paulo Santa Rita
6,1038,110703
_
MSN Messenger: converse com os seus amigos online.  
http://messenger.msn.com.br

=
Instruções para entrar na lista, sair da lista e usar a lista em
http://www.mat.puc-rio.br/~nicolau/olimp/obm-l.html
=


Re: [obm-l] Conjuntos - Justificativa

2003-07-14 Por tôpico Paulo Santa Rita
Ola Carissimo Jose Francisco
e demais colegas desta lista ... OBM-L,
Isso e realmente uma questao de definicao ... Intuitivamente sentimos que se 
um conjunto e limitado superiormente, entao, evidentemente, ele tem que ter 
um maximo. Isso e intuitivamente claro. Ocorre que na definicao de maximo, 
EXIGE-SE que este maximo PERTENCA ao conjunto. Assim, um conjunto pode ser 
limitado superiormente e nao ter maximo, EM VIRTUDE DA DEFINICAO QUE 
COMUMENTE SE ADOTA.

Exemplo :

Seja Xn =  2^(-1)  +  2^(-2)  +  ...  +  2^(-n) , n =1, 2, ... . Se 
definirmos A={Xn, n=1,2,...}  Esse conjunto A, infinito enumeravel, e 
limitado superiormente, mas, claramente, nao tem maximo, pois pode ser visto 
como o conjunto dos termos de uma sequencia monotona crescente.

Assim, fala-se em MAXIMO  como algo distinto de LIMITE SUPERIOR, ambos 
distinto de SUPREMO ( E INFIMO ). De posse destres tres conceitos ( e de 
outros tambem ) prova-se diversos fatos evidentes, tais como Se um conjunto 
( de numeros reais ) e finito entao ele tem MAXIMO e MAX=SUP.

Um fato notavel e que a intuicao nos guia corretamente ate um certo ponto, 
alem do qual e necessario definir com rigor os conceitos, so pena de nos 
tornarmos demasiadamente vagos. Uma boa construcao conceitual ira confirmar 
e tornar prreciso muitos dos sentimentos que previamente temos diante dos 
fenomenos matematicos.

A percepcao do CONCEITO ADEGUADO, salvo melhor juizo, parece ser uma 
qualidade de todo Grande Matematico e Cientista... Se olharmos as grandes 
conquistas humanas no campo cientifico, em ultima analise o que e feito e 
uma modificacao nos conceitos vigentes, de forma que as conquistas ja feitas 
sao preservadas e novas possibilidades se tornam factiveis. Einstein se 
pronunciou muitas vezes sobre esse tema :

1) Nao existe nenhum caminho logico que leva a compreensao das leis 
elementares, o unico caminho e o da intuicao

2) Propor um problema e muito mais importante que resolve-lo. Resolver um 
problema e uma questao de tecnica e de habilidade, propor um problema e uma 
questao de intuicao.

As palavras talvez nao tenham sido estas, mas o sentido foi.

O Willes ( o nome e mesmo esse ?), aquele Matematico de Princeton que provou 
a conjectura do Shimura ( a toda equacao eliptica esta associada uma forma 
modular ) e, de tabela, provou o ultimo teorema de Fermat, fala sobre isso. 
Ele explicitamente diz que diante de um problema deste porte, as referencias 
a conhecimentos adquiridos ajudam muito pouco, tudo sucedendo como se voce 
estivesse num imenso quarto escuro e tivesse que encontrar a porta de saida. 
Voce nao sabe  : O que vai usar pra resolver, onde procurar o que usar e se 
pode ter esperanca de descobrir qualquer destas coisas !

Diante de um misterio assim, parece que so o sentimento vago, a intuicao, 
pode delinear alguma forma de tratar a questao.

Esse ato de estar DIANTE DO MISTERIO, que sabemos que realmente existe e que 
desafia a nossa percepcao, e emocionante e estimulante, e parece ser fonte 
nao so das mais belas aventuras cientificas como tambem da experiencia 
mistica em face de Deus e de sua presenca. O cara que realmente acredita na 
existencia de Deus deve sentir algo diferente em face da vida, pois nao pode 
se achar so e completo, em nenhum momento ...

Po, eu deixei a imaginacao me levar e terminei saindo do foco da questao. 
Mas acho que nao escrevi tantas besteiras.

Um Abracao pra voce
Paulo Santa Rita
2,1045,140703





From: Jose Francisco Guimaraes Costa [EMAIL PROTECTED]
Reply-To: [EMAIL PROTECTED]
To: [EMAIL PROTECTED]
Subject: [obm-l] Conjuntos - Justificativa
Date: Sun, 13 Jul 2003 22:03:37 -0300
Comentário de um não-matemático que às vezes confunde definições com 
postulados com teoremas, sobre a pergunta original do Leandro.

Ora, se o conjunto é limitado superiormente,  nenhum de seus elementos 
pode ser maior que o limite superior. Logo, ele certamente tem um máximo 
(que é menor ou igual ao limite), e isto seria um corolário.

Falei bobagem?

JF

- Original Message -
From: Eduardo Casagrande Stabel [EMAIL PROTECTED]
To: [EMAIL PROTECTED]
Sent: Sunday, July 13, 2003 9:20 PM
Subject: Re: [obm-l] Conjuntos - Justificativa
 Caro Leandro.

 Este é o chamado axioma do sup. É equivalente a muitos outros, e não
 costuma-se demonstrá-lo e sim usá-lo como axioma. Se você ainda quiser
 demonstrá-lo, terá de estabalecer todos os axiomas dos reais, isto é, os 
que
 você está usando (ou o livro). Do contrário, fica impossível ajudá-lo.

 Abração!
 Duda.

 From: Leandro Fernandes [EMAIL PROTECTED]
  Pessoal, não consigo dar uma justificativa plausível para esta 
afirmação:
 
  Todo conjunto não vazio de números racionais limitado superiormente 
tem
  máximo
 
  Alguém tem alguma sugestão?
 
  Leandro
_
MSN Messenger: converse com os seus amigos online.  
http://messenger.msn.com.br

[obm-l] Jornal Matematico

2003-07-14 Por tôpico Paulo Santa Rita
Ola Pessoal !

Existe um jornal eletronico dedicado a Matematica que nao trata 
especificamente de problemas ou artigos tecnicos, mas de temas ( 
educacionais, filosoficos, etc ) relacionados a este ciencia. Existem 
artigos muito interessantes. O endereco e :

http://www.geocities.com/jcvmatem/

Vale a pena dar uma olhada !

Um Abraco a Todos
Paulo Santa Rita
2,1153,140703
_
MSN Messenger: converse com os seus amigos online.  
http://messenger.msn.com.br

=
Instruções para entrar na lista, sair da lista e usar a lista em
http://www.mat.puc-rio.br/~nicolau/olimp/obm-l.html
=


[obm-l] O Truque do Nicolau

2003-07-14 Por tôpico Paulo Santa Rita
 a resolver tambem esse caso ?

O problema e uma modelagem valida para inumeras circunstancias. Por exemplo 
:

PROBLEMA 1 ) Seja A = { 1, 2, ..., N }. Quantas funcoes F:A-A existem que 
cumprem as duas seguintes condicoes :
a) F(X) nao tem ponto fixo
b) F(F(X)) nao tem ponto fixo

PROBLEMA 2 ) Uma matrix de 3 linhas e N colunas e dita ser latina se todos 
os seus elementos pertencem ao conjunto A = {1, 2, ..., N } e nenhuma fila ( 
linha ou coluna ) tem elementos repetidos. Quantas Matrizes latinas 3xN 
existem tais que
A1j = j,  j = {1, 2, ..., N }. ( Aij e o elemento da linha i e coluna j )

PROBLEMA 3 )  Participam de uma mesa redonda N pessoas, cada uma das quais 
tendo um lugar pre-determinado para sentar e uma pasta com o seu nome para 
receber. Eremildo, que e um idiota, deve distribui as pessoas nos lugares 
bem como  as pastas. Qual a probabilidade de que nenhuma pessoa sente no seu 
lugar correto e nao receba a pasta correta ?





Um Abraco a Todos !
Paulo Santa Rita
2,1608,140703
_
MSN Hotmail, o maior webmail do Brasil.  http://www.hotmail.com
=
Instruções para entrar na lista, sair da lista e usar a lista em
http://www.mat.puc-rio.br/~nicolau/olimp/obm-l.html
=


Re: [obm-l] Combinatoria (In off)

2003-07-15 Por tôpico Paulo Santa Rita
Ola Manuel e demais
colegas desta lista ... OBM-L,
1) A sua mensagem, nao obstante nao tratar de algum problema especifico, e 
muito boa. Ela realmente enriquece a Lista e e o tipo de OFF que espera-se 
que ocorra.

2) A Matematica continua e continuara sendo o tema desta Lista. Ainda que 
algumas pessoas mal educadas e mal formadas estejam - acredito firmemente - 
deliberadamente atentando contra estes objetivos originais, ela se mantera 
com a alta qualidade que nos, Professores, Pesquisadores e estudantes 
serios, almejamos e que lutamos para que tenha,

3) E verdade que o AXIOMA DO SUPREMO pode surgir como uma propriedade em 
alguma construcao particular dos numeros reais ou pode ser adotado como mais 
um axioma em outras construcoes. No livro de Analise I, Projeto Euclides. O 
Prof Elon adota este ultima postura, mas explicitamente cita outros obras 
nas quais os reais sao construidos e diz que uma tal construcao e um 
processo instrutivo. Em sintese, nao existe nenhuma razao mais forte para se 
adotar uma vertente ou outra, alem de crencas subjetivas e individuais.

4) A imensa maioria das teorias matematicas surgem de forma altamente 
intuitivas, pouco formais, somente rrecebendo um tratamento axiomatico 
posteriormente. Foi assim com o Calculo, com a Topologia, com a teoria dos 
grupos e com muitas ( talvez todas ! ) outras teorias. A intuicao vai na 
frente, descobre e orienta a pesquisa;  a formalizacao ou axiomatizacao vem 
depois e fundamenta com rigor as conquistas ja feitas. A primeira e a 
faculdade da descoberta, atributo do genio; a segunda, ferramente de prova e 
de resolucao de problemas, obra do talento.

Voce nao gostaria de apresentar aqui uma construcao dos reais, via cortes  
ou sequencias de Cauchy, por exemplo, e desta construcao derivar o TEOREMA 
DO SUPREMO ?

Um Abraco
Paulo Santa Rita
3,1154,150703

From: Manuel Valentim Pera [EMAIL PROTECTED]
Reply-To: [EMAIL PROTECTED]
To: [EMAIL PROTECTED]
Subject: Re: [obm-l] Combinatoria (In off)
Date: Mon, 14 Jul 2003 19:11:05 -0300 (EST)
Boa noite,

  Sobre o trecho:


 O segundo caso (mais geral) que você colocou, realmente merece uma
 demonstração, eu acho.
 Mas na minha cabeça, esse Princípio de Dirichlet seria uma coisa tão
 intuitiva que não precisaria de provas.
 Aí eu me embolo... Quando uma proposição precisa ser provada e quando se
 admite que ela é intuitiva o suficiente para ser aceita sem
demonstração?

  No sentido que as palavras tem em matematica (e matematica era, ate'
algum tempo atras, o assunto desta lista) sua duvida nao tem uma resposta
absoluta, exceto a trivial, DEPENDE do que foi admitido como axioma no
contexto de seu estudo, isto inclui, entre outras coisas mais mundanas,
quais os axiomas de teoria dos conjuntos que voce esta' admitindo. Tudo o
que nao for axioma precisa ser demonstrado.
  Eu nunca vi o principio de Dirichlet (ou pigeonhole) ser colocado como
axioma, entao precisa de uma demonstracao (se voce estiver admitindo os
postulados de Peanno para o conjunto dos naturais e ZF, isso sai
trivialmente, mas e' a demonstracao que e' trivial, nao a afirmacao. Alias
essa afirmacao tao trivial caracteriza, em muitos contextos, conjuntos
finitos), mas esta frase diz apenas isso: eu, na minha limitadissima
experiencia, nunca vi...
  Poucas afirmacoes sao tao evidentes (maldita palavra) como a do
Teorema da Curva de Jordan, se alguem conhecer alguma demonstracao
trivial dela, por favor, mostre-ma!
  Apenas um adendo, COM O AVISO DE IN-OFF EM MAIUSCULAS.

  Axioma, em matematica, nada tem a ver com intuitivo, ou evidente. os
axiomas das geometrias nao-euclideanas sao, do ponto de vista matematico
(outros pontos de vista deveriam ser assunto de bate papo em mesa de
botequim, coisas muito interessantes alias essas conversas, mas nao desta
lista), tao intuitivas quanto as euclideanas.
  Num exemplo concreto, falou-se nesta lista ha' nao muito tempo em
axioma do supremo para o conjunto dos Reais, isso ser um axioma so'
faz sentido numa teoria em que o conjunto dos numeros Reais (R) e'
apresentado axiomaticamente. Se voce quiser construir, por exemplo a
partir dos numeros racionais, esse conjunto isso deixa de ser axioma e
passa a ser uma PROPRIEDADE e precisa ser demonstrada.
  Hoje em dia pode parecer estranho falar-se em construcao de R, pois
o metodo axiomatico e' a unica forma que i conjunto dos Reais e'
apresentado (como dizia N. Rodrigues, toda unaminidade e' burra) e
construcoes de R sao temas desconhecidos dos dois primeiros anos de cursos
de graduacao em matematica, mas para pelo menos um grande Professor de
Matematica que eu conheco isso esta' longe de ser uma virtude do atual
modelo de ensino...
Desculpem o carater in-off do adendo, ele foge completamente dos objetivos
desta lista, mas em virtude de certas perolas recentes, que nem ao menos
vem com o aviso de in-off no subject penso que o professor Nicolau
perdoara' este deslize.
Manuel Garcia

Re: [obm-l] IMO - Problema 2

2003-07-18 Por tôpico Paulo Santa Rita
Oi Prof Gugu !
Tudo Legal ?
Conforme voce diz, usualmente. Mas tenho certeza que voce sabe que muitos 
bons livros tratam dos casos para j qualquer. Ai no IMPA, com certeza tem, 
porque eu ja vi. Vou divulgar aqui na lista ao menos uma livro elementar 
sobre equacoes diofantinas que trata deste casos.

Vou escrever a solucao e te envio.

Carissimo Prof, francamente acho que as solucoes da IMO nao deveriam ser 
enviadas para a lista pelos Mestres que orientam nossos atletas, pois e 
natural que os estudantes se sintam desestimulados em buscar uma solucao 
propria quando sabemos que a solucao ja esta divulgada. Por outro lado, para 
o Sr ou para os Profs Nicolau, Morgado, Wagner e os outros orientadores, 
resolver as questoes da IMO nao significa ou indica nada, enquanto que para 
um estudante, olimpico ou nao, uma solucao propria representa muito.

Essa e a minha impressao ! Mas, reconheco que o Prof e demais orientadores 
sao muito mais experientes, teem muito mais conhecimentos, razao pela qual 
modifico imediatamente esta minha ideia se o Prof achar que isso e bobeira. 
Eu falei com o Prof Nicolau neste sentido.

Mudando de assunto. O Prof pensou na quetao 3 ?

Ontem, almocando no restaurante, eu pensei nela ( e a comida ficou fria e eu 
nao almocei ! ), mas nao consegui resolver naquele tempo. Observei o 
seguinte :

IMAGINADO o hexagono ABCDEF no primeiro quadrante, IMAGINANDO os vetores A, 
B, C, D, E, F, e IMAGIANANDO o segmento MN que une os pontos medios de AB e 
DE ( M em AB e N em DE ), segue que :

M = 1/2 * (A+B) e N = 1/2 * (D+E)
MN = 1/2 * modulo( (A+B) - (D+E) )
A propriedade diz que :

MN = sqrt(3)/2 * (AB+DE)

ou seja :

1/2 * modulo( (A+B) - (D+E) ) = sqrt(3)/2  * ( modulo(B-A) + modulo(D-E) )
modulo( (A+B) - (D+E) ) = sqrt(3)*( modulo(B-A) + modulo(D-E) )
Isto pode ser colocado assim :
modulo( (A - E) + (B - D) ) = sqrt(3)/2  * ( modulo( B - A) + modulo( D - E) 
)
Evidentemente que para os demais pares de lados opostos valera uma relacao 
semelhante.

Os vetores A-E e B-D e os outros derivados das outras duas relacoes dos 
outros dois pares de lados opostos sao subtendidos pelos angulos internos do 
hexagono, segue que seus modulos podem ser expressos em funcao de alguma 
funcao trigonometrica aplicada a estes angulos internos.

E verdade que se todos os angulos internos valerem 120 graus entao os pares 
de lados opostos sao paralelos e a relacao vale. Suponha agora que um dos 
angulos nao e 120, entao, aplicando o principio da casa dos pombos aos 
demais, segue que havera algum outro tambem diferente de 120.

Eu parei aqui. Havendo tempo ( Ah se eu tivesse tempo so pra estudar ! ) vou 
prosseguir por este caminho.

Um Abracao pro Prof !
Paulo Santa Rita
6,1029,180703
From: [EMAIL PROTECTED]
Reply-To: [EMAIL PROTECTED]
To: [EMAIL PROTECTED]
Subject: Re: [obm-l] IMO - Problema 2
Date: Thu, 17 Jul 2003 18:41:48 -0300
Caro Paulo,
Usualmente o termo equacao de Pell se refere ao caso j=1 (e o 
coeficiente
de b^2 nao depende nem de a nem de b). Nao entendi como concluir uma 
solucao na
linha que voce propos. Por outro lado eu consegui (depois de tropecar um 
pouco)
achar uma solucao, que reproduzo abaixo, depois de algum espaco, para nao
atrapalhar quem queira pensar mais no problema.

...



...



...



...



...



...



Vamos la':

   Se b=1 o problema e' achar todos os a tais que a^2/2a=a/2 e' inteiro. 
Isso
nos da' as solucoes {(a,1),a par}. Vamos supor agora b=2.
   Se 2ab^2-b^3+1 divide a^2 entao tambem divide 
a^2.(2b^2)-a(2ab^2-b^3+1)=
=a(b^3-1) e (1-b^3)(2ab^2-b^3+1)+(2b^2)(a(b^3-1))=(1-b^3)^2. Sejam entao
d=mdc(a,1-b^3), a=kd, 1-b^3=ud. Temos que mdc(k,u)=1 e que 2ab^2-b^3+1 
divide
mdc(a^2,(1-b^3)^2)=d^2, ou seja, d(2kb^2+u) divide d^2, e logo 2kb^2+u 
divide
d. Portanto, tambem temos que 2kb^2+u divide b(2kb^2+u)+2kud=ub+2k. Temos 
agora
dois casos:
   i)d=kb^2. Entao |u|=|(1-b^3)/d|  b^3/(kb^2)=b/k. Nesse caso,
|2kb^2+u|=2kb^2-|u|  2kb^2-b/k, enquanto |ub+2k|  b^2/k+2k. Como b=2, 
2kb^2-
b/k=kb^2+b(kb-1/k)=b^2/k+2(2k-1/k)=b^2/k+2k, donde |2kb^2+u|  |ub+2k|, e
portanto devemos ter ub+2k=0, donde b(1-b^3)+2a=dub+2dk=0, e logo 
a=b(b^3-1)/2.
Isso nos da' a^2/(2ab^2-b^3+1)=b^2/4, que e' inteiro quando b e' par. Isso 
nos
da' (todas) as solucoes nesse caso i): {(b(b^3-1)/2,b), b par}.
   ii)d  kb^2. Aqui, como 2kb^2+u divide d, devemos ter 
kb^2|d|=|2kb^2+u|=
=2kb^2-|u|, donde |u|kb^2. Assim, temos ab^2=kb^2d  |ud|=b^3-1  b^3, 
donde
a  b, ou seja, b=a+1. Como 2ab^2-b^3+1 e' congruente a 1 modulo b^2, ou
2ab^2-b^3+1=1 ou |2ab^2-b^3+1|=|1-b^2|=b^2-1=(a+1)^2-1=a^2+2a  a^2, mas,
nesse caso, 2ab^2-b^3+1 nao pode dividir a^2. Assim, devemos ter 
2ab^2-b^3+1=1,
donde 2ab^2=b^3, e b=2a. Isso nos da' as solucoes do caso ii): {(a,2a)}.

   Conclusao: as solucoes do problema sao dadas por: {(a,1), a inteiro 
positivo
par}, {(b(b^3-1)/2,b), b inteiro positivo par} ou {(a,2a), a inteiro 
positivo}.
   Abracos,
Gugu

Quoting Paulo Santa Rita [EMAIL PROTECTED]:

 Ola Cicero

[obm-l] IMO - Curiosidades.

2003-07-19 Por tôpico Paulo Santa Rita
Ola Pessoal,

No endereco :

http://vyasa.math.iisc.ernet.in/PEOPLE/halloffame.html

Voces podem ver varios fatos curiosos relacionados a IMO. Por exemplo, la 
voces poderao ver os medalhistas imo que conseguiram tambem ter uma medalha 
fields ( O Yoccoz, amigo do Prof Gugu e um deles : IMO em 1974 e FIELDS em 
1994 ), estudantes que competiram durante 3 anos consecutivos e conseguiram 
3 medalhas de ouro, estudantes com 5 medalhas ( 3 ouros + duas outras entre 
bronze e prata ) e assim sucessivamente.

O que me pareceu o desempenho mais notavel foi o de  uma atleta mulher que 
saindo de um pais ainda altamente preconceituoso ( URSS ) contra as mulheres 
disputou 3 anos (1989 a 1991 ) consecutivos e conseguiu 3 medalhas de ouro : 
 Evgenija Malinnikova. Essa notavel estudante venceu muito mais que tres 
IMO's. Venceu a enorme pobreza de suas origens e o forte preconceito contra 
as mulheres que ainda existe em seu pais.

Muitos outros fatos existem. Vale a pena dar uma olhada !

Um Abraco a Todos !
Paulo Santa Rita
7,2127,190703
_
MSN Messenger: converse com os seus amigos online.  
http://messenger.msn.com.br

=
Instruções para entrar na lista, sair da lista e usar a lista em
http://www.mat.puc-rio.br/~nicolau/olimp/obm-l.html
=


[obm-l] Re: [obm-l] Demonstração não encontrada

2003-07-20 Por tôpico Paulo Santa Rita
 modulo(P(X0))=0 = 
P(X0)=0, ou seja :

TODA EQUACAO POLINOMIAL p(x)=0 TEM AO MENOS UMA RAIZ !

E isto e o chamado TEOREMA FUNDAMENTAL DA ALGEBRA  que voce estava querendo 
ver como se prova. Note que voce procurou em livros e nao encontrou uma 
prova, mas, escrevendo pra essa lista de discussao ( e eu senti sinceridade 
em sua busca ! ) voce teve uma resposta a altura !

Um Abraco e parabens pela sua busca pelo conhecimento !
Paulo Santa Rita
1,1438,200703
-Original Message-
From: [EMAIL PROTECTED] [mailto:owner-obm-
[EMAIL PROTECTED] On Behalf Of brunos.pompeo
Sent: Saturday, July 19, 2003 7:22 PM
To: [EMAIL PROTECTED]
Subject: [obm-l] Demonstração não encontrada
Gostaria q alguém me desse a demonstração do teorema
fundamental da álgebra, ou seja, todo polinômio tem raíz.
Por favor, identifique o e-mail.
Obrigado


Bruno Pompeo

_
MSN Hotmail, o maior webmail do Brasil.  http://www.hotmail.com
=
Instruções para entrar na lista, sair da lista e usar a lista em
http://www.mat.puc-rio.br/~nicolau/olimp/obm-l.html
=


Re: [obm-l] Uma dificil de torneira !!!!!!!

2003-07-20 Por tôpico Paulo Santa Rita
Ola Faelccmm e demais
colegas desta lista ... OBM-L,
Este enunciado esta meio estranho ... 1/4 de hora a menos DO QUE ? Da 
operacao descrita ? Da torneira A agindo sozinha ? O Prof Morgado esta 
coberto de razao : O onus da clareza do enunciado cabe ao enunciador !

Bom, vou tentar.

Seja T o tempo gasto na primeira operacao, vale dizer, o tempo transcorrido 
para a torneira A esvaziar 1/4 do reservatorio. Claramente que :

1) A torneira A, sozinha, gastara 4T para esvaziar o reservatorio.
2) O tempo da segunda operacao e T + 5/4 ( supondo T em horas )
Na segunda operacao, as duas juntas gastam T + 5/4 para esvaziar 3/4 do 
reservatorio, ou seja, o tempo para esvaziarem 1/4 do reservatorio e (T + 
5/4)/3 e, portanto, as duas juntas, esvaziarao o reservatorio todo em 4(T+ 
5/4)/3.

Esse tempo e 1/4 de hora a menos que o tempo gasto antes, isto e :

4(T+5/4)/3 = T + (T + 5/4) - 1/4
(4T + 5)/3 = 2T + 1 ... T = 1
A torneira A sozinha gastara 4T=4 horas

E ai, ficou legal ? Voce disputa as Olimpiadas da OBM - Nivel I, certo ?

Um Abraco
Paulo Santa Rita
1,1841,200307

From: [EMAIL PROTECTED]
Reply-To: [EMAIL PROTECTED]
To: [EMAIL PROTECTED]
Subject: [obm-l] Uma dificil de torneira !!!
Date: Sun, 20 Jul 2003 15:18:11 EDT
Ola pessoal,

Como resolver esta:

Um reservatorio, que se acha cheio da agua, tem duas torneiras de descarga 
A
e B, de capacidades diferentes:abre-se a torneira A e deixa-se correr a 
agua
ate escoar-se 1/4; abre-se entao a torneira B e deixa-se a agua correr 
pelas
duas ate esvaziar o reservatorio. O tempo gasto na segunda operacao excede 
o da
primeira em 5/4 de hora. Abrindo-se as duas torneiras desde o comeco, o
reservatorio seria esvaziado
em 1/4 de hora menos. Calcular o tempo em que a torneira A, sozinha,
esvaziarah o reservatorio.

_
MSN Hotmail, o maior webmail do Brasil.  http://www.hotmail.com
=
Instruções para entrar na lista, sair da lista e usar a lista em
http://www.mat.puc-rio.br/~nicolau/olimp/obm-l.html
=


[obm-l] RE: [obm-l] Demonstração não encontrada

2003-07-20 Por tôpico Paulo Santa Rita
Ola Artur e demais
colegas desta lista ... OBM-L,
Eu nao havia lido esta mensagem do colega Artur - que tem participado 
construtivamente de nossas discussoes - quando enviei para esta lista a 
Prova do Teorema Fundamental da Algebra dada por Cauchy, que, de fato, 
conforme todos podem verificar diretamente, e simples e curta, SE COMPARADA 
COM AS DEMONSTRACOES QUE SOMOS OBRIGADOS A ESTUDAR ATUALMENTE.

Estou dizendo isso porque a minha mensagem poderia passar a ideia de 
desconsideracao e desprezo para com o ponto de vista de um colega que so tem 
produzido mensagens que honram as nossas tradicoes e que nos induzem a ter 
por ele somente consideracao e respeito.

Existem muitas demonstracoes deste Teorema que sao longas e complexas e que 
nao seria factivel reproduzi-las aqui. E a estas provas que o estimado 
colega muito provavelmente deve estar se referindo na mensagem abaixo.  Eu 
poderia falar muito sobre esse tema, inclusive falar sobre suas implicacoes, 
como, por exemplo, o Teorema de Bolzano.

Fica portanto esclarecida qualquer ma interpretacao futura.

Um Abraco a Todos !
Um Abraco especial ao Artur !
Paulo Santa Rita
1,2203,200703
From: Artur Costa Steiner [EMAIL PROTECTED]
Reply-To: [EMAIL PROTECTED]
To: [EMAIL PROTECTED]
Subject: [obm-l] RE: [obm-l] Demonstração não encontrada
Date: Sat, 19 Jul 2003 23:34:35 -0300
De forma mais precisa, este teorema diz que todo polinomio nao
constante, de coeficientes complexos, tem pelo menos uma raiz complexa.
Uma outra forma de enunciar este teorema eh dizer que o corpo dos
complexos eh algebricamente fechado, pois dizemos que um corpo eh
algebricamente fechado se todo polinomio nao constante, de coeficientes
pertencentes ao corpo, apresentar ao menos uma raiz pertencente ao
corpo. A demonstracao do TFA eh extensa e nao dah para reproduzir nesta
lista. Eu conheco algumas apresentadas nos textos que citei em uma outra
mensagem sobre este mesmo assunto.
Um abraco
Artur
 -Original Message-
 From: [EMAIL PROTECTED] [mailto:owner-obm-
 [EMAIL PROTECTED] On Behalf Of brunos.pompeo
 Sent: Saturday, July 19, 2003 7:22 PM
 To: [EMAIL PROTECTED]
 Subject: [obm-l] Demonstração não encontrada

 Gostaria q alguém me desse a demonstração do teorema
 fundamental da álgebra, ou seja, todo polinômio tem raíz.
 Por favor, identifique o e-mail.
 Obrigado



 Bruno Pompeo




__
 Acabe com aquelas janelinhas que pulam na sua tela.
 AntiPop-up UOL - É grátis!
 http://antipopup.uol.com.br/




=
 Instruções para entrar na lista, sair da lista e usar a lista em
 http://www.mat.puc-rio.br/~nicolau/olimp/obm-l.html


=
=
Instruções para entrar na lista, sair da lista e usar a lista em
http://www.mat.puc-rio.br/~nicolau/olimp/obm-l.html
=
_
MSN Messenger: converse com os seus amigos online.  
http://messenger.msn.com.br

=
Instruções para entrar na lista, sair da lista e usar a lista em
http://www.mat.puc-rio.br/~nicolau/olimp/obm-l.html
=


[obm-l] TFA - Teorema Fundamental da Algebra

2003-07-21 Por tôpico Paulo Santa Rita
 dois lados :

modulo( P(Z0+Z1) / P(Z0) ) = modulo ( 1 - P(R^a) - P(R^a)*Z1*F(Z1) ) )

A desigualdade modulo(a-b) = modulo(a) + modulo(b) aplica-se tambem aos 
numeros
complexos. Aplicando-a :

modulo( P(Z0+Z1) / P(Z0) )  =  modulo ( 1 - P(R^a) ) + modulo( 
P(R^a)*Z1*F(Z1) )
modulo( P(Z0+Z1) / P(Z0) )  =  modulo ( 1 - P(R^a) ) + 
P(R^a)*modulo(Z1)*modulo(F(Z1) )

mas modulo(Z1) = R. Assim :

modulo( P(Z0+Z1) / P(Z0) )  =  modulo ( 1 - P(R^a) ) + 
P(R^a+1)*modulo(F(Z1) )

PRESTE BASTANTE ATENCAO AQUI. Nos tomamos um Z=Z0+Z1, isto e, escolhemos
um Z0 de forma que Q+aS = pi. Ora, Z e um ponto da circunferencia do circulo 
de centro
Z0 e raio R. Portanto, se diminuirmos R e mantivermos a direcao de Z 
estaremos, de fato, como que fazendo um corte no circulo original de raio R 
... Isso ( a dimuicao de R ) vai diminuir
P(R^a) e P(R^a+1)*modulo(F(Z1) ), pois a e um numero natural fixo e R esta 
diminuindo. Claramente que INEVITAVELMENTE P(R^a) se tornara maior que 
P(R^a+1)*modulo(F(Z1) ) para algum R suficientemente pequeno. Quando isto 
ocorrer :

modulo ( 1 - P(R^a) ) + P(R^a+1)*modulo(F(Z1) )  1. Seguira que :

modulo( P(Z0+Z1) / P(Z0) )1= modulo( P(Z0+Z1) )   modulo( 
P(Z0))  ... ABSURDO !
Pois modulo(P(Z0)) e minimo !

Esse absurdo derivou do fato de postularmos que modulo(P(Z0))  0. Assim, 
esta tese e
insustentavel e temos que admitir que :

modulo(P(Z0)) = 0   =  P(Z0) = 0, isto e :

TEOREMA FUNDAMENTAL DA ALGEBRA : Toda equacao polinomial de qualquer grau N 
e com quaisquer coeficientes complexos tem uma raiz.

A PRIMEIRA IDEIA DE GAUSS : Em sua primeira demonstracao ( tese de doutorado 
) Gauss substitui cada numero complexo pelo binomio a+bi e divide o 
polinomio em duas funcoes de duas variaveis : Real(a,b) e Complexo(a,b). A 
seguir, tecendo consideracoes geometricas ele mostra que o sistema :

Real(a,b) = 0 e Complexo(a,b) = 0

Necessariamente tem uma solucao. Ele nao ficou satisfeito e tentou ffazer 
uma prova estritamente algebrica, mas nao conseguiu. Segundo Jean Dioudonne, 
Matematico frances do Grupo Boubarki, o TFA depende necessariamente de 
consideracoes topologicas e, portanto, a pretensao de Gauss era infundada.

O TFA tem muitas implicacoes. Uma, conhecida como TEOREMA DE BOLZANO, e 
muito bonita. Alguem gostaria de mostrar esta implicacao ?

Um Abraco a Todos
Paulo Santa Rita
2,1448,210703
_
MSN Messenger: converse com os seus amigos online.  
http://messenger.msn.com.br

=
Instruções para entrar na lista, sair da lista e usar a lista em
http://www.mat.puc-rio.br/~nicolau/olimp/obm-l.html
=


[obm-l] Um Problema Interessante ...

2003-07-21 Por tôpico Paulo Santa Rita
Ola Pessoal !

Em muitas Linguagens de Programacao de Computadores e possivel criarmos 
funcoes recurssivas, vale dizer, e possivel criarmos funcoes que chamam a si 
mesmas um numero arbitrario de vezes. A recurssividade pode ser de mais de 
um tipo e, em geral, usa intensamente o recurso de variaveis locais para 
guardar o status das inumeras iteracoes.

Matematicamente falando, a recurssividade pode ser modelada pelo processo de 
composicao de uma funcao com ela mesma. Se Y1=F(X), entao Y1=F(F(X)) seria 
uma rrecurssao.

Em programacao, em geral, nos nao estamos preocupados com a recurssao em si. 
mas sim na potencialidade de tal possibilidade representa, pois muitos 
algoritmicos se tornam de solucao facil e elegante se o implementamos por 
recurssao. Mas e claro que toda solucao recurssiva exige um conhecimento 
interno da funcao.

Por muitas razoes, eu estou precisando resolver o seguinte problema :

Seja Y=H(X) uma funcao continua, conhecida, e A, B e C inteiros e N um 
natural
maior que 2. Que condicoes Y=H(X) deve atender para que exista F(X) tal que 
:

A*F^N(X)  +  B*F^(N-1)(X) + C*F(X) = H(X)

Onde F^N(X) e a composicao de F(X) consigo mesma N vezes, isto e :

F^N(X) = FoFoFoFo...oF(X)N vezes

Pode ser que eu esteja querendo resolver um problema que ja foi resolvido. 
Neste caso, alguem sabe onde posso ver a solucao ?

Um Abraco a Todos
Paulo Santa Rita
2,1609,210703
_
MSN Hotmail, o maior webmail do Brasil.  http://www.hotmail.com
=
Instruções para entrar na lista, sair da lista e usar a lista em
http://www.mat.puc-rio.br/~nicolau/olimp/obm-l.html
=


Re: [obm-l] TFA - Teorema Fundamental da Algebra

2003-07-22 Por tôpico Paulo Santa Rita
Ola Frederico e demais
colegas desta lista ... OBM-L,
O Teorema de Bolzano a que me referi e o seguinte :

TEOREMA DE BOLZANO : Se Y=P(X) e um polinomio real de coeficientes reais 
definido no intervalo
]a,b[ entao :

1) Se o sinal de P(a) e igual ao sinal de P(b) ha um numero par de raizes no 
intervalo ]a,b[
( podendo ser 0 raizes )
2) Se o sinal de P(a) e diferente do sinal de P(b) entao ha um numero impar 
de raizes

Existem muitas coisas interessantes para serem vistas aqui, muitas das quais 
dependentes de algum conhecimento mais aprofundado. Por exemplo. A equacao :

Z^7 + 5bar(Z)^4 + Z = 0

tem 17 solucoes... E verdade ! 17 solucoes ! E a abordagem disso segue as 
pegadas do Gauss, ao abordar os sistemas R(a,b) e C(a,b) aos me referi. Mas 
e dificil falar sobre isso sem pressupor algum conhecimento alem do nivel da 
graduacao.

Nao sei ate onde voce estudou, mas talvez voce consiga ler o trabalho :

http://www.arxiv.org/abs/math.na/0209097

Muito provavelmente, dentre todos os Matematicos do Mundo, o Prof Nicolau 
Saldanha, nosso moderador, e o cara que melhor entende destas coisas e de 
suas implicacoes. Para nos Brasileiros e, em particular, nos aqui desta 
lista, isso e motivo de muito orgulho.

Um Abraco
Paulo Santa Rita
3,1129,220703
From: Frederico Reis Marques de Brito [EMAIL PROTECTED]
Reply-To: [EMAIL PROTECTED]
To: [EMAIL PROTECTED]
Subject: Re: [obm-l] TFA - Teorema Fundamental da Algebra
Date: Tue, 22 Jul 2003 10:14:37 -0300
Olá Paulo, bom ter reenviado a prova de Cauchy. Acaso o Teorema de Bolzano 
a que se refere é o tb conhecido como Teorema do Valor Intermediário ( ou 
em realidade algo equivalente a ele ) ? Se não, qual o enunciado?

Obrigado,
FRederico.

From: Paulo Santa Rita [EMAIL PROTECTED]
Reply-To: [EMAIL PROTECTED]
To: [EMAIL PROTECTED]
Subject: [obm-l] TFA - Teorema Fundamental da Algebra
Date: Mon, 21 Jul 2003 18:15:03 +
Ola Pessoal,

Revendo a mensagem na qual aprresento a PROVA DE CAUCHY  para o Teorema
Fundamental da Algebra achei-a um tanto confusa, pois eu estava escrevendo 
com
pressa. Como este Teorema e importante, dificilmente encontrado em livros 
do
ensino medio e sendo a prova de Cauchy simples, facilmente acompanhavel 
por
um estudante dedicado, resolvi re-escrever a prova, colocando detalhes de 
forma que
qualquer pessoa possa entender.

Esse Teorema tem provas mais longas e mais curvas. Usando Analise complexa 
a
prova e trivial e curtissima, mas nao acho que seja adeguado apresentar 
aqui, por
obvias razoes.

A IDEIA FUNDAMENTAL : A ideia subjacente a esta prova e a seguinte. Se 
everdade
que todo polinomio no plano de Argand tem raiz, entao esta raiz minimiza o 
seu modulo
e a suposicao de um minimo positivo deve conduzir a um absurdo. Como fazer 
este
absurdo surgir ? Considerando um circulo em torno do ponto que minimiza o 
modulo do
polinomio e tratando retas passando por este ponto. Em uma destas retas 
evidenciara
o absurdo. O resto e detalhe.

Segue a Prova de Cauchy :

Seja  P(X) = A0*(X^n) + A1*(X^n-1) + ... + An-1*X + An  um polinomio no 
qual os
coeficientes A0, A1, ..., An-1, An sao numeros complexos quaisquer e X e 
uma
variavel complexa. Queremos mostrar que existe Z complexo tal que :

P(Z) = A0*(Z^n) + A1*(Z^n-1) + ... + An-1*Z + An = 0.

Para tanto, seja M = MIN { MODULO( P(X) ), X variando em C }.

Como, por definicao, modulo( P(X) ) = 0. Segue que M = 0. Portanto, M 
pode
ser

PRIMEIRO CASO : M = 0.

Neste caso, existe um complexo Z0 tal que MODULO( P(Z0) ) = 0. Segue que
P(Z0) = 0 e portanto Z0 e uma raiz de P(X) e a demonstracao esta 
concluida.

SEGUNDO CASO : M  0.

Neste caso, seja Z0 o complexo tal que MODULO( P(Z0) ) = M. IMAGINANDO no
plano complexo um circulo de centro Z0 e raio R, segue que qualquer ponto 
Z na
circunferencia deste circulo pode ser imaginado como a extremidade de um 
vetor,
soma dos vetores :

Z0 : origem em (0,0) e extremidade no ponto Z0
Z1 : origem no ponto Z0, extremidade no ponto Z e modulo R
Assim, para qualquer Z na circunferencia do circulo, teremos :

Z = Z0 + Z1

Calculando agora P(Z), teremos :
P(Z)=P(Z0 + Z1)=A0*((Z0 + Z1)^n ) + A1*((Z0+Z1)^n-1 ) + ... + An-1*(Z0+Z1) 
+ An
Na expressao acima, ao expandirmos (Z0+Z)^p - p = 0,1,2, ..., n - usando o 
Binomio
de Newton, iremos obter as parcelas A0*(Z0^n), A1*(Z0^n-1), ..., An-1*Z0, 
An nas
quais nao aparece Z1 e diversas outras parcelas, nas quais sempre constara 
Z1 :

1) Sozinho, sem que apareca Z0.  Exemplos :
A0*(Z1^n), A1*(Z1^n-1), ..., An-1*Z1
2) Acompanhado de Z1. Exemplos :
BINOM(N,1)*A0*(Z0^n-1)*(Z1), BINOM(N,N-1)*A0*(Z0)*(Z1^n-1), ...
onde BINOM(N,P) = N! / ( P!*(N-P)! )
Esta observacao deixa claro que P(Z0+Z1) tera o seguinte aspecto :
P(Z0+Z1) = P(Z0) + B0*(Z1^n) + B1*(Z1^n-1) + ... + Bn*Z1
onde cada Bi e uma constante ou um  polinomio em Z0.
Claramente que dependendo dos Ai originais, de n e do valor de Z0, 
alguns destes
Bi poderao ser nulos. Se, alem de eliminar os Bi nulos, ordenarmos o 
polinomio em Z1
resultante segundo

[obm-l] A Equacao de Pell e o Gugu

2003-07-22 Por tôpico Paulo Santa Rita
Ola Pessoal,

_
MSN Messenger: converse com os seus amigos online.  
http://messenger.msn.com.br

=
Instruções para entrar na lista, sair da lista e usar a lista em
http://www.mat.puc-rio.br/~nicolau/olimp/obm-l.html
=


[obm-l] A MAE da Teoria

2003-07-23 Por tôpico Paulo Santa Rita
Ola Pessoal,

Toda teoria tem um Pai. O Pai da teoria dos Grupos todos sabem quem e : 
Galois. Mas o muitos nao sabem e que a teoria dos grupos tem uma MAE tambem 
... A Mae da teoria dos grupos e o Lagrange.

Este notavel Matematico, trabalhando no que ele chamou de Um artigo 
despretencioso, generalizou todos os raciocinios fundamentais que ate 
aquela epoca existiam sobre a resolucaos de equacoes algebricas, mostrando 
como empregar uma mesma e unica linha de raciocinio para resolver todas as 
equacoes. A seguir, ele mostrou que esta linha de raciocinio nao podia se 
empregada para as equacoes do 5 grau.

Esse trabalho do Lagrange, LIDO POR GALOIS, gerou a Teoria dos Grupos. 
Inclusive quem le a memoria original do lagrange comeca a advinhar o sentido 
de muitos conceitos e raciocinios abstratos ensinados em teoria dos grupos. 
E uma excelente atividade, que eu estimulo.

Tudo que eu falei nao e raro acontecer ... Em muitos outros momentos um 
Matematico de gabarito aplica um raciocinio engenhoso na resolucao de um 
problema e um outro Matematico, bem preparado, ENCHERGA no raciocinio do 
colega possibilidades que o autor original nao enchergou ...

Vamos portanto dizer que o autor em potencial das ideias e a MAE da 
teoria. O Pai e o cara que viu as coisas, que desenvolveu e aplicou.

Neste lista, alguguem publicou uma ideia. Nesta lista existe uma Mae ...

Um Abraco a Todos
Paulo Santa Rita
4,1432,230703
_
MSN Hotmail, o maior webmail do Brasil.  http://www.hotmail.com
=
Instruções para entrar na lista, sair da lista e usar a lista em
http://www.mat.puc-rio.br/~nicolau/olimp/obm-l.html
=


Re: [obm-l] Log-Integral e Numeros Primos

2003-07-25 Por tôpico Paulo Santa Rita
Ola Claudio e demais colegas
desta lista ... OBM-L,
Eu tambem achei a sua pergunta interessante, mas nao vi como responde-la 
rapidamente. Agora sei que uma resposta rapida seria muito dificil ...

Aqui na lista existem diversas mensagem sobre a HR, algumas muito 
interessantes. Em particular, uma, que achei genial, mostra um tratamento 
probabilistico da HR. O autor demonstra que a probabilidade da HR esta certa 
e 1 !! Esta mensagem foi fruto de uma adaptacao que eu fiz de um artigo 
publicado no Scientif American, mas nao me lembro do numero.

Mas ela esta nos arquivos desta nossa lista, sem duvida.

Um Abraco
Paulo Santa Rita
6,1513,250703


From: Claudio Buffara [EMAIL PROTECTED]
Reply-To: [EMAIL PROTECTED]
To: [EMAIL PROTECTED]
Subject: Re: [obm-l] Log-Integral e Numeros Primos
Date: Fri, 25 Jul 2003 14:26:34 -0300
Oi, pessoal:

Lendo mais sobre a Hipotese de Riemann, eu descobri que ela é equivalente à
afirmativa:
Pi(n) = Li(n) + O(raiz(n)*ln(n)).
(um dos meus objetivos é entender o que isso tem a ver com os zeros da
função zeta - aliás, uma curiosidade: o matemático que demonstrou esta
relação for o Helmut von Koch - o mesmo da curva de Koch (floco de neve
fractal))
Assim, se X(n) abaixo for limitada, então a HR (de fato, um resultado muito
mais forte) estará demonstrada.
Logo, é certo que ninguém (nem na lista nem fora dela) sabe a resposta da
pergunta abaixo.
A primeira vista, pode parecer estranho que mesmo que lim Pi(n)/Li(n) = 1 e
que X(n) mude de sinal infinitas vezes, X(n) possa ser ilimitada, mas o
seguinte exemplo ilustra este fenomeno:
Considere:
Y(n) = n^2 + n*sen(n)
Z(n) = n^2
Temos lim Y(n)/Z(n) = 1 e Y(n) - Z(n) = n*sen(n) muda de sinal infinitas
vezes.
No entanto, Y(n) - Z(n) é claramente ilimitada.
Um abraço,
Claudio.
- Original Message -
From: Claudio Buffara [EMAIL PROTECTED]
To: Lista OBM [EMAIL PROTECTED]
Sent: Thursday, July 24, 2003 9:56 PM
Subject: [obm-l] Log-Integral e Numeros Primos
 Caros colegas da lista:

 Alguem saberia dizer se a sequencia:
 X(n) = Pi(n) - Li(n) eh limitada e se sao conhecidos o lim inf e lim 
sup?

 onde:
 Pi(n) = no. de primos = n;
 Li(n) = log-integral de n = Integral(2 a n) dx/ln(x)

 OBS: Sabe-se que lim Pi(n)/Li(n) = 1 e que X(n) muda de sinal infinitas
 vezes.

 Um abraco,
 Claudio.

 
=
 Instruções para entrar na lista, sair da lista e usar a lista em
 http://www.mat.puc-rio.br/~nicolau/olimp/obm-l.html
 
=

=
Instruções para entrar na lista, sair da lista e usar a lista em
http://www.mat.puc-rio.br/~nicolau/olimp/obm-l.html
=
_
MSN Messenger: converse com os seus amigos online.  
http://messenger.msn.com.br

=
Instruções para entrar na lista, sair da lista e usar a lista em
http://www.mat.puc-rio.br/~nicolau/olimp/obm-l.html
=


Re: [obm-l] News from IMC!!!!!!!

2003-07-27 Por tôpico Paulo Santa Rita
Ola Prof Okakamo e demais
colegas desta lista ... OBM-L,
Protesto !

Eu, em OFF, estava dizendo ao Gugu que aguardassemos mais um tempo antes de 
enviar solucoes das questoes IMO, para que os demais participantes, 
sobretudo os que se preparam para as olimpiadas, tivessem oportunidade de 
enviar as suas solucoes ou parte delas. Eu francamente acreditava e continuo 
acreditando, que muitos membros serios, estudantes, tem totais condicoes de 
resolver qualquer das questoes, alem da 4 ( que e trivial ).

Como nao conhecia o ridiculo plagio do Dirichlet, disse : Olha ai Gugu, viu 
a solucao do Dirichlet ? Isso e uma prova de que devemos aguardar um pouco 
mais !

Agora o Prof Okakamo esclarece tudo ... Que feio ! O Plagiario e o ladrao da 
inteligencia ! Isso e um crime que consta na lei do direito autoral e cabe 
denuncia ao Ministerio Publico Federal : Eu sou do Ministerio Publico 
Federal e uma das minhas funcoes e justamente combater crimes ciberneticos ! 
Nao e muito mais facil mergulhar dentro de si mesmo e escutar o nosso Prof 
interno ? A solucao vem normalmente !

Em sintese, se o Prof Okakamo nao esta enganado, eu me associo a sua 
indignacao e repudio atitudes tao baixas quanto esta !

Um Abraco a Todos !
Paulo Santa Rita
1,1737,270703



From: okakamo kokobongo [EMAIL PROTECTED]
Reply-To: [EMAIL PROTECTED]
To: [EMAIL PROTECTED]
Subject: [obm-l] News from IMC!!!
Date: Sun, 27 Jul 2003 13:56:23 -0300 (ART)
  Oi pessoal pessoal da lista,

 Finalmente consegui internet aqui na Romenia (estou
com os alunos da IMC,
junto com o Luciano), tirando duvidas das solucoes dos
meus problemas
propostos (eu e o Luciano estamos tentando explicar a
ideia de alguns
problemas para o pessoal da banca).
 Gostaria de fazer um pequeno protesto:
O participante da lista Dirichlet estah plagiando
ideias de meu caro
e estimado pupilo Eduardo Tengan, que resolveu o
problema 6 da IMO (de uma
forma elegante) e o participante simplesmente copiou e
colou sem a minima
vergonha. Considero essa atitude desprezivel,
ridicula. E ja nao eh a
primeira vez que isso acontece. Ficar se gabando de
que uma coisa que
nao se fez eh simplesmente estupido. Seja honesto
(inclusive no nome)
O

_
MSN Messenger: converse com os seus amigos online.  
http://messenger.msn.com.br

=
Instruções para entrar na lista, sair da lista e usar a lista em
http://www.mat.puc-rio.br/~nicolau/olimp/obm-l.html
=


[obm-l] Plagio e Direito Autoral

2003-07-27 Por tôpico Paulo Santa Rita
Ola Pessoal !

Eu peco permissao a todos e, em particular, ao nosso moderador, pra indicar 
um Site que nao trata de Matematica, mas que esta diretamente ligada ao 
infeliz fato que recentemente presenciamos aqui.

http://www.persocom.com.br/brasilia/plagio1.htm

A lei pune para evitar danos a outrem ou a sociedade, mas a maior punicao 
para quem nao se corrige e o desprezo das pessoas de bem.

Um Abraco a Todos
Paulo Santa Rita
1,1923,270703
_
MSN Hotmail, o maior webmail do Brasil.  http://www.hotmail.com
=
Instruções para entrar na lista, sair da lista e usar a lista em
http://www.mat.puc-rio.br/~nicolau/olimp/obm-l.html
=


Re: [obm-l] Problema de matrizes

2003-07-28 Por tôpico Paulo Santa Rita
Ola Prof Morgado, Daibert e
demais colegas desta lista ... OBM-L,
Vou tentaracrescentar mais detalhes a resposta do Prof Morgado. Conforme o 
Prof assinalou, o erro na sua demonstracao esta na passagem :

fazendo para este novo sistema x1, x2, x3, ..., x(n - 1) = 0

0 + 0 + 0 + ... + 0 + kx(n) = 0
0 + 0 + 0 + ... + 0 + lx(n) = 0
.
0 + 0 + 0 + ... + 0 +  x(n) = 0
(Eu posso realmente fazer isso?? Se não posso, pq não posso?)NAO PODE!
Ao acrescentar a nova linha e a nova coluna, PRESSUPONDO QUE O DETERMINANTE 
DA MATRIZ A+I DE ORDEM N-1 E DIFERENTE DE ZERO, tudo que voce pode 
concluir e que A CARACTERISTICA DA NOVA MATRIZ A+I e pelo menos N-1, isto 
e, que se o determinante da nova matriz A+I for igual a zero entao, 
necessariamente, com base no teorema de Rouche-Capelli, ao atribuir um valor 
arbitrario ( digamos : ALFA ) a nova varialvel Xn e transformando a coluna N 
nos termos independentes, teremos um sistema de N-1 incognitas e N equacoes, 
possivel e determinado.

E interessante perceber que se A e anti-simetrica de ordem maior que 2, 
entao, em A+I, se suprirmos a primeira linha e a primeira coluna, a matriz 
resultante e ainda da forma A+I, com A anti-simetrica; igualmente, se 
suprirmos a ultima linha e a ultima coluna, a matriz resultante e da forma 
A+I, com A anti-simetrica. O que estou tentanto lhe dizer e que o raciocinio 
do paragrafo anterior podera ser aplicado duas vezes ...

Existe um teorema ( de Jacobi ou Cauchy, nao me lembro ao certo ) que os 
livros de ensino medio abordam, que e o seguinte :

TEOREMA : Se acrescentarmos a uma fila de uma matriz quadrada uma combinacao 
linear das demais filas paralelas, o determinante desta matriz nao se 
altera

COROLARIO : Se uma fila de uma matriz quadrada e uma combinacao linear das 
demais filas paralelas entao o determinante desta matriz e igual a zero

OBS : Estou usando fila como sinonimo de linha ou de coluna.

Um Abraco
Paulo Santa Rita
2,1110,280703
From: A. C. Morgado [EMAIL PROTECTED]
Reply-To: [EMAIL PROTECTED]
To: [EMAIL PROTECTED]
Subject: Re: [obm-l] Problema de matrizes
Date: Mon, 28 Jul 2003 08:46:15 -0300
ASSINALEI O ERRO.
Veja: o sistema  x+y=1, x-y=1 tem soluçao (1,0). O sistema  x+y +z =1, 
x-y+z=1, x+2y +3z=3 tem soluçao (0,0,1). O seu processo levaria a conclusao 
que este sistema eh impossivel.

Alexandre Daibert wrote:

Olha, eu fiz uma demonstração mas acho q está errada, gostaria que alguém 
achasse o erro na minha demonstração para mim.

A resolução usa a idéia da resolução da questão do IME q eu tinha enviado 
aos senhores por meio de sistemas lineares homogêneos. (dúvidas olhe no 
fim deste e-mail q também está postado)
resumindo a idéia principal da questão anterior:
no sistema linear homogêneo (onde X eh matriz-coluna das incógnitas)
(A + I)X=(0)   , X = (0)  implica q A é inversível (está provado na 
questão anterior)
provemos por indução finita q X=(0) para todo A anti-simétrico:
X=(0) denota a matriz coluna de ordem qualquer com todos os elementos 
iguais a zero

provando para matriz 1x1:
A (1x1) = matriz unidade [0]
X = matriz unidade [x]
AX = -X
[0]*[x] = -[x]
[0] = -[x]
x = 0   implica X = (0), logo a propriedade eh verdadeira para n=1
provamos q se é valida para matriz (n-1)x(n-1) é válida também para matriz 
nxn
o sistema linear homogêneo determinado para ordem (n-1) fica da seguinte 
forma (valendo-se da igualdade (A + I)X = (0)) :
x1   +  ax2  +  bx3  +  ...  =  0
-ax1 +   x2  +  dx3  +  ...  =  0
-bx1 + -dx2  +   x3  +  ...  =  0
..
-gx1 + -hx2  + -ix3  +  ...  =  0

por hipótese x1, x2, x3, ..., x(n - 1) = 0 , pois X=(0)

para A nxn temos:

x1   +  ax2  +  bx3  +  ...  + kx(n) =  0
-ax1 +   x2  +  dx3  +  ...  + lx(n) =  0
-bx1 + -dx2  +   x3  +  ...  + mx(n) =  0
...
-gx1 + -hx2  + -ix3  +  ...  + zx(n) =  0
-kx1 + -lx2  + -mx3  +  ...  +  x(n) =  0
fazendo para este novo sistema x1, x2, x3, ..., x(n - 1) = 0

0 + 0 + 0 + ... + 0 + kx(n) = 0
0 + 0 + 0 + ... + 0 + lx(n) = 0
.
0 + 0 + 0 + ... + 0 +  x(n) = 0
(Eu posso realmente fazer isso?? Se não posso, pq não posso?)NAO PODE!

da última equação, constatamos q x(n)=0
x(n)=0  =  X=(0)  =  det (A + I) diferente de zero  =  (A + I) é 
inversível para todo n

segundo o que acabamos de constatar, a propriedade seria válida não soh 
para matrizes antisimétricas, mas para toda matriz com a diagonal 
principal com todos os elementos iguais a zero, o que é estranho, pois não 
é válida para a seguinte matriz A:
|| 0 1 ||
|| 1 0 ||
cujo det (A + I) = 0

 Aguardo ansiosamente respostas

Alexandre Daibert







Alexandre Daibert escreveu:

Hehehe, vou ser sincero, naum entendi tudo, mas deu pra entender bastante 
coisa sim, vou dar mais uma relida pra ver se entendo tudo, hehehehe. 
Valeu aí!

Quero só deixar apara o pessoal da lista a resolução q eu tinha comentado 
por sistemas lineares homogêneos, q eu lembrei aki:

sendo BX=(0

Re: [obm-l] News from IMC!!!!!!! / Dirichlet

2003-07-28 Por tôpico Paulo Santa Rita
Ola Prof Gugu e demais
colegas desta lista ... OBM-L,
A mensagem a que o Prof Okakamo se refere e :

 INICIO DA MENSAGEM DO DIRICHLET

LEGALLEGALLEGAL!Consegui fechar o problema 3 da 
IMO!!Confiram ai pra ver se nao tem nenhum erro.

Agradeço ao Paulo Santa Rita pelas suas dicas de vetores e ao Marcio Afonso 
Assad Cohen pela ideia de usar desigualdades(e ao problema cinco da IMO da 
India pela fabulosa ideia) e ao Tengan por ter me ensinado a aplicar a 
tecnica  leve tudo ate as ultimas consequencias!!!.

O nosso hexagono tera o nome bastante original de ABCDEF cujos pontos medios 
sao o hexagono nao menos original MNPQRS,com M em AB,N em BC e assim por 
diante.
O enunciado afirma que MQ=(raiz(2) 3)/2 *(AB+DE)= e assim por diante.Devemos 
mostrar que o hexagono e equiangulo.
Seja t=sen 30°=COS 60º=(raiz(2) 3)/2.
Começaremos demonstrando que os lados opostos são paralelos.

Considere a origem em qualquer lugar.O ponto medio de AB e (A+B)/2 entao o 
vetor que liga os pontos medios de AB e DE e (A+B-D-E)/2.Assim 
|A+B-D-E|=2t*(|A-B|+|D-E|).Para melhorar a joça vamos tentar usar a 
desigualdade triangular para vetores.Sabemos que |A-B|+|D-E|=|A-B-D+E| com 
igualdade se e somente se AB//DE(entendeu o paralelismo ai?).Assim 
|AD+BE|=2t|AD-BE|.
Quadrando e usando produto interno euclidiano:
AD,AD+2DA,EB+EB,EB =
3*(AD,AD-2DA,EB+EB,EB) ou equivalentemente AD,AD+EB,EB 
=4*AD,EB.Analogamente
EB,EB²+|FC,FC|²=4*EB,FC
FC,FC²+AD,AD²=-4*FC,DA.
Somando,obtemos 2*(DA-EB+FC)²=0.Logo
DA-EB+FC=0,e os lados opostos são paralelos(verifique as igualdades!).

0=A-D-B+E+C-F=BA+DC+FE,logo os vetores AB,CD e EF podem fechar um 
triangulo.Isto ja e mais um ponto ao nosso favor!
Se transladarmos EF na direção ED,obtemos CG no prolongamento do lado 
BC,devido ao paralelismo.Em miudos, desenhamos os paralelogramos 
ADCH,ABEI,CFEG.Devido aos paralelismos produzimos um triângulo GHI com o 
nosso ABCDEF dentro.Basta demonstrar que o triangulo GHI e equilatero.Para 
tal:

DEVER DE CASA:cada mediana de um certo triangulo mede
sen 60°  vezes o lado correspondente.Calcule os angulos do 
triangulo.(DICA:umas SLCs podem ajudar,ou use Stewart)

A PARTIR DAQUI FAÇA UM BOM DESENHO!

Os triangulos HAB,FIE,CDG e HIG são semelhantes por paralelismo.E pelo mesmo 
motivo os tres primeiros triangulos sao congruentes e homoteticos ao 
quarto(se lembra dos vetores?).Assim IE=DG e EQ=DQ,logo IQ=QG e assim sendo 
Q e ponto medio de GI.Pela homotetia e visivel que H,M,Q  se alinham.
Vamos calcular a razao HQ/IG:
HQ/IG=HM/AB=(HM-HQ)/(AB-GI)=MQ/(AB+DE)=cos 60°,e de acordo com o dever de 
casa(voce fez?),COMEMORE!!

Sera que ressa vai pra Eureka! ?Vou enviar JA!!!

UM ABRAÇAO!Ass.:Johann

** FIM DA MENSAGEM DO DIRICHLET



From: Carlos Gustavo Tamm de Araujo Moreira [EMAIL PROTECTED]
Reply-To: [EMAIL PROTECTED]
To: [EMAIL PROTECTED]
Subject: Re: [obm-l] News from IMC!!! / Dirichlet
Date: Mon, 28 Jul 2003 10:54:21 -0300 (EST)
  Caros Paulo e Okakamo,
   Nao sei a qual mensagem do Dirichlet voces se referem, mas a que eu vi 
(e
da qual reproduzo abaixo uma parte) sobre o problema 6 menciona
explicitamente que e' do Tengan a solucao que ele copia. Se for assim acho
que voces estao exagerando um pouco...
   Abracos,
Gugu

Date: Mon, 21 Jul 2003 15:43:53 -0300 (ART)
From: [** iso-8859-1 charset **] Johann Peter Gustav Lejeune
Dirichlet[EMAIL PROTECTED]
Subject: Re: [obm-l] E-mail do Tengan sobre o IMO 6
To: [EMAIL PROTECTED]
---Executing: shownonascii
This message contains non-ASCII text, but the iso-8859-1 font
has apparently not yet been installed on this machine.
(There is no directory named /usr/local/lib/fonts.)
What follows may be partially unreadable, but the English (ASCII) parts
should still be readable.
Legal,esta ideia e parecida com a minha.Mas uma
coisa:alguem pode ser mais explicito nesta parte
de olhar a raiz primitiva de q?E como e que a
ordem e exatamente p?
 --- edmilson motta [EMAIL PROTECTED] escreveu:
 Ei pessoal,

 voces notaram que o problema 6 da prova e' uma
 versao simplificada de um problema que eu e o
 Ed
 mandamos em uma das listas de treinamento do
 ano
 passado?  O problema da lista era algo assim:

 Sejam a,r1 e p um primo.  Prove que existe um
 primo q tal que (a mod q) tem ordem p^r.

Ola Prof Okakamo e demais
colegas desta lista ... OBM-L,

Protesto !

Eu, em OFF, estava dizendo ao Gugu que aguardassemos mais um tempo antes 
de
enviar solucoes das questoes IMO, para que os demais participantes,
sobretudo os que se preparam para as olimpiadas, tivessem oportunidade de
enviar as suas solucoes ou parte delas. Eu francamente acreditava e 
continuo
acreditando, que muitos membros serios, estudantes, tem totais condicoes 
de
resolver qualquer das questoes, alem da 4 ( que e trivial ).

Como nao conhecia o ridiculo plagio do Dirichlet, disse : Olha ai Gugu, 
viu
a solucao do Dirichlet ? Isso e uma prova de que devemos aguardar um 
pouco
mais !

Agora o Prof

Re: [obm-l] News from IMC!!!!!!! / Dirichlet

2003-07-28 Por tôpico Paulo Santa Rita
Ola Prof Gugu !

Em OFF estavamos conversando e eu lhe disse que deviamos segurar um pouco as 
solucoes porque muitas pessoas poderiam ainda enviar e se nos enviassemos, 
muito provavelmente elas se sentiriam desistimuladas. Inclusive, citando o 
Prof Okakamo, mencionei que o Dirichlet havia enviado uma solucao, o que 
provava que eu estava com a razao.

Eu estava preparando uma mensagem na qual analisava a solucao do Dirichlet 
e, caso estivesse correta, iria lhe prestar os merecidos parabens e dizer 
que todos deveriam seguir o exemplo e pensar nas demais questoes, pois eu e 
voce, estavamos aguardando isso.

Foi ai que o Prof Okakamo enviou a sua mensagem, denunciando o plagio. Eu me 
senti um idiota por ter acreditado na sinceridade do Dirichlet ( Anderson 
Torres ). Eu, inclusive, na minha mensagem, PARA NAO COMETER INJUSTICA, 
FALEI :

SE O PROF OKAKAMO NAO ESTIVER ENGANADO, eu me associo a sua indignacao e 
repudio ...

E so conferir a minha mensagem que voce vai ver a expressao acima. Mas, pelo 
que estou entendendo agora, o que realmente ocorreu foi um lamentavel engano 
do nosso estimado Prof Okakamo.

Um Abraco a Todos
Paulo Santa Rita
1,1303,280703



From: Carlos Gustavo Tamm de Araujo Moreira [EMAIL PROTECTED]
Reply-To: [EMAIL PROTECTED]
To: [EMAIL PROTECTED]
Subject: Re: [obm-l] News from IMC!!! / Dirichlet
Date: Mon, 28 Jul 2003 12:40:15 -0300 (EST)
   Oi Paulo,
   O que o Okakamo disse no e-mail dele foi Problema 6:
(...)
 e estimado pupilo Eduardo Tengan, que resolveu o
 problema 6 da IMO (de uma
 forma elegante) e o participante simplesmente copiou e
(...)
   Como o Tengan mandou uma solucao do problema 6 para a lista, por
intermedio do Edmilson (que o Dirichlet copiou e colou, citando a fonte),
eu achei que era isso. Porque voce afirma que o Okakamo se referiu ao
problema 3, e nao ao 6 ?
   Abracos,
Gugu

Ola Prof Gugu e demais
colegas desta lista ... OBM-L,

A mensagem a que o Prof Okakamo se refere e :


 INICIO DA MENSAGEM DO DIRICHLET


LEGALLEGALLEGAL!Consegui fechar o problema 3 da
IMO!!Confiram ai pra ver se nao tem nenhum erro.

Agradeço ao Paulo Santa Rita pelas suas dicas de vetores e ao Marcio 
Afonso
Assad Cohen pela ideia de usar desigualdades(e ao problema cinco da IMO 
da
India pela fabulosa ideia) e ao Tengan por ter me ensinado a aplicar a
tecnica  leve tudo ate as ultimas consequencias!!!.

O nosso hexagono tera o nome bastante original de ABCDEF cujos pontos 
medios
sao o hexagono nao menos original MNPQRS,com M em AB,N em BC e assim por
diante.
O enunciado afirma que MQ=(raiz(2) 3)/2 *(AB+DE)= e assim por 
diante.Devemos
mostrar que o hexagono e equiangulo.
Seja t=sen 30°=COS 60º=(raiz(2) 3)/2.
Começaremos demonstrando que os lados opostos são paralelos.

Considere a origem em qualquer lugar.O ponto medio de AB e (A+B)/2 entao 
o
vetor que liga os pontos medios de AB e DE e (A+B-D-E)/2.Assim
|A+B-D-E|=2t*(|A-B|+|D-E|).Para melhorar a joça vamos tentar usar a
desigualdade triangular para vetores.Sabemos que |A-B|+|D-E|=|A-B-D+E| 
com
igualdade se e somente se AB//DE(entendeu o paralelismo ai?).Assim
|AD+BE|=2t|AD-BE|.
Quadrando e usando produto interno euclidiano:
AD,AD+2DA,EB+EB,EB =
3*(AD,AD-2DA,EB+EB,EB) ou equivalentemente AD,AD+EB,EB
=4*AD,EB.Analogamente
EB,EB²+|FC,FC|²=4*EB,FC
FC,FC²+AD,AD²=-4*FC,DA.
Somando,obtemos 2*(DA-EB+FC)²=0.Logo
DA-EB+FC=0,e os lados opostos são paralelos(verifique as igualdades!).

0=A-D-B+E+C-F=BA+DC+FE,logo os vetores AB,CD e EF podem fechar um
triangulo.Isto ja e mais um ponto ao nosso favor!
Se transladarmos EF na direção ED,obtemos CG no prolongamento do lado
BC,devido ao paralelismo.Em miudos, desenhamos os paralelogramos
ADCH,ABEI,CFEG.Devido aos paralelismos produzimos um triângulo GHI com o
nosso ABCDEF dentro.Basta demonstrar que o triangulo GHI e 
equilatero.Para
tal:

DEVER DE CASA:cada mediana de um certo triangulo mede
sen 60°  vezes o lado correspondente.Calcule os angulos do
triangulo.(DICA:umas SLCs podem ajudar,ou use Stewart)

A PARTIR DAQUI FAÇA UM BOM DESENHO!

Os triangulos HAB,FIE,CDG e HIG são semelhantes por paralelismo.E pelo 
mesmo
motivo os tres primeiros triangulos sao congruentes e homoteticos ao
quarto(se lembra dos vetores?).Assim IE=DG e EQ=DQ,logo IQ=QG e assim 
sendo
Q e ponto medio de GI.Pela homotetia e visivel que H,M,Q  se alinham.
Vamos calcular a razao HQ/IG:
HQ/IG=HM/AB=(HM-HQ)/(AB-GI)=MQ/(AB+DE)=cos 60°,e de acordo com o dever de
casa(voce fez?),COMEMORE!!

Sera que ressa vai pra Eureka! ?Vou enviar JA!!!

UM ABRAÇAO!Ass.:Johann


** FIM DA MENSAGEM DO DIRICHLET



From: Carlos Gustavo Tamm de Araujo Moreira [EMAIL PROTECTED]
Reply-To: [EMAIL PROTECTED]
To: [EMAIL PROTECTED]
Subject: Re: [obm-l] News from IMC!!! / Dirichlet
Date: Mon, 28 Jul 2003 10:54:21 -0300 (EST)

   Caros Paulo e Okakamo,
Nao sei a qual mensagem do Dirichlet voces se referem, mas a que eu 
vi
(e
da qual reproduzo abaixo uma parte) sobre o

[obm-l] Equacao ( Niv 2 )

2003-08-01 Por tôpico Paulo Santa Rita
Ola Pessoal !

Alguem me propos a questao ( que compartilho com voces ) :

Quantas solucoes reais tem X^X = 2^(- RAIZ_2(X)), onde RAIZ_2(X) e a raiz 
quadrada de X.

Regra : Nao vale usar calculo !
Dica : X=1/e pode ser um ponto importante ...
Um Abraco
Paulo Santa Rita
6,1508,010803
_
MSN Messenger: converse com os seus amigos online.  
http://messenger.msn.com.br

=
Instruções para entrar na lista, sair da lista e usar a lista em
http://www.mat.puc-rio.br/~nicolau/olimp/obm-l.html
=


Re: [obm-l] nunca vi

2003-08-04 Por tôpico Paulo Santa Rita
Ola Rafael e demais
colegas desta lista ... OBM-L,
Eu ja vi este simbolo algumas vezes, tanto no calculo da N-esima derivada de 
algumas funcoes quanto em problemas. Em todas as circunstancias ele era 
usado para simplificar a notacao e com o significado :

X!! = X*(X-2)*(X-4)* ... ate o menor inteiro positivo.

Exemplos :

3!! = 3*1 = 3
4!! = 4*2* = 8
No Problema : Seja dado um poligono convexo de N lados. De quantas maneiras 
distintas e possivel
dividi-lo em triangulos usando-se apenas suas diagonais ? Esse tipo de 
fatorial surge naturalmente.

Todavia, por uma questao de prudencia, acho que voce nao deveria usar esta 
notacao em alguma prova ou concurso oficial, pois a banca pode nao conhecer 
e considerar seu raciocinio invalido. Claramente que X!! pode ser expresso 
por um produtorio, que todos conhecem.

Um Abraco
Paulo Santa Rita
2,1110,040803
From: [EMAIL PROTECTED]
Reply-To: [EMAIL PROTECTED]
To: [EMAIL PROTECTED]
Subject: Re: [obm-l] nunca vi
Date: Sun, 3 Aug 2003 23:12:08 EDT
Ha tambem a possibilidade de ser apenas um ponto de exclamacao indicando a
enfase no final de uma determinada resolucao, em que a solucao seria x!, 
tipicos
dos problemas de permutacao.

Em uma mensagem de 3/8/2003 23:49:19 Hora padrão leste da Am. Sul,
[EMAIL PROTECTED] escreveu:

 Bom, eu tambem nunca vi, mas -  quem sabe? – talvez o autor quisesse 
apenas
 dizer fatorial do fatorial, ou seja x!! = (x!)!
 Artur

 -Original Message-
 From: [EMAIL PROTECTED]
 [mailto:[EMAIL PROTECTED] On Behalf Of Alexandre 
Terezan
 Sent: Sunday, August 03, 2003 7:43 PM
 To: OBM
 Subject: [obm-l] nunca vi

 Alguém poderia me dizer o que significa o sinal de duas exlamações 
seguidas
 em matemática?



 x! = fatorial de x



 Mas o que é x!! ?



 Desde já agradeço,

 Alexandre Terezan






_
MSN Hotmail, o maior webmail do Brasil.  http://www.hotmail.com
=
Instruções para entrar na lista, sair da lista e usar a lista em
http://www.mat.puc-rio.br/~nicolau/olimp/obm-l.html
=


Re: [obm-l] x^x = 2^(-raiz(x))

2003-08-06 Por tôpico Paulo Santa Rita
Ola Claudio e demais
colegas desta lista ... OBM-l,
A sua solucao e bonita e usa recursos do nivel no qual ela foi proposta.

Segundo a pessoa que me mostrou, a questao foi apresentada para alunos com 
grau de estudo equivalente ao nosso nivel 2 ( setima e oitava series ). Essa 
a razao de nao se poder usar calculo na resolucao. Nao sei de qual pais e.

Na resposta considerava-se que zero tambem era raiz.

Um Abraco
Paulo Santa Rita
3,2325,050803
From: Claudio Buffara [EMAIL PROTECTED]
Reply-To: [EMAIL PROTECTED]
To: [EMAIL PROTECTED]
Subject: [obm-l] x^x = 2^(-raiz(x))
Date: Tue, 05 Aug 2003 19:05:51 -0300
on 01.08.03 15:10, Paulo Santa Rita at [EMAIL PROTECTED] wrote:

 Ola Pessoal !

 Alguem me propos a questao ( que compartilho com voces ) :

 Quantas solucoes reais tem X^X = 2^(- RAIZ_2(X)), onde RAIZ_2(X) e a 
raiz
 quadrada de X.

 Regra : Nao vale usar calculo !
 Dica : X=1/e pode ser um ponto importante ...

 Um Abraco
 Paulo Santa Rita
 6,1508,010803

Oi, Paulo:

O universo de x tem que ser o conjunto dos reais positivos.

x^x = 2^(-raiz(x)) ==

(x^raiz(x))^raiz(x) = (1/2)^raiz(x) ==

x^raiz(x) = 1/2

Vamos supor que x = 1/2^n. Nesse caso:

x^raiz(x) = (1/2^n)^(1/2^(n/2)) = (1/2)^(n/2^(n/2)) = 1/2 ==
n/2^(n/2) = 1 ==
n = 2^(n/2) ==
n^2 = 2^n ==
n = 2  ou  n = 4  ou  n = -a,
onde a eh um numero real positivo menor do que 1 e tal que a^2 = 2^(-a)
(repare que os graficos de y = x^2 e y = 2^x se intersectam num ponto de
abscissa negativa igual a -a. Nao faco a menor ideia se a eh racional ou
irracional ou mesmo transcendente, mas apostaria nessa ultima alternativa)
n  4 == 2^n  n^2 == n = 4 eh a maior solucao de n^2 = 2^n

Portanto:
n = 2 == x = 1/4
Testando:
x^x = (1/4)^(1/4) = (1/2)^(1/2) = 1/raiz(2)
2^(-raiz(x)) = 2^(-raiz(1/4)) = 2^(-1/2) = 1/raiz(2) ==
x = 1/4 eh raiz
n = 4 == x = 1/16
Testando:
x^x = (1/16)^(1/16) = (1/2)^(4/16) = 1/2^(1/4)
2^(-raiz(x)) = 2^(-raiz(1/16)) = 1/2^(1/4) ==
x = 1/16 eh raiz
n = -a == x = 2^a
Testando:
x^x = (2^a)^(2^a) = 2^(a*2^a) = 2^(a/a^2) = 2^(1/a)
2^(-raiz(x)) = 2^(-raiz(2^a)) = 2^(-raiz(1/a^2)) = 2^(-1/a) ==
x = 2^a nao eh raiz
Assim, a equacao original tem 2 solucoes: x = 1/4 e x = 1/16.

Um abraco,
Claudio.


=
Instruções para entrar na lista, sair da lista e usar a lista em
http://www.mat.puc-rio.br/~nicolau/olimp/obm-l.html
=
_
MSN Messenger: converse com os seus amigos online.  
http://messenger.msn.com.br

=
Instruções para entrar na lista, sair da lista e usar a lista em
http://www.mat.puc-rio.br/~nicolau/olimp/obm-l.html
=


[obm-l] Re: [obm-l] Número de soluções de sistemalinear

2003-08-14 Por tôpico Paulo Santa Rita
Ola Daibert e demais colegas
desta lista ... OBM-L,
Conforme outros membros desta lista ja observaram, a questao que voce propos 
nao tem solucao. Todavia, se voce aceitar que as solucoes sejam formadas por 
INTEIROS NAO NEGATIVOS, ela tem solucao. Alias, uma generalizacao natural 
PODE SER :

(Generalizacao) Discutir as solucoes formadas por INTEIROS NAO-NEGATIVOS da 
equacao :

A1*X1 + A2*X2 + ... + An*Xn = B

onde os Ai e o B sao inteiros positivos.

Um abraco
Paulo Santa Rita
4,1113,060803
on 06.08.03 02:15, Alexandre Daibert at [EMAIL PROTECTED] wrote:

Gostaria de ajuda para o seguinte problema:

Calcular o número de soluções do sistema:
16a + 8b + 4c + 2d + e = 23
sendo a, b, c, d, e inteiros positivos.
se possível usar somente conhecimentos de ensino médio, se isto não for
possível, pelo tente explicar mais ou menos o q está fazendo para q um
ignorante aluno q ainda não entrou em um curso superior possa entender :-)
Alexandre Daibert - Juiz de Fora - [EMAIL PROTECTED]
_
MSN Messenger: converse com os seus amigos online.  
http://messenger.msn.com.br

=
Instruções para entrar na lista, sair da lista e usar a lista em
http://www.mat.puc-rio.br/~nicolau/olimp/obm-l.html
=


Re: [obm-l] ENQUETE - BELEZA MATEMATICA

2003-08-14 Por tôpico Paulo Santa Rita
 
inverossimilhancas, nao a CAUSA UNICA delas ... E que cada um engula isso 
como puder !

Na Matematica, como em tudo, a Normalidade, quando nao e a expressao viva da 
mediocridade, e apenas uma face da patologia ... E sempre ha uma multidao de 
sacerdotes dispostos a defender os credos antigos e acusar os raciocinios 
estranhos e diferentes com as formas modernas da inquisicao, mesmo que a 
historia esteja diuturnamente demonstrando que o progresso jamais promana 
daquilo que e comum e batido, daquelas implicacoes logicas limpinhas e 
bonitinhas.

Uma curva gaussiana das normalidades humanas, em qualquer ambiente, seja 
academico, social, desportivo e mesmo profissional, e sofrivel ... A pior 
desgraca que pode suceder a um ser humano, e ele ser normal !

Perdao por este OFF-DESABAFO final.

Um Abraco Cordial a Todos !
Paulo Santa Rita
1,2107,100803

From: A. C. Morgado [EMAIL PROTECTED]
Reply-To: [EMAIL PROTECTED]
To: [EMAIL PROTECTED]
Subject: Re: [obm-l] ENQUETE - BELEZA MATEMATICA
Date: Sun, 10 Aug 2003 19:00:05 -0300
Desculpe a ignorancia, poderia detalhar mais a segunda escolha?

Paulo Santa Rita wrote:

Ola Claudio !

Muito legal essa sua enquete. Bom, so pode entrar resultados elementares 
e/ou de facil compreensao, certo ? Entao me ocorre de imediato alguns 
resultados.

PRIMEIRO ( trivial, mas mercece um quadro na parece. Devido a Bernoulli )

1^P + 2^P + 3^P + ... + (N-1)^P + N^P = [(N+B)^P  -  B^P]/(P+1)
onde B^k deve ser interpretado como o K-esimo numero de bernoulli.
Alias, foi verificando as somas das potencias P-esimas dos numeros 
naturais que Bernoulli descobriu os fantasticos numeros que hoje levam o 
seu nome. Mais adiante, quando eu estiver mais tranquilo, vou escrever 
sobre este tema.

SEGUNDO ( Isso nao e um principio, e um Salmo do Profeta. Devido a Erdos )

Se em um conjunto de objeto, um objeto tem uma probabilidade menor que 1 
de ter uma determinada propriedade, entao existe um objeto do conjunto com 
aquela propriedade

Esse principio, nao obstante muito contestado e criticado por alguns, e 
poderoso e acredito que abre novas e imensas possibilidades para o 
pensamente matematico.

TERCEIRO ( trivial, mas facilita a prova de muitas coisas. A desigualdade 
Eduardo Wagner )

Em todo triangulo, o semi-perimetro nunca e menor que a soma dos produtos 
de cada lado pelo cosseno do angulo oposto

p = a*cosA + b*cosB + c*cosC

Com a desigualdade acima da pra derivar quase todas as desigualdades 
complicadas da Geometria Elemntar.

Um Abraco
Paulo Santa Rita
7,1425,090803
EM TEMPO. Sobre a beleza matematica :

A Divina Proporcao
Um Ensaio sobre a beleza na Matematica
H. E. Huntley
Editora UnB
O autor mostra como o numero fi, ( 1 + raiz_quadrada(5) )/2, aparece nas 
mais diversas circunstancias e inesperadas circunstancias, sempre com um 
toque de inegavel beleza. Eu acredito que este numero contribuem pelo 
menos com um resultado :

A UNICA progressao geometrica de termos positivos que na qual An+1 = An + 
An-1 e a sequencia :
1, fi, fi^2, fi^3, fi^4, ...


From: Claudio Buffara [EMAIL PROTECTED]
Reply-To: [EMAIL PROTECTED]
To: Lista OBM [EMAIL PROTECTED]
CC: Claudio Buffara [EMAIL PROTECTED]
Subject: [obm-l] ENQUETE - BELEZA MATEMATICA
Date: Sat, 09 Aug 2003 10:24:26 -0300
Caros colegas da lista:

Gostaria de contar com sua participacao numa enquete sobre beleza
matematica.
O que eu precisao eh que cada um de voces me envie uma lista contendo 
algo
como 5 a 10 problemas/teoremas que voces consideram os mais bonitos e 
cujas
solucoes/demonstracoes sao as mais elegantes e/ou inusitadas e/ou
engenhosas. Nao precisa incluir a solucao/demonstracao, apenas o 
enunciado.
No entanto, se voce tiver em mente uma solucao/demonstracao especifica
(entre varias existentes) nao deixe de mencionar pelo menos o metodo
utilizado.

A unica restricao eh que estes resultados devem ser de um nivel acessivel 
a
um aluno normal de 2o. grau (ou seja, o Ultimo Teorema de Fermat e o 
Porisma
de Poncelet estao fora, mas o caso n = 4 do UTF e a versao para 
triangulos
do Porisma poderiam ser incluidos).

Importante: os resultados devem ser acessiveis a um aluno normal de 2o.
grau, mas nao necessariamente fazer parte do curriculo normal do 2o. 
grau.

Tambem nao precisa responder hoje ou amanha ou mesmo na semana que vem. 
Acho
que vale a pena pensar por um tempo e consultar a literatura - as vezes 
pode
ter um resultado belissimo do qual voce simplesmente se esqueceu por nao
encontra-lo ha muito tempo. As Eurekas sao uma otima referencia. O 
Proofs
from the Book tambem, apesar de nem tudo lah ter nivel de 2o. grau.

Se houver um numero suficiente de respostas, eu me comprometo a publicar 
uma
compilacao dos problemas e teoremas mais votados.

Desde jah a gradeco o interesse de quem quiser participar.

Um abraco,
Claudio.
=

Instruções para entrar na lista, sair da lista e usar a lista em
http://www.mat.puc-rio.br/~nicolau/olimp/obm-l.html

Re: [obm-l] ENQUETE - BELEZA MATEMATICA

2003-08-14 Por tôpico Paulo Santa Rita
Ola Claudio e demais colegas
desta lista ... OBM-L,
Carissimo Claudio : a ironia funciona quando o ironizado tem alguma nocao do 
que seja o ridiculo, o que nao parece ser o caso 

Acredito firmemente que as pessoas que frequentam esta nossa lista sao, em 
sua grande maioria, estudantes dedicados, que seriamente buscam aqui 
adquirir conhecimentos e esclarecer suas duvidas, seja porque irao 
participar de Olimpiadas, seja porque prestarao algum vestibular que exija 
algum treinamento maior em matematica. Enfim, sao pessoas serias que nao 
podem e nao devem estar dispostas a perder tempo com besteiras e maluquices.

Perolas, como as destacadas pelo Claudio na mensagem abaixo, nao 
contribuem em nada para o aperfeicoamento desta nossa lista, servindo tao 
somente para desvaloriza-la, afastando algumas pessoas de real valor e 
formando uma imagem - que se propaga - negativa que nenhum de nos deseja ver 
associada aos lugares que prezamos e frequentamos.

Toda mensagem deve ser responsavel, vale dizer, IDEALMENTE, deve carregar o 
que ha de melhor em nosso espirito. Deve ser a expressao de uma duvida 
sincera ou um esclarecimento consciente.

E verdade que sempre ha uma distancia, grande ou pequena, entre aquilo que 
idealizamos e aquilo que realizamos, mas, as pessoas inteligentes percebem a 
diferenca entre uma digressao inocente ( um OFF) e uma atitude diuturna e 
malevola, fruto de uma paixao qualquer ... Como seres humanos podemos errar, 
mas nao podemos errar sempre, pois isso seria uma burrice ou maldade 
incompativel com a qualidade que almejamos para esta lista.

Enfim. essa lista e a nossa casa. Devemos, portanto, ter cuidados com ela. 
Devemos trazer para o nosso convivio as melhores pessoas que conhecermos e 
laborar para que elas permanecam e possam testemunhar sobre a qualidade de 
nosso trabalho e de nossa educacao.

A verdadeira mensagem subjacente a ironia do Claudio e um proteto 
silencioso, util, verdadeiro, que eu torno claro e que bem poderia ser 
tomado como ponto de partida para que as pessoas explicitamente mencionadas 
TIVESSEM VERGONHA NA CARA e deixassem de publicar asneiras e insanidades.

Um Abraco a Todos
Paulo Santa Rita
2,1937,110803
Os mafiosos tem comparsas; os criminosos, cumplices; os corruptos, 
associados : mas so os homens de bem tem amigos (Voltaire)

Amigo e coisa pra se guardar do lado esquerdo do peito !
(Milton Nascimento - Gravado na camisa de alguem que me visitou )
From: Claudio Buffara
To: Lista OBM
Sent: Monday, August 11, 2003 1:16 AM
Subject: [obm-l] ENQUETE - BELEZA MATEMATICA
Caros colegas:

Estou extremamente decepcionado com as listas de problemas supostamente
bonitos que foram enviadas pra lista ateh o presente momento. Imaginem soh 
-
teorema do valor intermediario, secoes conicas, poliedros regulares,
conjuntos enumeraveis. Onde voces estao com a cabeca? Isso tudo eh
matematica do tempo da carochinha. Infinitude dos primos? Isso eh tao velho
que ja devia ter sido revogado!

Pessoal, vamos olhar pra frente, prestigiar a modernidade e, mais
importante, os talentos matematicos nacionais. Com isso em mente, elaborei
uma lista que deixaria o David Hilbert morrendo de inveja. E o que eh
melhor, com problemas propostos pelas mentes mais privilegiadas da nossa
lista - aqueles que certamente vao cobrir o Brasil de gloria com medalhas
Fields, premios Abel e ainda fazer os suecos criarem o premio Nobel de
matematica. Sendo assim, aqui vao os problemas que pautarao o
desenvolvimento da matematica nos seculos vindouros.
As 7 primeiras perolas (perolas? brilhantes, isso sim!) sao de autoria do
nobre sabio Jorge Luiz de Fortaleza. As 5 restantes - puro genio - do nosso
querido e muitas vezes incompreendido mestre Dirichlet.
Problemas e teoremas do Jorge Luiz:

1) A pegadinha no Pereira.

2) O caramujo.

3) A margem de erro real.

4) A compra, venda, recompra e revenda.

5) Apostar ou jogar uma moeda.

6) As páginas dos dicionários.

7) O método cuca legal de multiplicação.

Problemas e teoremas do Dirichlet:

8) O da sequencia que sai facil por Kronecker - eh soh pegar os numeros das
pontas e das extremidades e ir multiplicando de fora pra dentro;
9) O do triangulo que voce expressa o seno em funcao da tangente da metade
do angulo externo do triangulo da pagina ao lado e cai num polinomio que os
fatores se cancelam e se nao se cancelarem eh que voce errou alguma conta
mas tudo bem que tambem tem a solucao cearense.
10) Aquele que voce aplica Schur 3 vezes e depois divide o intervalo em 4
sub-intervalos e aplica Schur mais umas 3 ou 4 vezes e ai fica trivial -
basta usar Fermat, ou Cauchy, ou Euler, ou um cara desses ai.
11) Tem o outro que sai pelo teorema de Turan-Erdos-Von Neumann e mais
outros dois ou tres hungaros que eu nao lembro o nome e ai com 4n^2+3  
2n+1
 7n^3+9  111  112  2m eh soh achar os dois grafos sem triangulos no 
meio do
emaranhado de pontinhos.

12) Finalmente, tem aquele da soma dos senos que eh soh ver a Eureka e - 
ah!
meu! Esse ai eh

Re: [obm-l] ENQUETE - BELEZA MATEMATICA

2003-08-14 Por tôpico Paulo Santa Rita
Ola Claudio !

Muito legal essa sua enquete. Bom, so pode entrar resultados elementares 
e/ou de facil compreensao, certo ? Entao me ocorre de imediato alguns 
resultados.

PRIMEIRO ( trivial, mas mercece um quadro na parece. Devido a Bernoulli )

1^P + 2^P + 3^P + ... + (N-1)^P + N^P = [(N+B)^P  -  B^P]/(P+1)
onde B^k deve ser interpretado como o K-esimo numero de bernoulli.
Alias, foi verificando as somas das potencias P-esimas dos numeros naturais 
que Bernoulli descobriu os fantasticos numeros que hoje levam o seu nome. 
Mais adiante, quando eu estiver mais tranquilo, vou escrever sobre este 
tema.

SEGUNDO ( Isso nao e um principio, e um Salmo do Profeta. Devido a Erdos )

Se em um conjunto de objeto, um objeto tem uma probabilidade menor que 1 de 
ter uma determinada propriedade, entao existe um objeto do conjunto com 
aquela propriedade

Esse principio, nao obstante muito contestado e criticado por alguns, e 
poderoso e acredito que abre novas e imensas possibilidades para o 
pensamente matematico.

TERCEIRO ( trivial, mas facilita a prova de muitas coisas. A desigualdade 
Eduardo Wagner )

Em todo triangulo, o semi-perimetro nunca e menor que a soma dos produtos de 
cada lado pelo cosseno do angulo oposto

p = a*cosA + b*cosB + c*cosC

Com a desigualdade acima da pra derivar quase todas as desigualdades 
complicadas da Geometria Elemntar.

Um Abraco
Paulo Santa Rita
7,1425,090803
EM TEMPO. Sobre a beleza matematica :

A Divina Proporcao
Um Ensaio sobre a beleza na Matematica
H. E. Huntley
Editora UnB
O autor mostra como o numero fi, ( 1 + raiz_quadrada(5) )/2, aparece nas 
mais diversas circunstancias e inesperadas circunstancias, sempre com um 
toque de inegavel beleza. Eu acredito que este numero contribuem pelo menos 
com um resultado :

A UNICA progressao geometrica de termos positivos que na qual An+1 = An + 
An-1 e a sequencia :
1, fi, fi^2, fi^3, fi^4, ...


From: Claudio Buffara [EMAIL PROTECTED]
Reply-To: [EMAIL PROTECTED]
To: Lista OBM [EMAIL PROTECTED]
CC: Claudio Buffara [EMAIL PROTECTED]
Subject: [obm-l] ENQUETE - BELEZA MATEMATICA
Date: Sat, 09 Aug 2003 10:24:26 -0300
Caros colegas da lista:

Gostaria de contar com sua participacao numa enquete sobre beleza
matematica.
O que eu precisao eh que cada um de voces me envie uma lista contendo algo
como 5 a 10 problemas/teoremas que voces consideram os mais bonitos e cujas
solucoes/demonstracoes sao as mais elegantes e/ou inusitadas e/ou
engenhosas. Nao precisa incluir a solucao/demonstracao, apenas o enunciado.
No entanto, se voce tiver em mente uma solucao/demonstracao especifica
(entre varias existentes) nao deixe de mencionar pelo menos o metodo
utilizado.
A unica restricao eh que estes resultados devem ser de um nivel acessivel a
um aluno normal de 2o. grau (ou seja, o Ultimo Teorema de Fermat e o 
Porisma
de Poncelet estao fora, mas o caso n = 4 do UTF e a versao para triangulos
do Porisma poderiam ser incluidos).

Importante: os resultados devem ser acessiveis a um aluno normal de 2o.
grau, mas nao necessariamente fazer parte do curriculo normal do 2o. grau.
Tambem nao precisa responder hoje ou amanha ou mesmo na semana que vem. 
Acho
que vale a pena pensar por um tempo e consultar a literatura - as vezes 
pode
ter um resultado belissimo do qual voce simplesmente se esqueceu por nao
encontra-lo ha muito tempo. As Eurekas sao uma otima referencia. O Proofs
from the Book tambem, apesar de nem tudo lah ter nivel de 2o. grau.

Se houver um numero suficiente de respostas, eu me comprometo a publicar 
uma
compilacao dos problemas e teoremas mais votados.

Desde jah a gradeco o interesse de quem quiser participar.

Um abraco,
Claudio.
=
Instruções para entrar na lista, sair da lista e usar a lista em
http://www.mat.puc-rio.br/~nicolau/olimp/obm-l.html
=
_
MSN Hotmail, o maior webmail do Brasil.  http://www.hotmail.com
=
Instruções para entrar na lista, sair da lista e usar a lista em
http://www.mat.puc-rio.br/~nicolau/olimp/obm-l.html
=


[obm-l] Re: [obm-l] Mat. = ciência exata?

2003-09-30 Por tôpico Paulo Santa Rita
Ola Duda e demais
colegas desta lista ... OBM-L,
Se bem entendi a sua colocacao, o pessoal da Licenciatura tem uma concepcao 
da Matematica semelhante a do Leibniz. Esse Matematico - que descobriu o 
Calculo Diferencial, entre muitas outras contribuicoes - reiteradamente 
afirmava  que Se nos percebermos internamente as necessidades externas do 
mundo, encontraremos caminhos validos para tratar dos grandes problemas 
matematicos

Segundo este Matematico, uma obra Matematica valida e um produto de carater 
universal e intemporal, que se impoe alem de toda critica por se fundamentar 
em provas incontestaveis. Assim, um Matematico, por mais egoista e simplorio 
que seja, ao descobrir e/ou desenvolver novos ramos, estara dando uma 
contribuicao para toda a humanidade e, neste sentido, a sua mente 
necessariamente entrou em sintonia com as grandes necessidades estruturais 
que caracrterizam um determinado momento historico.

Isto explicaria descobertas semelhantes por pessoas distintas que nao se 
conhecem : simplesmente e uma necessidade do momento, isto e, nao haveria 
uma outra forma de tratar e abordar as questoes. Entretanto, e importante 
que se diga que sentir qual a direcao do progresso humano em um ramo 
qualquer do conhecimento e uma qualidade rara, em cada seculo parecendo 
haver poucas pessoas com tal habilidade ( como, sem duvida, tinha o Leibniz 
).

Quais seriam, hoje, as necessidades do mundo ?

Um cara que desenvolveu esta tese ate as ultimas consequencias foi o Hegel, 
segundo o qual a historia ( cultura ) explica todos os fenomenos com base na 
triade dialectica tese-antitese-sintese : alguem propoe uma tese ( por 
exemplo, a mecanica newtoniana ) que vai se desgastando e mostrando 
fissuras. Surge uma antitese que procura contornar os problemas ( por 
exemplo, a mecanica quantica ). Com o passar do tempo vem a uniao harmonioso 
das duas, isto e, a sintese. A sintese passa a ser a tese e todo o ciclo se 
repete, sucessivamente.

Bom, fazer Matematica e sem duvida agradavel, mas, dado as implicacoes 
humanas que dela promanam, nao pode ser uma mera brincadeira ... Existe uma 
coisa seria aqui. Como Matematicos, qual a nossa responsabiolidade ? Buscar 
somente a propria gloria ? Evidentemente que nao e isso ...

A verdade sempre foi, e, e sera um dos maiores valores de nossa cultura ... 
E parece certo que a Matematica nao e a Verdade, mas, hoje, ja e 
igualmente certo que nao chegaremos a Verdade, qualquer que ela seja, sem 
antes passar pela Matematica.

Um Abraco
Paulo Santa Rita
3,1004,300903

From: Eduardo Casagrande Stabel [EMAIL PROTECTED]
Reply-To: [EMAIL PROTECTED]
To: [EMAIL PROTECTED]
Subject: [obm-l] Mat. = ciência exata?
Date: Tue, 30 Sep 2003 00:28:33 -0300
Olá!

Estou cursando a cadeira de História da Matemática, junto com o pessoal da
Licenciatura em Matemática. Um colega disse, em sala de aula, que a
Matemática é uma ciência humana. Eu achei a idéia muito boba, mas,
conversando com uma colega, constatei - para meu espanto - que há grupos de
pesquisa que estudam (?!) a possibilidade de a matemática não ser uma
ciência exata, querendo significar (pelo que eu entendi) que a matemática é
cultural, dependendo do contexto histórico ou algo assim.
Para ser franco, como muitas das coisas discutidas, eu não consegui
compreender sobre o que se falava. Parece que o pessoal da Licenciatura tem
uma visão de matemática muito identificada com educação matemática. Alguém
tem alguma idéia de o que eu estou falando?!
Abraço,
Duda.
=
Instruções para entrar na lista, sair da lista e usar a lista em
http://www.mat.puc-rio.br/~nicolau/olimp/obm-l.html
=
_
MSN Messenger: converse com os seus amigos online.  
http://messenger.msn.com.br

=
Instruções para entrar na lista, sair da lista e usar a lista em
http://www.mat.puc-rio.br/~nicolau/olimp/obm-l.html
=


[obm-l] Re: [obm-l] Re: [obm-l] Mat. = ciência exata?

2003-09-30 Por tôpico Paulo Santa Rita
Ola Bruno e demais
colegas desta lista ... OBM-L,
Eu li a obra do Kapra a que voce se refere, bem como as demais obras dele. 
Em A teia da vida, ele apresenta o que poderiamos chamar de suas alegacoes 
finais ... Em sintese, ele procura desenvolver conceitos que possam unificar 
diversos desenvolvimentos cientificos e culturais contemporaneos, 
delimitando assim um modelo conceitual que, segundo ele, nos permitirde 
compreender o porvir.

Mas Kapra nao faz ciencia, ele anuncia possibilidades. Talvez ele esteja 
para a nossa epoca assim como Francis Bacon estava para o Renascimento. De 
fato, no Novo Organon, este filosofo antecipou em decadas e com notavel 
precisao quase todos os desenvolvimentos cientificos e possibilidades 
tecnologicas, descrevendo perfeitamente o metodo esperimental. Assim, num 
futuro proximo nao se surpresa se algum historiador comparar Kapra com 
Bacon.

Mas Bacon nao era um Matematico, assim como Kapra nao e. Mas Leibniz, nao so 
foi um brilhante Matematico, mas Estadista de Escol tambem. Portanto, quando 
ele afirma que os problemas externos do mundo influenciam e sugerem caminhos 
validos para a solucao de Problemas especificos da Matematica, delineando 
assim um imbricamento de coisas que dificilmente suspeitariamos, trata-se da 
palavra de quem conhece muito bem ambas as coisas. E portanto, em algum 
sentido, um testemunho de maior peso. Eis a razao de eu ter citado este 
Matematico :

Numa lista de discussao de Matematica, a palavra de um ( Grande ? ) 
Matematico !

A proposito, aqui no Brasil tem havido uma enorme discussao sobre o cultivo 
e comercializacao da soja transgenica. Os doutores afirmam que e 
fundamental um Estudo de Impacto Ambiental, para que possamos prever as 
implicacoes nocivas ou nao da introducao de especies geneticamente 
modificadas. Ate parece que isto e uma tecnica bem conhecida, facil de fazer 
...

Em Verdade, nos NAO SABEMOS avaliar com a necessaria precisao os efeitos da 
introducao de novas especies em ecosistemas especificos, nao obstante SER 
PREMENTE aprendermos a fazer estes calculos e avaliacoes, sob pena de 
reiteradamente apreciarmos os desequilibrios que estamos provocando em 
funcao de nossa ignorancia, que ameacam mesmo a nossa propria sobrevivencia.

Todos estes problemas se referem, direta ou indiretamente, a nossa 
capacidade de avaliar o papel ou funcao que um objeto dotado de aparente 
intencionalidade vem a desempenhar quando introduzido num sistema dinamico 
que mantem uma delicada estabilidade ... Vai ocorrer uma catastrofe ? O 
sistema vai evoluir para uma nova e harmonica estabilidade ?

Tai um verdadeiro problema atual, objetivo e tragico ... Nos nao temos a 
opcao de nao resolve-lo ou de remediar eternamente a sua solucao. Nao e uma 
brincadeira, nao e diletantismo, nao e prazer. E agora Jose ? Como 
equaciona-lo ?

Um Abraco
Paulo Santa Rita
3,2210,300903
From: Bruno Simões [EMAIL PROTECTED]
Reply-To: [EMAIL PROTECTED]
To: [EMAIL PROTECTED]
Subject: Re: [obm-l] Re: [obm-l] Mat. = ciência exata?
Date: Tue, 30 Sep 2003 11:53:24 -0300 (ART)
Longe que querer parecer um especialista no assunto, e
já tentando contribuir para a lista, recomendo a
leitura de O Ponto de Mutação de Kapra. Lá há um bom
texto sobre as influências culturais sobre o
pensamento científico... Sobre os interesses dos
matemáticos, ouvi falar de um sujeito nos EUA que
parou de pesquisar sobre hiperespaços ao saber da
possível utilidade prática de seus resultados na
teoria de codificação de informações... O que pensam a
respeito?


 --- Paulo Santa Rita [EMAIL PROTECTED] escreveu: 
Ola Duda e demais
 colegas desta lista ... OBM-L,

 Se bem entendi a sua colocacao, o pessoal da
 Licenciatura tem uma concepcao
 da Matematica semelhante a do Leibniz. Esse
 Matematico - que descobriu o
 Calculo Diferencial, entre muitas outras
 contribuicoes - reiteradamente
 afirmava  que Se nos percebermos internamente as
 necessidades externas do
 mundo, encontraremos caminhos validos para tratar
 dos grandes problemas
 matematicos

 Segundo este Matematico, uma obra Matematica valida
 e um produto de carater
 universal e intemporal, que se impoe alem de toda
 critica por se fundamentar
 em provas incontestaveis. Assim, um Matematico, por
 mais egoista e simplorio
 que seja, ao descobrir e/ou desenvolver novos ramos,
 estara dando uma
 contribuicao para toda a humanidade e, neste
 sentido, a sua mente
 necessariamente entrou em sintonia com as grandes
 necessidades estruturais
 que caracrterizam um determinado momento historico.

 Isto explicaria descobertas semelhantes por pessoas
 distintas que nao se
 conhecem : simplesmente e uma necessidade do
 momento, isto e, nao haveria
 uma outra forma de tratar e abordar as questoes.
 Entretanto, e importante
 que se diga que sentir qual a direcao do progresso
 humano em um ramo
 qualquer do conhecimento e uma qualidade rara, em
 cada seculo parecendo
 haver poucas pessoas com tal habilidade ( como, sem
 duvida, tinha o Leibniz

Re: [obm-l] Mais problemas Sobre Grupos

2003-10-20 Por tôpico Paulo Santa Rita
Ola Carlos e demais colegas
desta lista ... OBM-L,
Vou contribuir um pouquinho ...

G) Sendo e a identidade, de  Y^2= e para todo Y em G concluimos que Y^-1 = 
Y ( Voce saberia dizer porque posso fazer esta afirmacao ? ). Sejam a e 
b dois elementos quaisquer do Grupo. Entao ab e (ab)^-1 estao em G e, pelo 
que vimos :

ab=(ab)-1 = ab=(b^-1)(a^-1)  mas b^-1=b e a^-1 = a. Segue que :
ab=ba, para quaisquer a e b em G. O grupo e portanto abeliano.
Observe que este resultado tem uma consequencia imediata, qual seja : Todo 
Grupo de ordem menor ou igual a 5 e ciclico. Prove isso !

Dois outros problemas elementares sobre Grupos :

1) Seja G um grupo e G' o subgrupo dos comutadores. Prove que o quociente 
G/G' e abeliano.

2) Seja G um grupo de ordem p^n, p primo e n =3. Mostre que se o centro de 
G tem ordem p entao existe uma classe de conjugacao de ordem p.

Um Abraco a Todos
Paulo Santa Rita
2,1012,201003
From: Carlos Maçaranduba [EMAIL PROTECTED]
Reply-To: [EMAIL PROTECTED]
To: [EMAIL PROTECTED]
Subject: [obm-l] Mais problemas Sobre Grupos
Date: Sun, 19 Oct 2003 20:32:19 -0300 (ART)
MIME-Version: 1.0
Received: from mc5-f8.hotmail.com ([65.54.252.15]) by mc5-s21.hotmail.com 
with Microsoft SMTPSVC(5.0.2195.5600); Mon, 20 Oct 2003 04:15:02 -0700
Received: from sucuri.mat.puc-rio.br ([139.82.27.7]) by mc5-f8.hotmail.com 
with Microsoft SMTPSVC(5.0.2195.5600); Mon, 20 Oct 2003 04:15:01 -0700
Received: (from [EMAIL PROTECTED])by sucuri.mat.puc-rio.br (8.9.3/8.9.3) 
id VAA05070for obm-l-MTTP; Sun, 19 Oct 2003 21:00:41 -0300
Received: from web21109.mail.yahoo.com (web21109.mail.yahoo.com 
[216.136.227.111])by sucuri.mat.puc-rio.br (8.9.3/8.9.3) with SMTP id 
UAA04957for [EMAIL PROTECTED]; Sun, 19 Oct 2003 20:59:41 -0300
Received: from [200.164.247.30] by web21109.mail.yahoo.com via HTTP; Sun, 
19 Oct 2003 20:32:19 ART
X-Message-Info: NDMZeIBu+soqT/9tqALIbVX3Lxac9UkwSv5iQMq7xO4=
Message-ID: [EMAIL PROTECTED]
In-Reply-To: [EMAIL PROTECTED]
Sender: [EMAIL PROTECTED]
Precedence: bulk
Return-Path: [EMAIL PROTECTED]
X-OriginalArrivalTime: 20 Oct 2003 11:15:01.0949 (UTC) 
FILETIME=[67D536D0:01C396FB]

Eu consegui provar a letra a o resto nao) ai vão:

Seja Z conjunto dos inteiros e x o subgrupo gerado
por x e Zm um grupo qualquer mod m. Mostre que a,b e m
inteiros( m= 2):
a)sendo B = b (mod m) e A = a (mod m) se a divide b
entao, como subgrupos de Zm,
B esta contido em A.(Esse eu consegui provar o
resto nao)
b)sendo A = a (mod m) se mdc(a,m) = 1 , entao A =
Zm.
c)sendo A = a (mod m) e D = d (mod m) se mdc(a,m) = d
, entao A = D.
d) De posse das informacoes acima, determine todos os
subgrupos de (Z36 , +).
e)Mostre  que se (G , *) é um grupo multiplicativo de
ordem 2 entao G é ciclico.
f)Mostre  que se (G , *) é um grupo multiplicativo de
ordem 3 entao G é ciclico.(Sugestao: Sendo G =
{x,a,b}, x o elemento neutrode G, pense sobre o que
poderia ser o elemento ab)
g)Mostre  que se (G , *) é um grupo multiplicativo de
elemento neutro x , mostre que se y^2 =x, para cada y
em G, entao G é abeliano.(Sugestao:Note que y^2 = x
implica que y^-1 = x .Tome 2 elementos quaisquer a e b
em G e comece escrevendo ab = (ab)^-1 = ...)
Yahoo! Mail - o melhor webmail do Brasil
http://mail.yahoo.com.br
=
Instruções para entrar na lista, sair da lista e usar a lista em
http://www.mat.puc-rio.br/~nicolau/olimp/obm-l.html
=
_
MSN Hotmail, o maior webmail do Brasil.  http://www.hotmail.com
=
Instruções para entrar na lista, sair da lista e usar a lista em
http://www.mat.puc-rio.br/~nicolau/olimp/obm-l.html
=


Re: [obm-l] SOMA(n=1) (1/n)*((2+sen(n))/3)^n

2003-10-23 Por tôpico Paulo Santa Rita
Oi Duda, Claudio e demais
colegas desta lista ... OBM-L,
Quando o Duda colocou a questao, a nossa lista estava sendo agredida por 
maluco qualquer e pensei que ele, corretamente,  desejando reafirmar a 
inegavel qualidade de nossas discussoes e colocar o maluco onde deveria 
ficar, isto e, no ostracismo, apresentava um daqueles exemplos em que e 
impossivel concluir sobre a convergencia usando os testes convencionais 
diretos mais conhecidos, tais como o teste da razao e da raiz.

Eu mesmo procedi assim, resolvendo algumas questoes elementares sobre grupos 
que algum membro havia proposto. Sem pensar muito, vou dizer o que vejo. Os 
colegas, por favor, corrijam os erros e completem as lacunas

Numa primeira analise, claramente que para qualquer N temos que 1/3  [( 2 + 
sen(N) )/3]  1 e dai segue que [ ( 2 + sen(N) )/3 ]^N  1=  (1/N)*[ ( 
2 + sen(N) )/3 ]^N  1/N, isto e, para todo natural N positivo  existe um 
real r(N)  1 tal que :

(1/N)*[ ( 2 + sen(N) )/3 ]^N = 1 / ( N^r(N) )

Agora :

S = somatorio(1 ate +INF) de i^[ - r(i) ] , r( i )  1, converge ? Para mim, 
e evidente que sim, e eu ficaria satisfeito em chegar aqui, pois, para N 
suficientemente grande, tomando r=MIN{ r(1), ...,r(N) }, teriamos r  1 e a 
serie de termo geral 1/N^r converge, alem de que 1/N^r = 1/[ N^r(N) ]. Mas 
claramente e possivel colocar mais rigor aqui ...

Finalmente, observo aqui uma semelhanca com as series de Fourier, na qual 
para tratarmos de continuidade e diferenciabilidade de serie de funcoes, 
precisamos considerar o conceito de convergencia uniforme para dar maior 
rigor as nossas conclusoes ...

Um Abraco a Todos
Paulo Santa Rita
5,1040,231003
From: Claudio Buffara [EMAIL PROTECTED]
Reply-To: [EMAIL PROTECTED]
To: [EMAIL PROTECTED]
Subject: [obm-l] SOMA(n=1) (1/n)*((2+sen(n))/3)^n Date: Thu, 23 Oct 2003 
08:13:02 -0200
MIME-Version: 1.0
Received: from mc3-f29.hotmail.com ([64.4.50.165]) by mc3-s14.hotmail.com 
with Microsoft SMTPSVC(5.0.2195.6713); Thu, 23 Oct 2003 03:13:54 -0700
Received: from sucuri.mat.puc-rio.br ([139.82.27.7]) by mc3-f29.hotmail.com 
with Microsoft SMTPSVC(5.0.2195.6713); Thu, 23 Oct 2003 03:11:51 -0700
Received: (from [EMAIL PROTECTED])by sucuri.mat.puc-rio.br (8.9.3/8.9.3) 
id HAA07413for obm-l-MTTP; Thu, 23 Oct 2003 07:10:44 -0300
Received: from ivoti.terra.com.br (ivoti.terra.com.br [200.176.3.20])by 
sucuri.mat.puc-rio.br (8.9.3/8.9.3) with ESMTP id HAA07408for 
[EMAIL PROTECTED]; Thu, 23 Oct 2003 07:10:43 -0300
Received: from bertioga.terra.com.br (bertioga.terra.com.br 
[200.176.3.77])by ivoti.terra.com.br (Postfix) with ESMTP id B345479C43Efor 
[EMAIL PROTECTED]; Thu, 23 Oct 2003 08:10:12 -0200 (BRST)
Received: from [200.182.232.19] (23219.virtua.com.br 
[200.182.232.19])(authenticated user claudio.buffara)by 
bertioga.terra.com.br (Postfix) with ESMTP id 3B0743F80CBfor 
[EMAIL PROTECTED]; Thu, 23 Oct 2003 08:10:12 -0200 (BRST)
X-Message-Info: vGzX0e+ktu4YML//s4F92ccRUO8Jol1Nf5SIT2lHQYk=
User-Agent: Microsoft-Outlook-Express-Macintosh-Edition/5.02.2022
Message-ID: [EMAIL PROTECTED]
In-Reply-To: [EMAIL PROTECTED]
X-MIME-Autoconverted: from quoted-printable to 8bit by 
sucuri.mat.puc-rio.br id HAA07410
Sender: [EMAIL PROTECTED]
Precedence: bulk
Return-Path: [EMAIL PROTECTED]
X-OriginalArrivalTime: 23 Oct 2003 10:11:53.0677 (UTC) 
FILETIME=[151607D0:01C3994E]

Bom, por um lado eh verdade que 1/3  (2+sen(n))/3  1, para todo n natural
e que, portanto, o termo geral tende a zero.
Alem disso, a primeira vista, o n-esimo termo seria majorado por algo da
forma a^n/n, com 1/3  a  1, o que implicaria em convergencia. Soh que 
isso
nao eh verdade. Aquela nossa discussao sobre densidade e equidistribuicao
mostrou que a sequencia cujo termo geral eh (2+sen(n))/3 tem uma
subsequencia convergindo pra 1 (apesar de (2+sen(n))/3 nao ser UD em 
[1/2,1]
- acho que o Gugu mandou uma mensagem mostrando isso ou algo muito 
similar).
Isso significa que nao existe a em (1/3,1) tal que a majoracao mencionada
acima ocorre para todo n.

Claro, o problema estah no expoente n (sem ele, cada termo teria 1/(3n) 
como
cota inferior e a serie seria divergente por comparacao com a serie
harmonica).

A serie SOMA(n=1) ((2+sen(n))/3)^n tambem me parece problematica, pois nao
pode ser majorada por nenhuma serie geometrica com razao  1. Nesse caso,
nao tenho nem certeza sobre se o termo geral tende a zero.
Talvez seja interessante considerar a serie SOMA(n=1) x(n)^n, onde x(n) eh
uma sequencia UD em [0,1]. Uma duvida que eu tenho eh se o expoente n puxa
os termos da serie pra baixo o suficiente pra que ele convirja. Serah que,
pelo menos, lim x(n)^n = 0 ?
Espero que alguem com mais conhecimentos entre na discussao e tire essas
duvidas.
Um abraco,
Claudio.
on 23.10.03 00:57, Eduardo Casagrande Stabel at [EMAIL PROTECTED] wrote:

 Oi Cláudio!

 Não sei a resposta. Eu deveria ter dito mais sobre o problema quando fiz 
a
 pergunta. Pelo que ouvi dizer, este é um problema que um professor 
copiou
 mal de um livro e propôs

Re: [obm-l] SOMA(n=1) (1/n)*((2+sen(n))/3)^n

2003-10-23 Por tôpico Paulo Santa Rita
Oi Claudio,

Infelizmente, sua observacao nao e consistente.

Voce se ateve a uma frase ( que destacou pra refutar ) e nao ao corpo da 
mensagem. Esta claro, devido a tudo que escrevi, que eu me refiri aos r(N) 
que resultam da equacao ( perdao se nao fui suficientemente claro ! ) :

(1/N)*[ (2+sen(N) / 3)]^N = 1/[ N^r(N) ] , r(N)   1 para qualquer N = 1. A 
sequencia Xn=sen(N) varia entre -1 e 1, isto e, -1  sen(N)  1.

NESTE CASO, volto a afirmar : a serie converge DEVIDO A PARTICULARIDADE DO 
sen(N) que estou destacando nesta mensagem. E necessario desenvolver mais 
este ponto ...

Um abraco a todos
Paulo Santa Rita
5,1609,231003
From: Cláudio \(Prática\) [EMAIL PROTECTED]
Reply-To: [EMAIL PROTECTED]
To: [EMAIL PROTECTED]
Subject: Re: [obm-l] SOMA(n=1) (1/n)*((2+sen(n))/3)^n
Date: Thu, 23 Oct 2003 15:29:02 -0200
MIME-Version: 1.0
Received: from mc6-f7.hotmail.com ([65.54.252.143]) by mc6-s20.hotmail.com 
with Microsoft SMTPSVC(5.0.2195.5600); Thu, 23 Oct 2003 10:35:30 -0700
Received: from sucuri.mat.puc-rio.br ([139.82.27.7]) by mc6-f7.hotmail.com 
with Microsoft SMTPSVC(5.0.2195.5600); Thu, 23 Oct 2003 10:29:01 -0700
Received: (from [EMAIL PROTECTED])by sucuri.mat.puc-rio.br (8.9.3/8.9.3) 
id PAA19598for obm-l-MTTP; Thu, 23 Oct 2003 15:22:50 -0200
Received: from ns3bind.bindtech.com.br ([200.230.34.5])by 
sucuri.mat.puc-rio.br (8.9.3/8.9.3) with ESMTP id OAA19594for 
[EMAIL PROTECTED]; Thu, 23 Oct 2003 14:22:49 -0300
Received: from servico2 ([200.230.34.229])by ns3bind.bindtech.com.br 
(8.11.6/X.XX.X) with SMTP id h9NHLfI10536for [EMAIL PROTECTED]; Thu, 
23 Oct 2003 15:21:42 -0200
X-Message-Info: vGzX0e+ktu7ypwUeYQqq1iuL4jIELQokqOpMWY8YQ5o=
Message-ID: [EMAIL PROTECTED]
References: [EMAIL PROTECTED]
X-Priority: 3
X-MSMail-Priority: Normal
X-Mailer: Microsoft Outlook Express 6.00.2600.
X-MimeOLE: Produced By Microsoft MimeOLE V6.00.2600.
Sender: [EMAIL PROTECTED]
Precedence: bulk
Return-Path: [EMAIL PROTECTED]
X-OriginalArrivalTime: 23 Oct 2003 17:29:03.0500 (UTC) 
FILETIME=[274764C0:01C3998B]


 S = somatorio(1 ate +INF) de i^[ - r(i) ] , r( i )  1, converge ? Para
mim,
 e evidente que sim.
Oi, Paulo:

Infelizmente isso não é verdade.
Por exemplo, para cada n = 3, tome r(n) = 1 + ln(ln(n))/ln(n)  1.
Isso resulta em n^r(n) = n*ln(n) ==
SOMA(n=3) n^(-r(n)) =  SOMA(n =3) 1/(n*ln(n)), que diverge, pelo teste da
integral.
*

O problema do Duda parece ser bem mais complicado.
Por exemplo, um bom começo seria determinar se a sequência x(n) = sen(n)^n 
é
convergente ou não.

Um abraço,
Claudio.
=
Instruções para entrar na lista, sair da lista e usar a lista em
http://www.mat.puc-rio.br/~nicolau/olimp/obm-l.html
=
_
MSN Messenger: converse com os seus amigos online.  
http://messenger.msn.com.br

=
Instruções para entrar na lista, sair da lista e usar a lista em
http://www.mat.puc-rio.br/~nicolau/olimp/obm-l.html
=


Re: [obm-l] SOMA(n=1) (1/n)*((2+sen(n))/3)^n

2003-10-24 Por tôpico Paulo Santa Rita
Oi Claudio,

Gostaria de poder participar com mais tranquilidade, como fazia antes, mas 
podes crer que o meu dia precisaria ter cerca 48 horas para que eu pudesse 
atender bem tantas coisas ...

Eu continuo achando que e evidente que a serie converge devido as 
particularidades Xn = sen(N) e considero que o que eu vejo e muito 
semelhante a convergencia uniforme que se usa, dentre outros lugares, na 
Analise de Fourier. Vou tentar falar um pouco mais sobre isso.

Nos sabemos que os r(N) sao definidos pela equacao :

(1/N)*{ [ (2 + sen(N) / 3 ]^N } = 1/( N^r(N) )=r(N)  1 para todo N

A serie : 1 + 1/( 2^r(2) ) + ... + 1/( N^r(N) ) + ... evidentemente que 
converge pois r(N)  1 e tomando r=MIN { r(1), r(2), ... } claramente que 
1/N^r converge e 1/N^r = 1/N^r(N). Tudo isso parace com a convergencia 
uniforme ( devido a particularidade de Xn = sen(N) )

Eu deveria ter detalhado mais aqui para que meu raciocinio ficasse mais 
claro ...

Seja r um real em (-1,1). Tome um E (epsilon) suficientemente pequeno. Entao 
(r-E,r+E) sera suficientemente pequeno e claramente ainda teremos 
I=(r-E,r+E) contido em (-1,1). Pois bem :
Se o natural N1 e tal que sen(N1) esta em I entao o proximo natural N2 ( da 
sequencia Xn=sen(N) ) que cair em I sera tal que :

1  r(N1)  r(N2)

reiterando, teriamos : r(N1)  r(N2)  ...  r(Ni)  ou seja, a subsequencia 
que cai dentro de I e linitada superiomente por 1/( N1 ^r(N1) ). Como 
claramente somatorio de 1/N^r(N1) converge entao
evidentemente a serie correspondente converge.

O QUE ME PARECE EVIDENTE e que e possivel decompor (-1,1) numa quantidade 
finita de intervalos suficientemente pequenos de forma que todos os termos 
da serie que caem dentro de um intervalo especifico da decomposicao vao 
atender o detalhe que expliquei acima. Como todas as series vao convergir e 
sao em numero finito ( pois os intervalos sao em numero finito ) entao toda 
a serie vai convergir...

Assim, a convergencia me pareceu evidente por causa desta evidente 
particularidade de Xn = sen(N)

Observe que poderemos usar UM MESMO E( epsilon ) suficientemente pequeno 
para todos os intervalos e, por esta razao, eu achei uma certa semelhanca 
com a convergencia uniforme. Por isso eu disse : vejo uma semelhanca com a 
convergencia uniforme usada em analise de fourier.

Se a minha sensibilidade estiver errada, e no pressuposto expresso neste 
ultimo paragrafo. Mas eu nao sinto isso ...

Esta a ideia que me ocorreu numa analise rapida. E claramente necessario 
provar alguns passos elementares para transformar a ideia numa demonstracao, 
mas voce vai concordar comigo que visualizar estas coisas e muito facil.

Um Abraco a Todos
Um abraco especial ao Duda e ao Claudio
Paulo Santa Rita
6,1040,241003
From: Claudio Buffara [EMAIL PROTECTED]
Reply-To: [EMAIL PROTECTED]
To: [EMAIL PROTECTED]
Subject: Re: [obm-l] SOMA(n=1) (1/n)*((2+sen(n))/3)^n
Date: Thu, 23 Oct 2003 20:02:02 -0200
MIME-Version: 1.0
Received: from mc5-f27.hotmail.com ([65.54.252.34]) by mc5-s2.hotmail.com 
with Microsoft SMTPSVC(5.0.2195.5600); Thu, 23 Oct 2003 15:42:03 -0700
Received: from sucuri.mat.puc-rio.br ([139.82.27.7]) by mc5-f27.hotmail.com 
with Microsoft SMTPSVC(5.0.2195.5600); Thu, 23 Oct 2003 15:03:16 -0700
Received: (from [EMAIL PROTECTED])by sucuri.mat.puc-rio.br (8.9.3/8.9.3) 
id TAA32527for obm-l-MTTP; Thu, 23 Oct 2003 19:59:47 -0200
Received: from paiol.terra.com.br (paiol.terra.com.br [200.176.3.18])by 
sucuri.mat.puc-rio.br (8.9.3/8.9.3) with ESMTP id SAA32523for 
[EMAIL PROTECTED]; Thu, 23 Oct 2003 18:59:46 -0300
Received: from araci.terra.com.br (araci.terra.com.br [200.176.3.44])by 
paiol.terra.com.br (Postfix) with ESMTP id 677C08480A9for 
[EMAIL PROTECTED]; Thu, 23 Oct 2003 19:59:16 -0200 (BRST)
Received: from [200.182.232.19] (23219.virtua.com.br 
[200.182.232.19])(authenticated user claudio.buffara)by araci.terra.com.br 
(Postfix) with ESMTP id 22E4921EF4Dfor [EMAIL PROTECTED]; Thu, 23 Oct 
2003 19:59:12 -0200 (BRST)
X-Message-Info: vGzX0e+ktu4erC0zmrqvkg59w/93sa7bES3uBHMVtSI=
User-Agent: Microsoft-Outlook-Express-Macintosh-Edition/5.02.2022
Message-ID: [EMAIL PROTECTED]
In-Reply-To: [EMAIL PROTECTED]
X-MIME-Autoconverted: from quoted-printable to 8bit by 
sucuri.mat.puc-rio.br id SAA32524
Sender: [EMAIL PROTECTED]
Precedence: bulk
Return-Path: [EMAIL PROTECTED]
X-OriginalArrivalTime: 23 Oct 2003 22:03:17.0991 (UTC) 
FILETIME=[76EB7770:01C399B1]

Oi, Paulo:

Mesmo assim eu nao estou convencido. O que esse r(n) (relativo a equacao do
Duda) tem de especial que faz a serie convergir? O meu exemplo anterior
mostra que o simples fato de termos r(n)  1 para todo n nao eh suficiente.
Tudo bem. Concordo que -1  sen(n)  1 implica 1/3  (2+sen(n))/3  1 e 
que,
portanto, o n-esimo termo da serie eh menor do que 1/n, mas dai a concluir
que ela eh convergente...

Mas talvez eu tenha entendido mal: a que particularidade de sen(n) voce se
refere?
Uma duvida mais basica: A sequencia x(n) = ((2+sen(n))/3)^n converge?

Um abraco

[obm-l] SOMA(n=1) (1/n)*((2+sen(n))/3)^n - correcao

2003-10-24 Por tôpico Paulo Santa Rita
Ola Pessoal !

Na mensagem anterior, escrevi errado. O correto e :

r(N1)  r(N2)  r(N3)  ...  r(Ni)  ...
O que da (na sub-serie do intervalo I ) : 1/( N^r(N1) ) convergindo e 1/( 
Ni^r(N1) )1/(Ni^r(Ni))

Um Abraco
Paulo Santa Rita
6,1128,241003
From: Claudio Buffara [EMAIL PROTECTED]
Reply-To: [EMAIL PROTECTED]
To: [EMAIL PROTECTED]
Subject: Re: [obm-l] SOMA(n=1) (1/n)*((2+sen(n))/3)^n
Date: Thu, 23 Oct 2003 20:02:02 -0200
MIME-Version: 1.0
Received: from mc5-f27.hotmail.com ([65.54.252.34]) by mc5-s2.hotmail.com 
with Microsoft SMTPSVC(5.0.2195.5600); Thu, 23 Oct 2003 15:42:03 -0700
Received: from sucuri.mat.puc-rio.br ([139.82.27.7]) by mc5-f27.hotmail.com 
with Microsoft SMTPSVC(5.0.2195.5600); Thu, 23 Oct 2003 15:03:16 -0700
Received: (from [EMAIL PROTECTED])by sucuri.mat.puc-rio.br (8.9.3/8.9.3) 
id TAA32527for obm-l-MTTP; Thu, 23 Oct 2003 19:59:47 -0200
Received: from paiol.terra.com.br (paiol.terra.com.br [200.176.3.18])by 
sucuri.mat.puc-rio.br (8.9.3/8.9.3) with ESMTP id SAA32523for 
[EMAIL PROTECTED]; Thu, 23 Oct 2003 18:59:46 -0300
Received: from araci.terra.com.br (araci.terra.com.br [200.176.3.44])by 
paiol.terra.com.br (Postfix) with ESMTP id 677C08480A9for 
[EMAIL PROTECTED]; Thu, 23 Oct 2003 19:59:16 -0200 (BRST)
Received: from [200.182.232.19] (23219.virtua.com.br 
[200.182.232.19])(authenticated user claudio.buffara)by araci.terra.com.br 
(Postfix) with ESMTP id 22E4921EF4Dfor [EMAIL PROTECTED]; Thu, 23 Oct 
2003 19:59:12 -0200 (BRST)
X-Message-Info: vGzX0e+ktu4erC0zmrqvkg59w/93sa7bES3uBHMVtSI=
User-Agent: Microsoft-Outlook-Express-Macintosh-Edition/5.02.2022
Message-ID: [EMAIL PROTECTED]
In-Reply-To: [EMAIL PROTECTED]
X-MIME-Autoconverted: from quoted-printable to 8bit by 
sucuri.mat.puc-rio.br id SAA32524
Sender: [EMAIL PROTECTED]
Precedence: bulk
Return-Path: [EMAIL PROTECTED]
X-OriginalArrivalTime: 23 Oct 2003 22:03:17.0991 (UTC) 
FILETIME=[76EB7770:01C399B1]

Oi, Paulo:

Mesmo assim eu nao estou convencido. O que esse r(n) (relativo a equacao do
Duda) tem de especial que faz a serie convergir? O meu exemplo anterior
mostra que o simples fato de termos r(n)  1 para todo n nao eh suficiente.
Tudo bem. Concordo que -1  sen(n)  1 implica 1/3  (2+sen(n))/3  1 e 
que,
portanto, o n-esimo termo da serie eh menor do que 1/n, mas dai a concluir
que ela eh convergente...

Mas talvez eu tenha entendido mal: a que particularidade de sen(n) voce se
refere?
Uma duvida mais basica: A sequencia x(n) = ((2+sen(n))/3)^n converge?

Um abraco,
Claudio.
on 23.10.03 16:08, Paulo Santa Rita at [EMAIL PROTECTED] wrote:

 Oi Claudio,

 Infelizmente, sua observacao nao e consistente.

 Voce se ateve a uma frase ( que destacou pra refutar ) e nao ao corpo da
 mensagem. Esta claro, devido a tudo que escrevi, que eu me refiri aos 
r(N)
 que resultam da equacao ( perdao se nao fui suficientemente claro ! ) :

 (1/N)*[ (2+sen(N) / 3)]^N = 1/[ N^r(N) ] , r(N)   1 para qualquer N = 
1. A
 sequencia Xn=sen(N) varia entre -1 e 1, isto e, -1  sen(N)  1.

 NESTE CASO, volto a afirmar : a serie converge DEVIDO A PARTICULARIDADE 
DO
 sen(N) que estou destacando nesta mensagem. E necessario desenvolver 
mais
 este ponto ...

 Um abraco a todos
 Paulo Santa Rita
 5,1609,231003

 From: Cláudio \(Prática\) [EMAIL PROTECTED]
 Reply-To: [EMAIL PROTECTED]
 To: [EMAIL PROTECTED]
 Subject: Re: [obm-l] SOMA(n=1) (1/n)*((2+sen(n))/3)^n
 Date: Thu, 23 Oct 2003 15:29:02 -0200


 S = somatorio(1 ate +INF) de i^[ - r(i) ] , r( i )  1, converge ? 
Para
 mim,
 e evidente que sim.

 Oi, Paulo:

 Infelizmente isso não é verdade.
 Por exemplo, para cada n = 3, tome r(n) = 1 + ln(ln(n))/ln(n)  1.
 Isso resulta em n^r(n) = n*ln(n) ==
 SOMA(n=3) n^(-r(n)) =  SOMA(n =3) 1/(n*ln(n)), que diverge, pelo 
teste da
 integral.

 *

 O problema do Duda parece ser bem mais complicado.
 Por exemplo, um bom começo seria determinar se a sequência x(n) = 
sen(n)^n
 é
 convergente ou não.

 Um abraço,
 Claudio.




=
Instruções para entrar na lista, sair da lista e usar a lista em
http://www.mat.puc-rio.br/~nicolau/olimp/obm-l.html
=
_
MSN Messenger: converse com os seus amigos online.  
http://messenger.msn.com.br

=
Instruções para entrar na lista, sair da lista e usar a lista em
http://www.mat.puc-rio.br/~nicolau/olimp/obm-l.html
=


Re: [obm-l] SOMA(n=1) (1/n)*((2+sen(n))/3)^n

2003-10-26 Por tôpico Paulo Santa Rita
Ola Claudio e demais
colegas desta lista ... OBM-L,
Peco desculpas a todos por demorara a responder.

O correto seria eu parar e pensar na questao com seriedade, para entao 
apresentar uma demonstracao formal ou me convencer que estou enganado, mas 
no momento nao da pra mim fazer isso. O que fundamenta a minha intuicao e o 
seguinte :

Como An = (1/N)*[ ( 2+sen(N) )/3]^N = 1/( N^r(N) ) e sendo claro que  An  
1/N e igualmente claro que R(N)  1. Mas e igualmente claro que tomando um 
intervalo BEM PEQUENO em (-1,1) os sucessivos sen(N) que caem nele, por 
estarem muito proximo, AFETAM MUITO POUCO a parcela  sen(N) de :

[ ( 2+sen(N) )/3]^N

Assim, para um intervalo de (-1,1) BEM PEQUENO, se N1 e N2 sao dois indices 
consecutivos de
Xn=sen(N) que cai no intervalo :

(1/N1)*[ ( 2+sen(N1) )/3]^N1  (1/N2)*[ ( 2+sen(N2) )/3]^N2  = 1/[N1^r(N1)] 
 1/[N2^r(N2)]

Evidentemente que isso vale para quaisquer dois termos consecutivos da 
subsequencia que cai no intervalo, isto :

1/[N1^r(N1)]  1/[N2^r(N2)]   1/[N3^r(N3)]   ...

Ou seja, a sub-sequencia e monotona decrescente. Observe que os r(N i) nao 
sao so MAIORES QUE UM eles  podem ser MUITO MAIORES QUE UM ... se Ni  3 
entao 1/Ni  ( 2+sen(N1) )/3 dai :

1/[ N1^( r(N1)+1 ) ]  (1/N1)*[ ( 2+sen(N1) )/3 ]=1/( N1^r(N1) ) = r(N1)  
N1 + 1
como N2  N1 entao 1/[ N2^( r(N1)+1 ) ]  1/[ N1^( r(N1)+1 ) ], podendo 
r(N2)  N2 + 1

Evidentemente que os r(Ni) crescem na medida que o pequeno subintervalo se 
aproxima do extremo
-1 do intervalo (-1,1). Ocorre que o subintervalo e bem pequeno, isto e, 
Ni+1  Ni, isto e, os fatores a mais que N1 ( N2 - N1 ) em :

(1/N2)*[ ( 2+sen(N2) )/3]^N2 =(1/N2)*{ [ ( 2+sen(N2) )/3]^N1 }*{ [ ( 
2+sen(N2) )/3]^(N2-N1) }=
(1/N1)*{ [ ( 2+sen(N2) )/3]^N1 }*(N1/N2)*{ [ ( 2+sen(N2) )/3]^(N2-N1) }

isto e : fatores excedentes ... (N1/N2)*{ [ ( 2+sen(N2) )/3]^(N2-N1) } sao 
tais que :

1/[ N2^r(N2) ]  1/[ N2^( r(N1)+1 ) ]  1/[ N1^( r(N1)+1 ) ]  1/( N1^r(N1) 
)
r(N2)   r(N1) + 1r(N1)   =   r(N1)  r(N2)

O que acho intuitivo ( mas que precisa ser provado ) e que sendo o intervalo 
bem pequeno e devidamente ajustado, o proximo Ni+1 apos o Ni e muito 
maior que o Ni, o excesso obrigando
r(Ni)  r(Ni+1)

Evidentemente que isto nao e uma prova, apenas o fruto de uma analise rapida 
derivada da intuicao e que reforca a minha crenca na convergencia. Eu 
deveria aqui prova algo assim ...

se N2  N1 e  modulo(N1-N2)  E, E positivo e suficientemente pequeno, entao 
N2  N1 + i, para algum i natural. Dai mostra que o excesso que falei 
(N2-N1i) cumpre de fato o que a minha intuicao esta me dizendo, isto e, que 
r(N1)  r(N2).

Se for assim, entao para cada intervalo teremos uma serie :

1/[N1^r(N1)]  1/[N2^r(N2)]   1/[N3^r(N3)]   ...
com r(N1)  r(N2)  r(N3)  ... e
e a serie evidentemente converge. E portanto a serie toda converge.

Bom, eu acho que detalhei bem o que vi numa primeira olhada. Mas estou sem 
tempo para pensar seriamente na questao. Todavia, considero praticamente 
certo que outro membro que pense seriamente sobre a questao vai achar um 
caminho sem maiores dificuldades ( engracado, neste exato momento que estou 
escrevendo estas linhas me ocorreu outra ideia. )

O problema e legal e de qualidade, como tudo que vem do Duda. Ele nao se 
dobrou diante de nenhuma investida trivial, tais como os teste de banais de 
convergencia e os teoremas que se aprende num bom curso de analise. Esta 
assim em consonancia com a proposta original desta desta nossa lista, que e 
discutir problemas olimpicos e praticar a verdadeira Matematica, alta e 
bela.

Um Abraco a Todos !
Paulo Santa Rita
1,2126,261003
From: Claudio Buffara [EMAIL PROTECTED]
Reply-To: [EMAIL PROTECTED]
To: [EMAIL PROTECTED]
Subject: Re: [obm-l] SOMA(n=1) (1/n)*((2+sen(n))/3)^n
Date: Fri, 24 Oct 2003 13:45:18 -0200
MIME-Version: 1.0
Received: from mc5-f16.hotmail.com ([65.54.252.23]) by mc5-s3.hotmail.com 
with Microsoft SMTPSVC(5.0.2195.5600); Fri, 24 Oct 2003 08:50:24 -0700
Received: from sucuri.mat.puc-rio.br ([139.82.27.7]) by mc5-f16.hotmail.com 
with Microsoft SMTPSVC(5.0.2195.5600); Fri, 24 Oct 2003 08:45:19 -0700
Received: (from [EMAIL PROTECTED])by sucuri.mat.puc-rio.br (8.9.3/8.9.3) 
id NAA08038for obm-l-MTTP; Fri, 24 Oct 2003 13:43:02 -0200
Received: from ivoti.terra.com.br (ivoti.terra.com.br [200.176.3.20])by 
sucuri.mat.puc-rio.br (8.9.3/8.9.3) with ESMTP id MAA08034for 
[EMAIL PROTECTED]; Fri, 24 Oct 2003 12:43:01 -0300
Received: from marova.terra.com.br (marova.terra.com.br [200.176.3.39])by 
ivoti.terra.com.br (Postfix) with ESMTP id 9962D79C1F0for 
[EMAIL PROTECTED]; Fri, 24 Oct 2003 13:42:30 -0200 (BRST)
Received: from [200.182.232.19] (23219.virtua.com.br 
[200.182.232.19])(authenticated user claudio.buffara)by marova.terra.com.br 
(Postfix) with ESMTP id B61563DC12Afor [EMAIL PROTECTED]; Fri, 24 Oct 
2003 13:42:29 -0200 (BRST)
X-Message-Info: vGzX0e+ktu63y9sk04jGZSfa9j6I2+MIsty5ondVrzI=
User-Agent: Microsoft-Outlook-Express-Macintosh-Edition/5.02.2022
Message-ID: [EMAIL

Re: [obm-l] a^3+b^3+c^3 = 3abc

2003-11-05 Por tôpico Paulo Santa Rita
Ola Daniel e demais
colegas desta lista ... OBM-L,
Muito legal a prova por determinante. Vou tentar produzir uma prova 
diferente :

a + b + c= 0  =  a + b = -c  = (a+b)^3 = (-c)^3
a^3  + 3(a^2)b  + 3a(b^2)  + b^3 = -c^3
a^3 + b^3 + c^3 =  -3(a^2)b - 3a(b^2)
a^3 + b^3 + c^3 =  -3ab(a + b)
como a + b = -c :
a^3 + b^3 + c^3 =  -3ab(-c)  = a^3 + b^3 + c^3 - 3abc = 0
um outro legal, bem simples,  na mesma linha de raciocinio :

Se A + B  CeD + E  F  entao ( A,B,C,D,E,F sao reais positivos )  :

raiz_quad(A^2 + E^2) + raiz_quad(B^2 + D^2)  raiz_quad(C^2 + F^2)



From: Daniel Faria [EMAIL PROTECTED]
Reply-To: [EMAIL PROTECTED]
To: [EMAIL PROTECTED]
Subject: Re: [obm-l] a^3+b^3+c^3 = 3abc
Date: Wed, 05 Nov 2003 03:06:56 -0200
MIME-Version: 1.0
X-Originating-IP: [200.173.170.47]
X-Originating-Email: [EMAIL PROTECTED]
Received: from mc2-f38.hotmail.com ([65.54.237.45]) by mc2-s6.hotmail.com 
with Microsoft SMTPSVC(5.0.2195.5600); Tue, 4 Nov 2003 21:10:26 -0800
Received: from sucuri.mat.puc-rio.br ([139.82.27.7]) by mc2-f38.hotmail.com 
with Microsoft SMTPSVC(5.0.2195.5600); Tue, 4 Nov 2003 21:09:12 -0800
Received: (from [EMAIL PROTECTED])by sucuri.mat.puc-rio.br (8.9.3/8.9.3) 
id DAA29324for obm-l-MTTP; Wed, 5 Nov 2003 03:07:29 -0200
Received: from hotmail.com (bay8-f8.bay8.hotmail.com [64.4.27.8])by 
sucuri.mat.puc-rio.br (8.9.3/8.9.3) with ESMTP id CAA29320for 
[EMAIL PROTECTED]; Wed, 5 Nov 2003 02:07:28 -0300
Received: from mail pickup service by hotmail.com with Microsoft SMTPSVC; 
Tue, 4 Nov 2003 21:06:56 -0800
Received: from 200.173.170.47 by by8fd.bay8.hotmail.msn.com with HTTP;Wed, 
05 Nov 2003 05:06:56 GMT
X-Message-Info: HQbIehuYceSItGGrcCSeWiDHpA256mgG71rW/dGB/hs=
Message-ID: [EMAIL PROTECTED]
X-OriginalArrivalTime: 05 Nov 2003 05:06:56.0697 (UTC) 
FILETIME=[A29B3290:01C3A35A]
Sender: [EMAIL PROTECTED]
Precedence: bulk
Return-Path: [EMAIL PROTECTED]

Gostaria de tentar uma resoluçao sobre o enunciado, só que fazendo um 
caminho inverso:

Dado   a+b+c=0,
quero chegar em
a^3 + b^3 + c^3 - 3abc = 0.

Partindo de:

a^3 + b^3 + c^3 - 3abc

Farei a linha acima por determinante:

a b c
c a b
b c a
A soma de cada linha deste deteminante eh  a+b+c que como jah eh sabido eh 
zero.

logo o determinante acima eh igual a zero.

Assim temos:

a^3 + b^3 + c^3 - 3abc = 0

e

a^3 + b^3 + c^3 = 3abc

Por favor me corrijam se eu estiver errado.

Obrigado.


-- Mensagem original --

Ola pessoal,

Depois de alguns meses afastado da lista e sem estudar matematica, pois
estava estudando para um concurso e acabei de faze-lo. Agora eh esperar
ansioso

pelo resultado que sairah em menos de 2 semanas. Para nao ficar 
off-topic
vou
re-comecar a postar minhas duvidas. Vamos la:

1) Prove que se a + b + c = 0, entao a^3 + b^3 + c^3 = 3abc

Obs: Como estou voltando agora, desculpem me se o problema for trivial.

Preciso me desenferrujar aos poucos ;-) em matematica e pegar o ritmo de
novo.





--
Use o melhor sistema de busca da Internet
Radar UOL - http://www.radaruol.com.br


=
Instruções para entrar na lista, sair da lista e usar a lista em
http://www.mat.puc-rio.br/~nicolau/olimp/obm-l.html
=
_
MSN Hotmail, o maior webmail do Brasil.  http://www.hotmail.com
=
Instruções para entrar na lista, sair da lista e usar a lista em
http://www.mat.puc-rio.br/~nicolau/olimp/obm-l.html
=
_
MSN Hotmail, o maior webmail do Brasil.  http://www.hotmail.com
=
Instruções para entrar na lista, sair da lista e usar a lista em
http://www.mat.puc-rio.br/~nicolau/olimp/obm-l.html
=


[obm-l] Prova do IME

2003-11-05 Por tôpico Paulo Santa Rita
Ola Pessoal !

Todos devem ter notado que novamente estamos sendo agredidos por um imbecil 
qualquer. Nao se responde a este tipo de gente. Vamos ignorar as mensagens 
ofensivas e trata-las como sao : lixo produzido por lixo.

Se nao me engano, ontem foi a prova de Matematica do IME. Alguem tem a prova 
ou sabe onde encontra-la na Internet ? Seria interessante disponibiliza-la 
aqui na nossa lista, para que pudessemos discutir algumas questoes.

Um Abraco a Todos !
Paulo Santa Rita
4,1120,051103
_
MSN Hotmail, o maior webmail do Brasil.  http://www.hotmail.com
=
Instruções para entrar na lista, sair da lista e usar a lista em
http://www.mat.puc-rio.br/~nicolau/olimp/obm-l.html
=


[obm-l] Um site interessante

2003-11-05 Por tôpico Paulo Santa Rita
Ola Pessoal !

No Site :
http://turnbull.mcs.st-and.ac.uk/
Click em : MacTutor history of Mathematics Archive

La existem muitas informaçoes interessantes, tais como um aruivo de curvas 
notaveis e os Principais Matematicos ( com omissoes ! ) por paises. Vale a 
pena dar uma olhada.

Um Abraco a Todos
Paulo Santa Rita
4,1139,051103
From: Paulo Santa Rita [EMAIL PROTECTED]
Reply-To: [EMAIL PROTECTED]
To: [EMAIL PROTECTED]
Subject: [obm-l] Prova do IME
Date: Wed, 05 Nov 2003 13:20:15 +
MIME-Version: 1.0
X-Originating-IP: [200.142.58.18]
X-Originating-Email: [EMAIL PROTECTED]
Received: from mc2-f20.hotmail.com ([65.54.237.27]) by mc2-s16.hotmail.com 
with Microsoft SMTPSVC(5.0.2195.5600); Wed, 5 Nov 2003 05:28:10 -0800
Received: from sucuri.mat.puc-rio.br ([139.82.27.7]) by mc2-f20.hotmail.com 
with Microsoft SMTPSVC(5.0.2195.5600); Wed, 5 Nov 2003 05:24:06 -0800
Received: (from [EMAIL PROTECTED])by sucuri.mat.puc-rio.br (8.9.3/8.9.3) 
id LAA04832for obm-l-MTTP; Wed, 5 Nov 2003 11:20:57 -0200
Received: from hotmail.com (sea2-f50.sea2.hotmail.com [207.68.165.50])by 
sucuri.mat.puc-rio.br (8.9.3/8.9.3) with ESMTP id LAA04828for 
[EMAIL PROTECTED]; Wed, 5 Nov 2003 11:20:52 -0200
Received: from mail pickup service by hotmail.com with Microsoft SMTPSVC; 
Wed, 5 Nov 2003 05:20:17 -0800
Received: from 200.142.58.18 by sea2fd.sea2.hotmail.msn.com with HTTP;Wed, 
05 Nov 2003 13:20:15 GMT
X-Message-Info: HQbIehuYceT6VeKGoNWA63UYML5cWlLPc1N/atNLXfQ=
Message-ID: [EMAIL PROTECTED]
X-OriginalArrivalTime: 05 Nov 2003 13:20:17.0135 (UTC) 
FILETIME=[8DD797F0:01C3A39F]
Sender: [EMAIL PROTECTED]
Precedence: bulk
Return-Path: [EMAIL PROTECTED]

Ola Pessoal !

Todos devem ter notado que novamente estamos sendo agredidos por um imbecil 
qualquer. Nao se responde a este tipo de gente. Vamos ignorar as mensagens 
ofensivas e trata-las como sao : lixo produzido por lixo.

Se nao me engano, ontem foi a prova de Matematica do IME. Alguem tem a 
prova ou sabe onde encontra-la na Internet ? Seria interessante 
disponibiliza-la aqui na nossa lista, para que pudessemos discutir algumas 
questoes.

Um Abraco a Todos !
Paulo Santa Rita
4,1120,051103
_
MSN Hotmail, o maior webmail do Brasil.  http://www.hotmail.com
=
Instruções para entrar na lista, sair da lista e usar a lista em
http://www.mat.puc-rio.br/~nicolau/olimp/obm-l.html
=
_
MSN Hotmail, o maior webmail do Brasil.  http://www.hotmail.com
=
Instruções para entrar na lista, sair da lista e usar a lista em
http://www.mat.puc-rio.br/~nicolau/olimp/obm-l.html
=


Re: [obm-l] Prova do IME

2003-11-05 Por tôpico Paulo Santa Rita
Ola Prof Morgado e demais
colegas desta lista ... OBM-L,
Deu uma olhada no site do GPI. A prova esta la, questao por questao. Mas ... 
FEITA ! Que pena, nao vamos ter a alegria de descobrir as solucoes. Mas eu 
proponho o seguinte :

Vamos encontrar, pra cada questao, uma maneira diferente de fazer (  mesmo 
que seja mais feia ou longa ) ? Eu começo :

1 QUESTAO ) Existe uma regra, chamada regra de Chio, que permite abaixar a 
ordem de uma matriz. Basta que A11=1 ( se nao me falha a memoria ). Entao, 
aplicando a regra de Chio duas vezes vamos cair numa matriz 2x2, cujo 
determinante e facil calcular.

Calculando o determinante em funcao de N, igualamos a 5 e resolvemos a 
equacao.

Quem faz a questao 2, com solucao diferente da do GPI ? Para que todos 
possam participar, voces aceitam que uma pessoa so possa fazer uma questao ( 
nao duas ou mais ) ?

Um Abraco a Todos !
Paulo Santa Rita
4,1403,051103




From: Augusto Cesar de Oliveira Morgado [EMAIL PROTECTED]
Reply-To: [EMAIL PROTECTED]
To: [EMAIL PROTECTED]
Subject: Re: [obm-l] Prova do IME
Date: Wed, 5 Nov 2003 14:22:54 -0200
MIME-Version: 1.0
Received: from mc2-f14.hotmail.com ([65.54.237.21]) by mc2-s17.hotmail.com 
with Microsoft SMTPSVC(5.0.2195.5600); Wed, 5 Nov 2003 07:25:29 -0800
Received: from sucuri.mat.puc-rio.br ([139.82.27.7]) by mc2-f14.hotmail.com 
with Microsoft SMTPSVC(5.0.2195.5600); Wed, 5 Nov 2003 07:24:22 -0800
Received: (from [EMAIL PROTECTED])by sucuri.mat.puc-rio.br (8.9.3/8.9.3) 
id NAA07638for obm-l-MTTP; Wed, 5 Nov 2003 13:23:26 -0200
Received: from gorgo.centroin.com.br (gorgo.centroin.com.br 
[200.225.63.128])by sucuri.mat.puc-rio.br (8.9.3/8.9.3) with ESMTP id 
NAA07634for [EMAIL PROTECTED]; Wed, 5 Nov 2003 13:23:25 -0200
Received: from centroin.com.br (trex.centroin.com.br [200.225.63.134])by 
gorgo.centroin.com.br (8.12.10/8.12.9) with ESMTP id hA5FMtad007873for 
[EMAIL PROTECTED]; Wed, 5 Nov 2003 13:22:55 -0200 (EDT)
X-Message-Info: HQbIehuYceSUWy5LlRlpF6fIONaJJTv2iApCFDJ/N8U=
Message-Id: [EMAIL PROTECTED]
In-Reply-To: [EMAIL PROTECTED]
References: [EMAIL PROTECTED]
X-Mailer: CIP WebMail 2.10 experimantal 20030731a
X-OriginatingIP: 200.141.90.78 (morgado)
Sender: [EMAIL PROTECTED]
Precedence: bulk
Return-Path: [EMAIL PROTECTED]
X-OriginalArrivalTime: 05 Nov 2003 15:24:23.0275 (UTC) 
FILETIME=[E41663B0:01C3A3B0]

www.gpi.g12.br

--
CIP WebMAIL - Nova Geração - v. 2.1
CentroIn Internet Providerhttp://www.centroin.com.br
Tel: (21) 2542-4849, (21) 2295-3331  Fax: (21) 2295-2978
Empresa 100% Brasileira - Desde 1992
-- Original Message ---
From: Paulo Santa Rita [EMAIL PROTECTED]
To: [EMAIL PROTECTED]
Sent: Wed, 05 Nov 2003 13:20:15 +
Subject: [obm-l] Prova do IME
 Ola Pessoal !

 Todos devem ter notado que novamente estamos sendo agredidos por um
 imbecil qualquer. Nao se responde a este tipo de gente. Vamos
 ignorar as mensagens ofensivas e trata-las como sao : lixo produzido
 por lixo.

 Se nao me engano, ontem foi a prova de Matematica do IME. Alguem tem
 a prova ou sabe onde encontra-la na Internet ? Seria interessante
 disponibiliza-la aqui na nossa lista, para que pudessemos discutir
 algumas questoes.

 Um Abraco a Todos !
 Paulo Santa Rita
 4,1120,051103

 _
 MSN Hotmail, o maior webmail do Brasil.  http://www.hotmail.com

 
=
 Instruções para entrar na lista, sair da lista e usar a lista em
 http://www.mat.puc-rio.br/~nicolau/olimp/obm-l.html
 
=
--- End of Original Message ---

=
Instruções para entrar na lista, sair da lista e usar a lista em
http://www.mat.puc-rio.br/~nicolau/olimp/obm-l.html
=
_
MSN Messenger: converse com os seus amigos online.  
http://messenger.msn.com.br

=
Instruções para entrar na lista, sair da lista e usar a lista em
http://www.mat.puc-rio.br/~nicolau/olimp/obm-l.html
=


Re: [obm-l] Prova do IME

2003-11-05 Por tôpico Paulo Santa Rita
Ola Pessoal e demais colegas
desta lista ... OBM-L,
Alguem encontrou uma forma de resolver a questao 2 diferente da forma 
apresentada so Site do GPI ?
Eu nao vou fazer por fidelidade a regra que propus, segundo a qual uma 
pessoa so pode fazer uma questao ( diferente da solucao GPI ). Mas vou 
ajudar falando sobre algo que, muito provavelmente,  nem todos os estudantes 
sabem :

Numa equacao da forma : x^3 + ax + b=0 , a expressao : (b/2)^2 + (a/3)^3 e 
chamada DISCRIMINANTE. Prova-se que a equacao so tem tres raizes reais nao 
nulas se o DISCRIMINANTE e negativo ... Dai ...

Um Abraco a Todos
Paulo Santa Rita
4,1509,051103

From: Paulo Santa Rita [EMAIL PROTECTED]
Reply-To: [EMAIL PROTECTED]
To: [EMAIL PROTECTED]
Subject: Re: [obm-l] Prova do IME
Date: Wed, 05 Nov 2003 16:04:30 +
MIME-Version: 1.0
X-Originating-IP: [200.142.58.18]
X-Originating-Email: [EMAIL PROTECTED]
Received: from mc3-f23.hotmail.com ([64.4.50.159]) by mc3-s2.hotmail.com 
with Microsoft SMTPSVC(5.0.2195.6713); Wed, 5 Nov 2003 08:06:32 -0800
Received: from sucuri.mat.puc-rio.br ([139.82.27.7]) by mc3-f23.hotmail.com 
with Microsoft SMTPSVC(5.0.2195.6713); Wed, 5 Nov 2003 08:06:28 -0800
Received: (from [EMAIL PROTECTED])by sucuri.mat.puc-rio.br (8.9.3/8.9.3) 
id OAA08497for obm-l-MTTP; Wed, 5 Nov 2003 14:05:13 -0200
Received: from hotmail.com (sea2-f19.sea2.hotmail.com [207.68.165.19])by 
sucuri.mat.puc-rio.br (8.9.3/8.9.3) with ESMTP id OAA08493for 
[EMAIL PROTECTED]; Wed, 5 Nov 2003 14:05:10 -0200
Received: from mail pickup service by hotmail.com with Microsoft SMTPSVC; 
Wed, 5 Nov 2003 08:04:34 -0800
Received: from 200.142.58.18 by sea2fd.sea2.hotmail.msn.com with HTTP;Wed, 
05 Nov 2003 16:04:30 GMT
X-Message-Info: HQbIehuYceQPI18leHWRVRTadU7O9EmVASR5S6iv19Q=
Message-ID: [EMAIL PROTECTED]
X-OriginalArrivalTime: 05 Nov 2003 16:04:34.0594 (UTC) 
FILETIME=[81587820:01C3A3B6]
Sender: [EMAIL PROTECTED]
Precedence: bulk
Return-Path: [EMAIL PROTECTED]

Ola Prof Morgado e demais
colegas desta lista ... OBM-L,
Deu uma olhada no site do GPI. A prova esta la, questao por questao. Mas 
... FEITA ! Que pena, nao vamos ter a alegria de descobrir as solucoes. Mas 
eu proponho o seguinte :

Vamos encontrar, pra cada questao, uma maneira diferente de fazer (  mesmo 
que seja mais feia ou longa ) ? Eu começo :

1 QUESTAO ) Existe uma regra, chamada regra de Chio, que permite abaixar a 
ordem de uma matriz. Basta que A11=1 ( se nao me falha a memoria ). Entao, 
aplicando a regra de Chio duas vezes vamos cair numa matriz 2x2, cujo 
determinante e facil calcular.

Calculando o determinante em funcao de N, igualamos a 5 e resolvemos a 
equacao.

Quem faz a questao 2, com solucao diferente da do GPI ? Para que todos 
possam participar, voces aceitam que uma pessoa so possa fazer uma questao 
( nao duas ou mais ) ?

Um Abraco a Todos !
Paulo Santa Rita
4,1403,051103




From: Augusto Cesar de Oliveira Morgado [EMAIL PROTECTED]
Reply-To: [EMAIL PROTECTED]
To: [EMAIL PROTECTED]
Subject: Re: [obm-l] Prova do IME
Date: Wed, 5 Nov 2003 14:22:54 -0200
MIME-Version: 1.0
Received: from mc2-f14.hotmail.com ([65.54.237.21]) by mc2-s17.hotmail.com 
with Microsoft SMTPSVC(5.0.2195.5600); Wed, 5 Nov 2003 07:25:29 -0800
Received: from sucuri.mat.puc-rio.br ([139.82.27.7]) by 
mc2-f14.hotmail.com with Microsoft SMTPSVC(5.0.2195.5600); Wed, 5 Nov 2003 
07:24:22 -0800
Received: (from [EMAIL PROTECTED])by sucuri.mat.puc-rio.br (8.9.3/8.9.3) 
id NAA07638for obm-l-MTTP; Wed, 5 Nov 2003 13:23:26 -0200
Received: from gorgo.centroin.com.br (gorgo.centroin.com.br 
[200.225.63.128])by sucuri.mat.puc-rio.br (8.9.3/8.9.3) with ESMTP id 
NAA07634for [EMAIL PROTECTED]; Wed, 5 Nov 2003 13:23:25 -0200
Received: from centroin.com.br (trex.centroin.com.br [200.225.63.134])by 
gorgo.centroin.com.br (8.12.10/8.12.9) with ESMTP id hA5FMtad007873for 
[EMAIL PROTECTED]; Wed, 5 Nov 2003 13:22:55 -0200 (EDT)
X-Message-Info: HQbIehuYceSUWy5LlRlpF6fIONaJJTv2iApCFDJ/N8U=
Message-Id: [EMAIL PROTECTED]
In-Reply-To: [EMAIL PROTECTED]
References: [EMAIL PROTECTED]
X-Mailer: CIP WebMail 2.10 experimantal 20030731a
X-OriginatingIP: 200.141.90.78 (morgado)
Sender: [EMAIL PROTECTED]
Precedence: bulk
Return-Path: [EMAIL PROTECTED]
X-OriginalArrivalTime: 05 Nov 2003 15:24:23.0275 (UTC) 
FILETIME=[E41663B0:01C3A3B0]

www.gpi.g12.br

--
CIP WebMAIL - Nova Geração - v. 2.1
CentroIn Internet Providerhttp://www.centroin.com.br
Tel: (21) 2542-4849, (21) 2295-3331  Fax: (21) 2295-2978
Empresa 100% Brasileira - Desde 1992
-- Original Message ---
From: Paulo Santa Rita [EMAIL PROTECTED]
To: [EMAIL PROTECTED]
Sent: Wed, 05 Nov 2003 13:20:15 +
Subject: [obm-l] Prova do IME
 Ola Pessoal !

 Todos devem ter notado que novamente estamos sendo agredidos por um
 imbecil qualquer. Nao se responde a este tipo de gente. Vamos
 ignorar as mensagens ofensivas e trata-las como sao : lixo produzido
 por lixo.

 Se nao me engano, ontem foi a prova de Matematica do IME. Alguem tem
 a prova ou sabe

Re: [obm-l] Prova do IME

2003-11-05 Por tôpico Paulo Santa Rita
Ola Pessoal !

Vejam que agora ja temos tres solucoes para a questao 2. Quem faz a 3, de 
uma forma diferente da do GPI ? Nao pode ser eu ou o Claudio.

Um Abraco a Todos
Paulo Santa Rita
4,1531,051103
From: Cláudio \(Prática\) [EMAIL PROTECTED]
Reply-To: [EMAIL PROTECTED]
To: [EMAIL PROTECTED]
Subject: Re: [obm-l] Prova do IME
Date: Wed, 5 Nov 2003 15:08:53 -0200
MIME-Version: 1.0
Received: from mc1-f9.hotmail.com ([64.4.50.16]) by mc1-s3.hotmail.com with 
Microsoft SMTPSVC(5.0.2195.6713); Wed, 5 Nov 2003 09:04:07 -0800
Received: from sucuri.mat.puc-rio.br ([139.82.27.7]) by mc1-f9.hotmail.com 
with Microsoft SMTPSVC(5.0.2195.6713); Wed, 5 Nov 2003 09:03:13 -0800
Received: (from [EMAIL PROTECTED])by sucuri.mat.puc-rio.br (8.9.3/8.9.3) 
id PAA09707for obm-l-MTTP; Wed, 5 Nov 2003 15:02:04 -0200
Received: from ns3bind.bindtech.com.br ([200.230.34.5])by 
sucuri.mat.puc-rio.br (8.9.3/8.9.3) with ESMTP id PAA09702for 
[EMAIL PROTECTED]; Wed, 5 Nov 2003 15:02:03 -0200
Received: from servico2 ([200.230.34.224])by ns3bind.bindtech.com.br 
(8.11.6/X.XX.X) with SMTP id hA5H0Xl06494for [EMAIL PROTECTED]; Wed, 5 
Nov 2003 15:00:33 -0200
X-Message-Info: HQbIehuYceTqLXMEyHBvn7Pw6Fl0HXM8zdhH8t2Jk4M=
Message-ID: [EMAIL PROTECTED]
References: [EMAIL PROTECTED]
X-Priority: 3
X-MSMail-Priority: Normal
X-Mailer: Microsoft Outlook Express 6.00.2600.
X-MimeOLE: Produced By Microsoft MimeOLE V6.00.2600.
Sender: [EMAIL PROTECTED]
Precedence: bulk
Return-Path: [EMAIL PROTECTED]
X-OriginalArrivalTime: 05 Nov 2003 17:03:14.0593 (UTC) 
FILETIME=[B36DB110:01C3A3BE]

- Original Message -
From: Paulo Santa Rita [EMAIL PROTECTED]
To: [EMAIL PROTECTED]
Sent: Wednesday, November 05, 2003 2:04 PM
Subject: Re: [obm-l] Prova do IME

 Quem faz a questao 2, com solucao diferente da do GPI ? Para que todos
 possam participar, voces aceitam que uma pessoa so possa fazer uma 
questao
(
 nao duas ou mais ) ?

Questão:
P(x) = x^3 + ax + b (b  0) tem 3 raízes reais. Prove que a  0.

A solução do GPI usou as relações de Girard.

Aqui vai uma solução alternativa:
Se a = 0, então P(x) tem uma única raiz real, igual a (-b)^(1/3).
Se a  0, então P'(x) = 3x^2 + a  0, para todo x ==
P(x) é estritamente crescente ==
Como lim(x--inf) P(x) = -inf e lim(x - +inf) P(x) = +inf, P(x) tem uma
única raiz real.
Logo, só pode ser a  0.

Um abraço,
Claudio.
=
Instruções para entrar na lista, sair da lista e usar a lista em
http://www.mat.puc-rio.br/~nicolau/olimp/obm-l.html
=
_
MSN Messenger: converse com os seus amigos online.  
http://messenger.msn.com.br

=
Instruções para entrar na lista, sair da lista e usar a lista em
http://www.mat.puc-rio.br/~nicolau/olimp/obm-l.html
=


Re: [obm-l] Prova do IME

2003-11-06 Por tôpico Paulo Santa Rita
Oi Marcio,
Tudo legal comigo.  E voce, como vai ?
Obrigado. É, a prova foi mesmo ontem. O Prof morgado ja havia dito que o GPI 
disponibilizara a prova e as solucoes ( alias, as que vi estavam muito boas 
! ). Vou dar uma olhada nas do PONTO DE  ENSINO.

Como toda questao tem varias maneiras de solucao, tai um problema legal : 
olhar a solucao GPI, olhar a solucao PONTO DE ENSINO e entao apresentar, se 
possivel, uma maneira de resolver, diferente das dos dois cursos, mesmo que 
mais longa e/ou mais feia e/ou usando conhecimentos pouco divulgados e/ou 
usando tecnicas um pouquinho mais avancadas.

Para muitos de nos aqui desta lista, apresentar diversas solucoes para as 
questoes do IME e apenas uma ginastica mental. Nao representa merito ou 
valor algum. Mas para um aluno que se prepara para um tal concurso, e 
meritoso e saudavel buscar isso e todos devem ser estimulados neste sentido.

Os proprios cursos, para darem mais qualidade ao seu trabalho deveriam fazer 
isso, isto e, para cada questao, mostrar diversos caminhos validos de 
solucao, isto e, procederem a uma verdadeira analise do problema. fazendo 
assim acredito que eles provariam duas coisas : 1) a verdadeira qualidade da 
sua equipe de professores, 2) Que estao mais preocupados com o aluno.

O primeiro curso que fizer isso ( nao conheco algum que faca ) e registrar 
este trabalho num paper, vai verdadeiramente demonstrar sua qualidade e 
conquistar a confianca dos bons alunos.

A corrida para ser o primeiro a apresentar o gabarito e muito mais para 
atender ou a uma disputa entre-cursos que satisfaz interesses comercias de 
divulgacao, nao estando prioritariamente centrada aluno; ou o reflexo do 
preconceito infantil segundo o qual quem faz primeiro e o melhor; ou uma 
combinacao destas coisas.

Nos podemos fazer isso aqui nesta nossa lista.

Como trata-se de trabalho pesado, cada um aborda uma questao e faz uma 
analise exaustiva dela, mostrando  diversos caminhos ( comum sao muitos 
caminhos, alguns serao necessariamente omitidos. importa aqueles que os 
vestibulandos IME conhecem bem ) de solucao. Poderiamos fazer isso todo ano 
com as provas do IME, por exemplo. Seria uma contribuicao valiosa para a 
nossa lista e muito importante para os vestibulandos IME que nos assistem.

Se as pessoas gostarem da ideia e um bom numero se propor a colaborar, eu 
participo e inicio o trabalho. Podem me escrever em off neste sentido.

Um Grande Abraco a Todos
Paulo Santa Rita
5,1050,061103
From: [EMAIL PROTECTED]
Reply-To: [EMAIL PROTECTED]
To: [EMAIL PROTECTED]
Subject: Re: [obm-l] Prova do IME
Date: Wed, 5 Nov 2003 19:17:13 -0200
MIME-Version: 1.0
X-Originating-IP: [200.179.240.74]
Received: from mc2-f28.hotmail.com ([65.54.237.35]) by mc2-s18.hotmail.com 
with Microsoft SMTPSVC(5.0.2195.5600); Wed, 5 Nov 2003 13:21:51 -0800
Received: from sucuri.mat.puc-rio.br ([139.82.27.7]) by mc2-f28.hotmail.com 
with Microsoft SMTPSVC(5.0.2195.5600); Wed, 5 Nov 2003 13:20:11 -0800
Received: (from [EMAIL PROTECTED])by sucuri.mat.puc-rio.br (8.9.3/8.9.3) 
id TAA15246for obm-l-MTTP; Wed, 5 Nov 2003 19:17:48 -0200
Received: from smtp-37.ig.com.br (smtp-37.ig.com.br [200.226.132.188])by 
sucuri.mat.puc-rio.br (8.9.3/8.9.3) with ESMTP id TAA15241for 
[EMAIL PROTECTED]; Wed, 5 Nov 2003 19:17:47 -0200
Received: (qmail 32158 invoked from network); 5 Nov 2003 21:17:18 -
Received: from indesk-1.ig.com.br (HELO localhost) ([200.226.127.141])  
(envelope-sender [EMAIL PROTECTED])  by 
smtp-37.ig.com.br (qmail-ldap-1.03) with SMTP  for 
[EMAIL PROTECTED]; 5 Nov 2003 21:17:18 -
X-Message-Info: HQbIehuYceShQnGMYKHIqZ22XJlIdrR7iw+2/Xc2Hkw=
Message-Id: [EMAIL PROTECTED]
X-Mailer: InMail by Insite - www.insite.com.br
X-user: [EMAIL PROTECTED]
Sender: [EMAIL PROTECTED]
Precedence: bulk
Return-Path: [EMAIL PROTECTED]
X-OriginalArrivalTime: 05 Nov 2003 21:20:13.0077 (UTC) 
FILETIME=[998FA450:01C3A3E2]

_
MSN Messenger: converse com os seus amigos online.  
http://messenger.msn.com.br
---BeginMessage---
Oi Paulo, tudo bem? 
Ontem foi mesmo a prova de matematica do IME. Achei a prova bem legal 
por sinal. Voce pode ve-la em www.pensi.com.br . La tem inclusive o gabarito 
da prova. Uma opcao menos parcial eh o proprio site do ime: www.ime.eb.br . Eles costumam deixar a prova 
no site, mas nao sei se ja atualizaram. 
Abracos, 
Marcio 
 
Em 05 Nov 2003, [EMAIL PROTECTED] escreveu:  
Ola Pessoal ! 
 
Todos devem ter notado que novamente estamos sendo agredidos 
por um imbecil 
qualquer. Nao se responde a este tipo de gente. Vamos ignorar 
as mensagens 
ofensivas e trata-las como sao : lixo produzido por lixo. 
 
Se nao me engano, ontem foi a prova de Matematica do IME. 
Alguem tem a prova 
ou sabe onde encontra-la na Internet ? Seria interessante 
disponibiliza-la 
aqui na nossa lista, para que pudessemos discutir algumas 
questoes. 
 
Um Abraco a Todos ! 
Paulo Santa Rita 
4,1120,051103

Re: [obm-l] off-topic (fisica)

2003-11-10 Por tôpico Paulo Santa Rita
Ola Leonardo e demais
colegas desta lista ... OBM-L,
O nosso moderador, Prof Nicolau, autorizou discutirmos Fisica aqui. Assim, 
voce pode propor problemas de Fsica nesta lista. Talvez os Sites abaixo 
sejam do seu interesse :

http://www.sbfisica.org.br/olimpiada

http://www.fisica.ufc.br

Um Abraco a Todos
Paulo Santa Rita
2,1618,101103
From: leonardo mattos [EMAIL PROTECTED]
Reply-To: [EMAIL PROTECTED]
To: [EMAIL PROTECTED]
Subject: [obm-l] off-topic (fisica)
Date: Mon, 10 Nov 2003 03:26:35 +
MIME-Version: 1.0
X-Originating-IP: [200.141.99.2]
X-Originating-Email: [EMAIL PROTECTED]
Received: from mc5-f11.hotmail.com ([65.54.252.18]) by mc5-s21.hotmail.com 
with Microsoft SMTPSVC(5.0.2195.5600); Sun, 9 Nov 2003 19:28:49 -0800
Received: from sucuri.mat.puc-rio.br ([139.82.27.7]) by mc5-f11.hotmail.com 
with Microsoft SMTPSVC(5.0.2195.5600); Sun, 9 Nov 2003 19:28:48 -0800
Received: (from [EMAIL PROTECTED])by sucuri.mat.puc-rio.br (8.9.3/8.9.3) 
id BAA05517for obm-l-MTTP; Mon, 10 Nov 2003 01:27:09 -0200
Received: from hotmail.com (sea2-f37.sea2.hotmail.com [207.68.165.37])by 
sucuri.mat.puc-rio.br (8.9.3/8.9.3) with ESMTP id BAA05513for 
[EMAIL PROTECTED]; Mon, 10 Nov 2003 01:27:07 -0200
Received: from mail pickup service by hotmail.com with Microsoft SMTPSVC; 
Sun, 9 Nov 2003 19:26:35 -0800
Received: from 200.141.99.2 by sea2fd.sea2.hotmail.msn.com with HTTP;Mon, 
10 Nov 2003 03:26:35 GMT
X-Message-Info: TiNwL5K19MGoMoYs5DfU1JBOixdGunHWNNGvi0uEWHo=
Message-ID: [EMAIL PROTECTED]
X-OriginalArrivalTime: 10 Nov 2003 03:26:35.0954 (UTC) 
FILETIME=[72077920:01C3A73A]
Sender: [EMAIL PROTECTED]
Precedence: bulk
Return-Path: [EMAIL PROTECTED]

Ola a todos,
 Sera q algum de vcs conhece alguma lista de fisica q se asemelhe a 
nossa???

_
MSN Messenger: converse com os seus amigos online.  
http://messenger.msn.com.br

=
Instruções para entrar na lista, sair da lista e usar a lista em
http://www.mat.puc-rio.br/~nicolau/olimp/obm-l.html
=
_
MSN Messenger: converse com os seus amigos online.  
http://messenger.msn.com.br

=
Instruções para entrar na lista, sair da lista e usar a lista em
http://www.mat.puc-rio.br/~nicolau/olimp/obm-l.html
=


Re: [obm-l] off-topic (fisica)

2003-11-10 Por tôpico Paulo Santa Rita
Ola Leonardo e demais
colegas desta lista ... OBM-L,
Uma Lista de discussão de Problemas de Fisica como esta nossa Lista de 
discussão de problemas de Matematica eu não conheco, no Brasil. Mas o nosso 
moderador, Prof Nicolau, autorizou discutirmos aqui problemas de Fisica. 
Portanto, voce pode propor aqui problemas de desta disciplina.

Algo que acredito que sera do seu interesse e que TENHO CERTEZA pode te 
ajudar no estudo da fisica para o tipo de vestibular que voce voce encontra 
( em portugues ) em :


From: leonardo mattos [EMAIL PROTECTED]
Reply-To: [EMAIL PROTECTED]
To: [EMAIL PROTECTED]
Subject: [obm-l] off-topic (fisica)
Date: Mon, 10 Nov 2003 03:26:35 +
MIME-Version: 1.0
X-Originating-IP: [200.141.99.2]
X-Originating-Email: [EMAIL PROTECTED]
Received: from mc5-f11.hotmail.com ([65.54.252.18]) by mc5-s21.hotmail.com 
with Microsoft SMTPSVC(5.0.2195.5600); Sun, 9 Nov 2003 19:28:49 -0800
Received: from sucuri.mat.puc-rio.br ([139.82.27.7]) by mc5-f11.hotmail.com 
with Microsoft SMTPSVC(5.0.2195.5600); Sun, 9 Nov 2003 19:28:48 -0800
Received: (from [EMAIL PROTECTED])by sucuri.mat.puc-rio.br (8.9.3/8.9.3) 
id BAA05517for obm-l-MTTP; Mon, 10 Nov 2003 01:27:09 -0200
Received: from hotmail.com (sea2-f37.sea2.hotmail.com [207.68.165.37])by 
sucuri.mat.puc-rio.br (8.9.3/8.9.3) with ESMTP id BAA05513for 
[EMAIL PROTECTED]; Mon, 10 Nov 2003 01:27:07 -0200
Received: from mail pickup service by hotmail.com with Microsoft SMTPSVC; 
Sun, 9 Nov 2003 19:26:35 -0800
Received: from 200.141.99.2 by sea2fd.sea2.hotmail.msn.com with HTTP;Mon, 
10 Nov 2003 03:26:35 GMT
X-Message-Info: TiNwL5K19MGoMoYs5DfU1JBOixdGunHWNNGvi0uEWHo=
Message-ID: [EMAIL PROTECTED]
X-OriginalArrivalTime: 10 Nov 2003 03:26:35.0954 (UTC) 
FILETIME=[72077920:01C3A73A]
Sender: [EMAIL PROTECTED]
Precedence: bulk
Return-Path: [EMAIL PROTECTED]

Ola a todos,
 Sera q algum de vcs conhece alguma lista de fisica q se asemelhe a 
nossa???

_
MSN Messenger: converse com os seus amigos online.  
http://messenger.msn.com.br

=
Instruções para entrar na lista, sair da lista e usar a lista em
http://www.mat.puc-rio.br/~nicolau/olimp/obm-l.html
=
_
MSN Hotmail, o maior webmail do Brasil.  http://www.hotmail.com
=
Instruções para entrar na lista, sair da lista e usar a lista em
http://www.mat.puc-rio.br/~nicolau/olimp/obm-l.html
=


[obm-l] Mecanica Quantica

2003-11-14 Por tôpico Paulo Santa Rita
Ola a Todos,

Quem quiser ver uma excelente introducao ao formalismo da Mecanica Quantica, 
inclusive com uma breve ( porem clara ) exposicao dos seus fundamentos 
matematicos ( espacos de Hilbert ) e as discussões associadas a estes 
fundamentos, olhe em :

http://www.ifi.unicamp.br/tesesonline/teses/IF671.pdf

De maneira geral, ha teses muito boas ali. Talvez seja necessário fazer um 
cadastramento previo.

No trabalho especifico que estou indicando, há uma obordagem bastante lucida 
do Teorema de Bell. Ali voce tambem vai poder verificar que a base 
Matematica para se entender a Mecanica Quantica de forma alguma e dificil, 
resumindo-se, em ultima analise, as Espacos de Hilbert ( Espacos Vetorias 
com produto interno completo em relacao a norma )

Um Abraco a Todos
Paulo Santa Rita
6,1132,141103
OBS : O Teorema de Bell e uma das maiores conquistas humanas do seculo 
passado e a sua compreensao basicamente não exige conhecimento previo algum. 
Qualquer pessoa pode entende-lo.

_
MSN Hotmail, o maior webmail do Brasil.  http://www.hotmail.com
=
Instruções para entrar na lista, sair da lista e usar a lista em
http://www.mat.puc-rio.br/~nicolau/olimp/obm-l.html
=


Re: [obm-l] Mecanica Quantica

2003-11-15 Por tôpico Paulo Santa Rita
Ola a Todos !

Use o endereco abaixo :

www.ifi.unicamp.br/ccjdr/teses/apresentacao.php3?filename=IF671

A seguir, faca o download do arquivo.

Um Abraco
Paulo Santa Rita
7,1927,151103
From: Domingos Jr. [EMAIL PROTECTED]
Reply-To: [EMAIL PROTECTED]
To: [EMAIL PROTECTED]
Subject: Re: [obm-l] Mecanica Quantica
Date: Fri, 14 Nov 2003 16:19:59 -0200
MIME-Version: 1.0
Received: from mc5-f7.hotmail.com ([65.54.252.14]) by mc5-s19.hotmail.com 
with Microsoft SMTPSVC(5.0.2195.6713); Fri, 14 Nov 2003 10:23:56 -0800
Received: from sucuri.mat.puc-rio.br ([139.82.27.7]) by mc5-f7.hotmail.com 
with Microsoft SMTPSVC(5.0.2195.6713); Fri, 14 Nov 2003 10:22:54 -0800
Received: (from [EMAIL PROTECTED])by sucuri.mat.puc-rio.br (8.9.3/8.9.3) 
id QAA01542for obm-l-MTTP; Fri, 14 Nov 2003 16:20:10 -0200
Received: from smtp.uol.com.br (smtp.uol.com.br [200.221.11.52])by 
sucuri.mat.puc-rio.br (8.9.3/8.9.3) with ESMTP id QAA01538for
[EMAIL PROTECTED]; Fri, 14 Nov 2003 16:20:09 -0200
Received: from gauss (200-158-97-231.dsl.telesp.net.br [200.158.97.231])by 
scorpion4.uol.com.br (Postfix) with SMTP id 0CBD1D8FEfor 
[EMAIL PROTECTED]; Fri, 14 Nov 2003 16:19:37 -0200 (BRST)
X-Message-Info: TiNwL5K19MEFK830b0Uf3HNnmYM2ed92sfTRoUCss+k=
Message-ID: [EMAIL PROTECTED]
References: [EMAIL PROTECTED]
X-Priority: 3
X-MSMail-Priority: Normal
X-Mailer: Microsoft Outlook Express 6.00.2800.1158
X-MimeOLE: Produced By Microsoft MimeOLE V6.00.2800.1165
Sender: [EMAIL PROTECTED]
Precedence: bulk
Return-Path: [EMAIL PROTECTED]
X-OriginalArrivalTime: 14 Nov 2003 18:22:55.0710 (UTC) 
FILETIME=[52E9FFE0:01C3AADC]

Não consegui baixar! onde tem que se cadastrar?

- Original Message -
From: Paulo Santa Rita [EMAIL PROTECTED]
To: [EMAIL PROTECTED]
Sent: Friday, November 14, 2003 11:34 AM
Subject: [obm-l] Mecanica Quantica
=
Instruções para entrar na lista, sair da lista e usar a lista em
http://www.mat.puc-rio.br/~nicolau/olimp/obm-l.html
=

_
MSN Hotmail, o maior webmail do Brasil.  http://www.hotmail.com
=
Instruções para entrar na lista, sair da lista e usar a lista em
http://www.mat.puc-rio.br/~nicolau/olimp/obm-l.html
=


[obm-l] Principio implicito na Teoria da Relatividade

2003-11-15 Por tôpico Paulo Santa Rita
Ola a todos !

Para uma boa compreensao de como o PRINCIPIO DA ANTECEDENCIA DAS CAUSAS esta 
implicitamente pressuposto ( gratuitamente ) na Teoria da Relatividade, uma 
exposicao lucida pode ser vista em :

http://ghtc.ifi.unicamp.br/pdf/ram-29.pdf

Um abraco aTodos
Paulo Santa Rita
7,2018,151103
_
MSN Hotmail, o maior webmail do Brasil.  http://www.hotmail.com
=
Instruções para entrar na lista, sair da lista e usar a lista em
http://www.mat.puc-rio.br/~nicolau/olimp/obm-l.html
=


RE: [obm-l] Principio implicito na Teoria da Relatividade

2003-11-16 Por tôpico Paulo Santa Rita
 isso ? Significa que se eu formular uma lei fisica tal 
que fazendo uma mudanca
de referencial, essa lei mude ou perca a validade, entao eu nao formulei uma 
lei fisica ou a minha
lei fisica esta errada. Ora, um Fisico, crente neste fato, vai buscar na 
Matematica algo que corresponda
e exprima RIGOROSAMENTE esta percepcao. Na Matematica, isso ocorre com toda  
formulacao
COVARIANTE. Assim, o Fisico fala : TODA LEI FISICA TEM QUE TER UMA 
FORMULACAO
COVARIANTE, caso contrario, nao e a formulacao de uma lei fisica.

Assim :

( Fisica ) Mesma Lei em todos os referenciais = ( Matematica ) Formulacao 
Covariante

Isto e :

Ao percebemos ( com a intuicao ) uma qualidade do mundo buscamos na 
Matematica uma
a forma adeguada e rigorosa de falarmos aquilo que percebemos.

E neste contexto que se insere a sua pergunta. De fato, e conveniente 
representarmos o tempo por
um complexo.

Einstein mostrou que o tempo nao e absoluto, isto e, que existe o tempo 
proprio de cada referencial.
Assim, quando OCORRE ALGUMA COISA, um evento, ele ocorre num determinado 
instante PARA
UM REFERENCIAL ESPECIFICO, quero dizer, ele ocorre em UM LUGAR ( TRES 
COORDENADAS )
e num determinado INSTANTE ( QUARTA COORDENADA ). Em Newton, a quarta 
coordenada, o tempo,
era igual para todos, nao havendo o problema de especifica-lo quando 
quisessemos falar de um
determinado fato. Em Einstein ha esse problema e portanto precisamos dizer 
EM QUE INSTANTE
o fato ocorre. Essas quatro coordenadas sao, evidentemente, um espaco 
quadri-dimensional e precisamos
passar esses quatro numeros ao falarmos sobre algo.

Um ponto neste espaco e chamado PONTO DE UNIVERSO e a cada particula em 
consideracao
corresponde uma LINHA DE UNIVERSO. Para simplificar, vou imaginar que as 
tres coordenadas
espaciais estao sintetizadas em um unico eixo ( o eixo das abscissas ). O 
eixo das ordenadas sera
o eixo dos tempos.

Exemplo : Qual a linha de Universo de um corpo em repouso ? Claramente uma 
reta da forma
X = r ... O tempo vai passando mais ela nao sai do lugar. Em geral, a linha 
de universo de um
corpo comum e uma funcao continua, pois ele muda continuamente de posicao e 
o tempo passa
continuamente por ele

E conveniente representarmos o tempo por um numero complexo porque isso nos 
permite tratar com
maior clareza problemas de singularidades temporais e paradoxos, mas nao ha 
uma exigencia quanto
a isso. Para ver isso, considere um sinal a velocidade da luz que parte de 
X1,Y1,Z1 no instante
T1 e se dirige para o local X2,Y2,Z2 que ocorrera no instante T2. Como a 
velocidade e C ( velocidade
da luz ), entao, a distancia percorrida e :

C (T2 - T1 )

Mas, esta mesma distancia, usando a metrica euclidiana, e :

raiz( X1-X2)^2 + (Y1-Y2)^2 + (Z1-Z2)^2 ) . Isto e :
C(T2 - T1 ) = raiz( X1-X2)^2 + (Y1-Y2)^2 + (Z1-Z2)^2 )
(X1-X2)^2 + (Y1-Y2)^2 + (Z1 -Z2)^2 - (C^2)(T2 - T1)^2 = 0
Se estivermos tratando de quaiquer dos eventos, o numero ( em Relatividade 
Geral ) :
S12 = raiz (  (X1-X2)^2 + (Y1-Y2)^2 + (Z1 -Z2)^2 - (C^2)(T2 - T1)^2  ) e 
chamado
INTERVALO entre os dois eventos. Se adotarmos a convencao de chamarmos de Ta 
(tau)
o valor iCT, isto e : Ta=iCt, entao a formulacao diferencial do arco da 
linha de universo
ficara como :

ds^2 = - (dx^2 + dy^2 + dz^2 + dTa^2 )

E a essa grandeza Ta = iCt que NOS PERMITE DAR UM TRATAMENTO TRANQUILO
ao comprimento de arco na linha do universo que leva muitos fisicos a 
dizerem que o tempo
pode ser representado como um numero complexo. Observe, entretando, que isto 
e apenas
uma forma inteligente de obtermos, para dois pontos de universos, uma 
formulacao do comprimento
de arco equivalente ao valor bem conhecido da analise matematica :

ds^2 = dx^2 + dy^2

Assim, respondendo a sua pergunta : realmente, o tempo ( ou a grandeza Ta 
relacionada com o
tempo tal como expliquei ) e convenientemente expresso por um numero 
imaginario porque isso
facilita enormemente a expressao diferencial das leis relativisticas ( da 
relatividade geral ). Em particular,
o comprimento de arco numa linha de universo, o que da origem a um 
tratamento facilitado de
outrso aspectos.

Gostaria de ver voce abordando um assunto que sempre me empolgou.

Alguem ( Nao me lembro o nome agora ) definiu os Espacos Metricos e a 
consequente definicao
de Metrica, com aquelas tres propriedades que todo mundo sabe. Por que 
aquelas tres
propriedades ? Por que nao 4 ? Por que exatamente aquelas tres ? Em que 
sentido aqueles
leis atendem de forma satisfatoria as nossas necessidades ? Por que nao e, 
por exemplo :

D(X,Y)  0
D(X,Y) = D(Y,X)
D(X,Y) = 2D(X,Z) + 3D(Z,Y)
?

Um Grande Abraco a Todos
Um Abraco especial ao Artur
Paulo Santa Rita
1,2300,161103
From: Artur Coste Steiner [EMAIL PROTECTED]
Reply-To: [EMAIL PROTECTED]
To: [EMAIL PROTECTED]
Subject: RE: [obm-l] Principio implicito na Teoria da Relatividade
Date: Sat, 15 Nov 2003 21:06:08 -0200
MIME-Version: 1.0
Received: from mc1-f40.hotmail.com ([64.4.50.47]) by mc1-s10.hotmail.com 
with Microsoft SMTPSVC

Re: [obm-l] O problema do camelo

2003-11-18 Por tôpico Paulo Santa Rita
Ola Rogerio de demais
colegas desta lista ... OBM-L,
O que eu deve entender por ele deve beber ( continuamente ) um litro de 
agua por quilometro ?

Vou supor que o Oasis e o marco zero ( zero quilometro ).

IMAGINE que o camelo esta no Oasis. Ele e entao carregado com 100 litros 
agua. Ao atingir o marco 1, ele andou 1 quilometro e, portanto, vai beber 1 
litro de agua.  Ao atingir o marco 2, bebe mais um litro. Sobram entao 98 
litros dos 100 litros com que ele partiu. Ele deixa 97 no marco 2 e volta. 
Ao atingir o marco 1, bebe o ultimo litro de que dispoe. Andando mais um 
kilometro ele chega ao Oasis, onde ha agua em abundancia e, portanto, bebe 
um litro desta agua.

Assim, saindo com N litros do Oasis, N = 100, ele pode deixar 100 - 2K + 1 
litros no marco K
( K = 50 ) e o Oasis ficou reduzido em 101 litros de agua.  Como ha agua em 
abundancia no Oasis, repetindo esta operacao um grande numero de vezes ele 
pode colocar ate um Oceano de Agua no marco K, isto e, a partir de um 
certo momento ele nao precisa mais voltar ao oasis original ... Ele vai 
poder partir sempre do marco K.

Mas nos queremos o minimo de agua que deve ter no oasis original. Seja M 
esse minimo. Posso portanto supor, com base na observacao acima, que apos um 
numero finito de vezes, R,  o oasis foi deslocado para o marco K, isto e, 
no marco K ha M - 101R ( para algum R natural e supondo que ele sempre 
parte com 100 litros ) o oasis original estara vazio e, portanto, o camelo 
nao deve e nao precisa voltar ao oasis original.

E possivel usar esta estrategia ? Ou o camelo sempre precisa voltar ate o 
Oasis original ?
Nao esta claro isto no texto !

Existe um outro problema. O que e beber continuamente ?

Suponha que o camelo parte com 100 litros de agua e vai ate o ponto 
raiz_quadrada(2). Devo supor que ele bebeu raiz_quadrada(2) litros de agua ? 
Neste caso ao atingir o marco K ( K inteiro ) e voltar ele dixa 100 - 2K, 
consumindo 2k litros de agua, isto e, quando, na volta, ele atingir o oasis, 
ele ja consumiu 2K litros de agua e  nao, como parece, 2K-1. Note que, neste 
caso, precisamos supor alguma coisa sobre a forma do caminho que liga o 
oasis ao sindicato, pois, bebendo continuamente, ele vai beber menos se a 
ligacao oasis-sindicato for um segmento de reta ...

Existe um outro problema. O que e ele pode deixar depositos de agua em 
qualquer lugar do caminho ?

O camelo so pode deixar agua em marcos quilometricos inteiros ? ou, por 
exemplo, ele pode se dirigir uma posicao R, R real, depositar 100 - 2R de 
agua ali. Neste caso absolutamente continuo, isto e, onde o camelo bebe 
continuamente e pode depositar agua em qualquer posicao real, me parece que 
e melhor substituir o camelo ...

SALVO UM MELHOR JUIZO, que eu apreciaria ver, me parece que o problema quer 
usar um detalhe matematico que nao se coaduna convenientemente com o contexo 
usado ou foram admitidos pressupostos que nao ficaram suficientemente claro 
no enunciado.

O problema e bonito e engenhoso e imaginar uma forma de levar 1000 litros 
ate a posicao 1000 e bastante facil, mesmo trivial. Mas determinar uma 
estrategia otima no sentido de consumir uma quantidade minima de agua nao me 
parece um problema simples, sobretudo porque nao esta claro quais 
pressupostos podemos admitir ...



From: Rogerio Ponce [EMAIL PROTECTED]
Reply-To: [EMAIL PROTECTED]
To: [EMAIL PROTECTED]
Subject: [obm-l] O problema do camelo
Date: Sun, 16 Nov 2003 21:18:44 +
MIME-Version: 1.0
X-Originating-IP: [200.214.109.236]
X-Originating-Email: [EMAIL PROTECTED]
Received: from mc2-f16.hotmail.com ([65.54.237.23]) by mc2-s1.hotmail.com 
with Microsoft SMTPSVC(5.0.2195.6713); Sun, 16 Nov 2003 13:20:12 -0800
Received: from sucuri.mat.puc-rio.br ([139.82.27.7]) by mc2-f16.hotmail.com 
with Microsoft SMTPSVC(5.0.2195.6713); Sun, 16 Nov 2003 13:20:11 -0800
Received: (from [EMAIL PROTECTED])by sucuri.mat.puc-rio.br (8.9.3/8.9.3) 
id TAA16393for obm-l-MTTP; Sun, 16 Nov 2003 19:19:17 -0200
Received: from hotmail.com (bay9-f38.bay9.hotmail.com [64.4.47.38])by 
sucuri.mat.puc-rio.br (8.9.3/8.9.3) with ESMTP id TAA16388for 
[EMAIL PROTECTED]; Sun, 16 Nov 2003 19:19:15 -0200
Received: from mail pickup service by hotmail.com with Microsoft SMTPSVC; 
Sun, 16 Nov 2003 13:18:44 -0800
Received: from 200.214.109.236 by by9fd.bay9.hotmail.msn.com with HTTP;Sun, 
16 Nov 2003 21:18:44 GMT
X-Message-Info: TiNwL5K19MHsk4VxzSki9pnCOmcwpv/nq0oFfSMx1Cw=
Message-ID: [EMAIL PROTECTED]
X-OriginalArrivalTime: 16 Nov 2003 21:18:44.0576 (UTC) 
FILETIME=[375AB600:01C3AC87]
Sender: [EMAIL PROTECTED]
Precedence: bulk
Return-Path: [EMAIL PROTECTED]

Repassando o problema do camelo...

Um camelo deve fazer uma entrega de 1000 litros de água ao Sindicato dos 
Beduínos, que fica a 1000 km de distância de seu oásis de partida. O camelo 
pode carregar até 100 litros de água e deve beber (continuamente) 1 litro 
de água por quilômetro. Ele pode deixar depósitos de água em qualquer ponto 
do caminho. De quanta água (no 

Re: [obm-l] O problema do camelo

2003-11-18 Por tôpico Paulo Santa Rita
Ola Pessoal !

Leia a minha mensagem que voce vai ver que em nenhum momento eu falei em 
andar 3 Km. Mas, tudo bem. Entendi o que voce quer. Obrigado.

Um abraco
Paulo Santa Rita
3,1450,181103
From: Rogerio Ponce [EMAIL PROTECTED]
Reply-To: [EMAIL PROTECTED]
To: [EMAIL PROTECTED]
Subject: Re: [obm-l] O problema do camelo
Date: Tue, 18 Nov 2003 14:50:57 +
MIME-Version: 1.0
X-Originating-IP: [200.244.74.46]
X-Originating-Email: [EMAIL PROTECTED]
Received: from mc7-f4.hotmail.com ([65.54.253.11]) by mc7-s15.hotmail.com 
with Microsoft SMTPSVC(5.0.2195.6713); Tue, 18 Nov 2003 06:55:09 -0800
Received: from sucuri.mat.puc-rio.br ([139.82.27.7]) by mc7-f4.hotmail.com 
with Microsoft SMTPSVC(5.0.2195.6713); Tue, 18 Nov 2003 06:54:34 -0800
Received: (from [EMAIL PROTECTED])by sucuri.mat.puc-rio.br (8.9.3/8.9.3) 
id MAA21367for obm-l-MTTP; Tue, 18 Nov 2003 12:51:36 -0200
Received: from hotmail.com (bay9-f7.bay9.hotmail.com [64.4.47.7])by 
sucuri.mat.puc-rio.br (8.9.3/8.9.3) with ESMTP id MAA21363for 
[EMAIL PROTECTED]; Tue, 18 Nov 2003 12:51:32 -0200
Received: from mail pickup service by hotmail.com with Microsoft SMTPSVC; 
Tue, 18 Nov 2003 06:50:57 -0800
Received: from 200.244.74.46 by by9fd.bay9.hotmail.msn.com with HTTP;Tue, 
18 Nov 2003 14:50:57 GMT
X-Message-Info: TiNwL5K19MHf1feKAmjG+8cQq2J4R9ZCeusC9BX42Hk=
Message-ID: [EMAIL PROTECTED]
X-OriginalArrivalTime: 18 Nov 2003 14:50:57.0416 (UTC) 
FILETIME=[5FDF5880:01C3ADE3]
Sender: [EMAIL PROTECTED]
Precedence: bulk
Return-Path: [EMAIL PROTECTED]

Olá Paulo,
a estratégia de andar 3 km e deixar 97 litros não funciona, pois o camelo 
precisa de água para voltar.
Talvez fique mais fácil pensar no problema da forma como eu o repassei para 
alguns amigos :



Uma base militar precisa levar 1000 litros de
gasolina para um posto avançado no deserto,
situado a 1000 km de distância da base.
Será usado um jipe , cujo tanque tem a capacidade
de 100 litros, e que consome 1 litro por quilômetro.
Ele pode deixar depósitos de gasolina em qualquer
ponto do caminho.
De quantos litros, no mínimo, ele precisará para
cumprir sua missão?o jipe leva no máximo 100 litros , porra !
E mais : ele não precisa voltar. A missão é entregar somente.
Para isso , ele terá que sucessivas vezes ir e voltar , deixando
depósitos ao longo do caminho, até conseguir, na última ida,
completar a missão.
-- Observações que fiz na outra lista após repassar o problema 
:

O jipe leva no máximo 100 litros !
E mais : ele não precisa voltar. A missão é entregar somente.
Para isso , ele terá que sucessivas vezes ir e voltar , deixando
depósitos ao longo do caminho, até conseguir, na última ida,
completar a missão.
A gasolina que o jipe deixa é a sobra tirada do seu próprio tanque.
Assim, ele poderia partir da base com tanque cheio, deixar 98 litros
a 1 km da mesma , e voltar , por exemplo.
[]´s
Rogério.
PS: O número é astronômico, com certeza. Li esse problema ontem, e
não sei ainda qual a solução.
_
MSN Hotmail, o maior webmail do Brasil.  http://www.hotmail.com
=
Instruções para entrar na lista, sair da lista e usar a lista em
http://www.mat.puc-rio.br/~nicolau/olimp/obm-l.html
=
_
MSN Messenger: converse com os seus amigos online.  
http://messenger.msn.com.br

=
Instruções para entrar na lista, sair da lista e usar a lista em
http://www.mat.puc-rio.br/~nicolau/olimp/obm-l.html
=


<    1   2   3   4   5   6   7   >